Location via proxy:   [ UP ]  
[Report a bug]   [Manage cookies]                

JAWETZ End Chapter Questions

Download as pdf or txt
Download as pdf or txt
You are on page 1of 84

Chapter I: The Science of Microbiology 8.

A 16-year-old female patient presented to her family


physician with a complaint of an abnormal vaginal
Review Questions discharge and pruritus (itching). The patient denied
1. Which one of the following terms characterizes the having sexual activity and recently completed a course of
interaction between herpes simplex virus and a human? doxycycline for the treatment of her acne. An
(A) Parasitism examination of a Gram-stained vaginal smear revealed
(B) Symbiosis the presence of gram-positive oval cells about 4–8 μm in
(C) Endosymbiosis diameter. Her vaginitis is caused by which of the
(D) Endoparasitism following agents?
(E) Consortia (A) Bacterium
(B) Virus
2. Which one of the following agents lacks nucleic acid? (C) Protozoa
(A) Bacteria (D) Fungus
(B) Viruses (E) Prion
(C) Viroids
(D) Prions 9. A 65-year-old man develops dementia, progressive
(E) Protozoa over several months, along with ataxia and somnolence.
An electroencephalographic pattern shows paroxysms
3. Which one of the following is a prokaryote? with high voltages and slow waves, suggestive of
(A) Bacteria Creutzfeldt-Jakob disease (CJD). By which of the
(B) Algae following agents is this disease caused?
(C) Protozoa (A) Bacterium
(D) Fungi (B) Virus
(E) Slime molds (C) Viroid
(D) Prion
4. Which one of the following agents simultaneously (E) Plasmid
contains both DNA and RNA?
(A) Bacteria 10. Twenty minutes aſter ingesting a raw clam, a 35-year-
(B) Viruses old man experiences paresthesias of the mouth and
(C) Viroids extremities, headache, and ataxia. These symptoms are
(D) Prions the result of a neurotoxin produced by algae called
(E) Plasmids (A) Amoeba
(B) Blue-green algae
5. Which of the following cannot be infected by viruses? (C) Dinoflagellates
(A) Bacteria (D) Kelp
(B) Protozoa (E) None of the above
(C) Human cells
(D) Viruses
(E) None of the above

6. Viruses, bacteria, and protists are uniquely


characterized by their respective size. True or false?
(A) True (B) False

7. Quorum sensing in prokaryotes involves


(A) Cell–cell communication
(B) Production of molecules such as acetylated
homoserine lactone (AHL)
(C) An example of multicellular behavior
(D) Regulation of genes involved in diverse physiologic
processes
(E) All of the above
Chapter II: Cell Structure (D) Flagella
(E) Pili
Review Questions
1. A 22-year-old man presents with a painless 1-cm ulcer 6. Group A streptococci are the most common bacterial
cause of pharyngitis in school-age children 5–15 years of
on the shaſt of his penis. Inguinal lymphadenopathy is
age. The most important cell component involved in
present. The patient admits trading drugs for sex and has adherence of this bacteria to fibronectin, which covers
several sexual partners. An RPR test result is positive, and the epithelial surface of the nasopharynx is
syphilis is suspected; however, a Gram stain of a swab (A) Capsule
specimen from the ulcer shows no bacteria. Treponema (B) Lipoteichoic acid
pallidum, the causative agent of syphilis, cannot be (C) Flagella
visualized by light microscopy because (D) Lipoprotein
(A) It is transparent. (E) O-antigen
(B) It cannot be stained by ordinary stains.
(C) It has a diameter of less than 0.2 μm 7. In the fall of 2001, a series of letters containing spores
(D) The wavelength of white light is too long. of Bacillus anthracis were mailed to members of the
(E) Rapid movement of the organism prevents media and to U.S. Senate offices. The result was 22 cases
visualization. of anthrax, with five deaths. The heat resistance of
bacterial spores, such as those of Bacillus anthracis, is
2. Chloramphenicol, an antibiotic that inhibits bacterial partly attributable to their dehydrated state and partly
protein synthesis, will also affect which of the following to the presence of large amounts of
eukaryotic organelles? (A) Diaminopimelic acid
(A) Mitochondria (B) d-Glutamic acid
(B) Golgi complex (C) Calcium dipicolinate
(C) Microtubules (D) Sulfydryl-containing proteins
(D) Endoplasmic reticulum (E) Nuclear membrane (E)Lipid A

3. Which of the following structures is not part of the 8. Which of the following terms does NOT describe the
bacterial cell envelope? bacterial chromosome?
(A) Peptidoglycan (A) Haploid
(B) Lipopolysaccharide (B) Diploid
(C) Capsule (C) Circular
(D) Gas vacuole (D) Nucleoid
(E) S-layer (E) Feulgen positive

4. A group of teenagers became ill with nausea, vomiting, 9. Lysozyme cleaves the β1→4 linkage between
severe abdominal cramps, and diarrhea aſter eating (A) d-Alanine and the pentaglycine bridge
undercooked hamburgers from a local restaurant. Two of (B) N-Acetylmuramic acid and d-alanine
the teenagers were hospitalized with hemolytic-uremic (C) Lipid A and KDO
syndrome. Escherichia coli O157:H7 was isolated from (D) N-Acetylmuramic acid and N-acetylglucosamine
the patient’s stools as well as from uncooked (E) d-Alanine and d-alanine
hamburgers. The H7 refers to which bacterial structure?
(A) Lipopolysaccharide 10. Mycoplasma species lack which of the following
(B) Capsule components?
(C) Flagella (A) Ribosomes
(D) Fimbriae (B) Plasma membrane
(E) S-layer (C) Both DNA and RNA
(D) Lipids
5. Which of the following components is present in gram- (E) Peptidoglycan
negative bacteria but not in gram-positive bacteria?
(A) Peptidoglycan
(B) Lipid A
(C) Capsule
Chapter III: Classification of Bacteria (C)Halophiles
(D) Termophiles
Review Questions (E) Chemolithotrophs
1. Eubacteria that lack cell walls and do not synthesize
the precursors of peptidoglycan are called 6.A bacteria with a genome having a G + C content of 45%
(A)Gram-negative bacteria harbors a plasmid encoding a gene with a G + C content
(B)Viruses of 55%. Which one of the following conclusion could be
(C)Mycoplasmas drawn?
(D)Serovar variant (A)Tis gene codes for a cell wall peptidyl transferase
(E)Bacilli (B)Tis gene codes for a critical bacterial cytochrome
(C)Tis gene codes for a unique transfer RNA
2. Archaebacteria can be distinguished from eubacteria (D)Tis gene codes for a plasmid RNA-dependent DNA-
by their lack of polymerase
(A)DNA (E)Tis gene codes for an antigenically diverse capsular
(B)RNA polysaccharide
(C)Ribosomes
(D)Peptidoglycan
(E)Nucleus

3. A 16-year-old cystic fibrosis patient is admitted to the


hospital. A sputum culture yields Burkholderia cepacia.
Subsequently, there are two other patients with B
cepacia bacteremia, and the organism is cultured from
the sputum of four additional patients. During this
nosocomial outbreak of B cepacia, 50 environmental and
7 patient isolates are being subtyped to identify the
source of the outbreak. Which of the following
techniques would be most useful in this endeavor?
(A)Culture
(B)Ribotyping
(C)16S rRNA sequencing
(D)Antimicrobial susceptibility testing
(E)Nucleic acid sequencing

4. An unculturable gram-positive microorganism has


been visualized in tissue specimens obtained from
patients with a previously undescribed disease. Which of
the following techniques would be most useful in
identifying this organism?
(A)Serology
(B)PCR amplification and sequencing of rRNA genes
(C)Multilocus enzyme electrophoresis
(D)SDS-polyacrylamide gel electrophoresis
(E)Pulsed field gel electrophoresis

5. The DNA polymerase from Termus aquaticus is an


important component of DNA amplification methods
such as the polymerase chain reaction. Tis organism is
capable of growing at temperatures above 100°C.
Organisms that are capable of growth at these
temperatures are referred to as
(A)Mesophiles
(B)Psychrophiles
Chapter IV: Growth, Survival and Death of corresponds to a growth rate constant in vitro of (κ) of
Microorganisms 0.04 h-1. Estimating that the biomass of this single
mycobacterial organism is 2.3 × 10−13 g and assuming
Review Questions that this organism is immediately going to enter log
1. A 23-year-old woman has 10 Escherichia coli phase growth, how many hours will it take to produce
inoculated into her bladder while having sex. Tis 10−6 g of biomass?
(A)4 hours
organism has a generation time of 20 minutes. Aſter a lag (B)40 hours
of 20 minutes, the E coli enter the logarithmic phase of (C)400 hours
growth. Aſter 3 hours of logarithmic growth, the total (D)4000 hours
number of cells is (E)40,000 hours
(A)2560
(B)5012 6. A specimen of pasteurized goat’s milk is cultured for
(C)90 the presence of Brucella melitensis, an organism known
(D)1028 to infect animals on an adjacent farm. The milk is
(E)1,000,000 declared safe to consume; however, some of those who
consume it develop brucellosis. Which one of the
2. A 73-year-old woman is admitted to the hospital for following would best explain the disparity between the
intravenous treatment of an abscess caused by culture results and the patients’ illnesses?
Staphylococcus aureus. Subsequent to her treatment (A)Bacteria in the milk were viable but not cultivatable
and discharge from the hospital, it is necessary to (B)Incomplete pasteurization of the milk
disinfect the hospital room. One thousand of the S (C)The organisms in the milk were in the lag phase when
tested
aureus cells are exposed to a disinfectant. Aſter 10
(D)The milk had a high level of a bactericidal antibiotic in
minutes, 90% of the cells are killed. How many cells it when tested
remain viable aſter 20 minutes? (E) There was posttest contamination of the milk
(A) 500
(B) 100 7. Working as a medical missionary in rural India, you
(C) 10 spray the umbilicus of a newly born infant with a solution
(D) 1 containing the chemical structure in the associated
(E) 0 figure to prevent a tetanus infection. Which class of
chemical agent does this structure belong?
3. The action of which of the following agents or
processes on non-spore forming bacteria can be
reversed?
(A) A disinfectant
(B) A bactericidal agent
(C) A bacteriostatic agent
(D) Autoclaving at 121°C for 15 minutes
(E) Dry heat at 160–170°C for 1 hour (A)Alcohol class
(B)Aldehyde class
4.The growth rate of bacteria during the exponential (C)Bisphenol class
phase of growth is (D)Peroxygen class
(A)Zero (E)Quaternary ammonium class
(B)Increasing
(C)Constant 8. Your superior requests that you sterilize some surgical
(D)Decreasing instruments. Which one of the following agents would
(E)Negative you use?
(A) Benzoic acid (2%)
5. A physician obtains a sputum specimen from a patient (B) Isopropyl alcohol (2%)
suffering from tuberculosis. Tis sputum sample contains (C) Glutaraldehyde (2%)
one viable Mycobacterium tuberculosis, an organism (D) Hydrogen peroxide (2%)
with a slow doubling time in vitro of 48 hours, which (E) Quaternary ammonium compound (2%)
9. The growth rate of bacteria during the maximum Chapter V: Cultivation of Microorganisms
stationary phase of growth is
(A)Zero Review Questions
(B)Increasing 1. Most microorganisms pathogenic for humans grow
(C)Constant best in the laboratory when cultures are incubated at
(D)Decreasing (A) 15–20°C
(E)Negative (B) 20–30°C
(C) 30–37°C
10. Chemical agents can interfere with the normal (D) 38–50°C
reaction between a specific enzyme and its substrate (E) 50–55°C
(chemical antagonism). Which one of the following
inhibits energy-yielding cellular processes? 2. The process by which microorganisms form ATP during
(A)5-Methyltryptophan the fermentation of glucose is characterized by
(B)Cyanide (A)Coupling of ATP production with the transfer of
(C)Hydrogen peroxide electrons
(D)Ethanol (B)Denitrification
(E)Lysozyme (C)The reduction of oxygen
(D)Substrate phosphorylation
11. Which of the following is the most resistant to (E)Anaerobic respiration
destruction by chemicals and heat?
(A)Spores of Aspergillus fumigatus 3. The major effect of a temperature of 60oC on the
(B)Mycobacterium tuberculosis growth of a mesophile such as Escherichia coli is to
(C)Ebola virus (A)Destroy the cell wall
(D)E coli (B)Denature proteins
(E)Spores of Bacillus anthracis (C)Destroy nucleic acids
(D)Solubilize the cytoplasmic membrane
(E)Cause formation of endospores

4. Polymerization of building blocks (eg, amino acids) into


macromolecules (eg, proteins) is achieved largely by
(A)Dehydration
(B)Reduction
(C)Oxidation
(D)Assimilation
(E)Hydrolysis

5. A strain of E coli does not require vitamins when grown


in a defined medium consisting of glucose, mineral salts,
and ammonium chloride. This is because E coli
(A)Does not use vitamins for growth
(B)Obtains vitamins from its human host
(C)Is a chemoheterotroph
(D)Can synthesize vitamins from the simple compounds
provided in the medium
(E)Ammonium chloride and mineral salts contain trace
amounts of vitamins

6. Which of the following is NOT a mechanism for


generating metabolic energy by microorganisms?
(A)Fermentation
(B)Protein synthesis
(C)Respiration
(D)Photosynthesis
(E)C and D Chapter VI: Microbial Metabolism

7. Which of the following terms best describes a Review Questions


microorganisms that grows at 20°C? 1. The synthesis of which of the following cell
(A)Neutralophile components is dependent on a template?
(B)Psychrotroph (A)Lipopolysaccharide
(C)Mesophile (B)Peptidoglycan
(D)Osmophile (C)Capsular polysaccharide
(E)Termophile (D)Deoxyribonucleic acid
(E)Phospholipids
8. The ability to assimilate N2 reductively via NH3 is
called 2. The synthesis of which of the following cell
(A)Ammonification components is determined entirely by enzyme
(B)Anammox specificities?
(C)Assimilatory nitrate reduction (A)DNA
(D)Deamination (B)Ribosomal RNA
(E)Nitrogen fixation (C)Flagella
(D)Lipopolysaccharide
9. Which of the following is NOT assimilated by (E)Protein
eukaryotic cells?
(A)Glucose 3. The steps leading to the synthesis of peptidoglycan
(B)Lactate occur in the cytoplasm, on the cytoplasmic membrane,
(C)Sulfate (SO42-) and extracellularly. Which antibiotic inhibits an
(D)Nitrogen (N2) extracellular step in peptidoglycan biosynthesis?
(E)Phosphate (PO43-) (A)Cycloserine
(B)Rifampin
10. Bacteria that are obligate intracellular pathogens of (C)Penicillin
humans (eg, Chlamydia trachomatis) are considered to (D) Bacitracin
be (E) Streptomycin
(A)Autotrophs
(B)Photosynthetic 4. Amino acids are found in the protein, peptidoglycan,
(C)Chemolithotrophs and capsule of bacteria. Which of the following amino
(D)Hyperthermophiles acids is found only in peptidoglycan?
(E)Heterotrophs (A) l-Lysine
(B) Diaminopimelic acid
(C) d-Glutamate
(D) l-Alanine
(E) None of the above

5. The ability to use compounds and ions other than


oxygen as terminal oxidants in respiration is a
widespread microbial trait. Tis capacity is called
(A)Photosynthesis
(B)Fermentation
(C)Anaerobic respiration
(D)Substrate phosphorylation
(E)Nitrogen fixation
6. The primary route of carbon assimilation used by 11. Which of these pathways gives a cell the potential to
organisms that can use CO2 as a sole source of carbon is produce the most ATP?
(A)Hexose monophosphate shunt (A)TCA cycle
(B)Entner-Doudoroff pathway (B)Pentose phosphate pathway
(C)Embden-Meyerhof pathway (C)Glycolysis
(D)Glyoxalate cycle (D)Lactic acid fermentation
(E)Calvin cycle (E)Entner-Doudoroff pathway

7. The peptidoglycan biosynthetic pathway is of 12. During the process of oxidative phosphorylation, the
particular importance in medicine because it provides a energy of the proton motive force is used to generate
basis for selective antibacterial action of several (A)NADH
chemotherapeutic agents. All of the following antibiotics (B)ADP
inhibit steps in peptidoglycan biosynthe-sis EXCEPT (C)NADPH
(A)Cycloserine (D)Acetyl CoA
(B)Vancomycin (E)ATP
(C)Bacitracin
(D)Streptomycin
(E)Penicillin

8. The regulation of enzyme activity provides fine control


of metabolic pathways. Which of the following
regulatory mechanisms provides fine control of a
biosynthetic pathway?
(A)Catabolite repression
(B)Induction
(C)Feedback inhibition
(D)Attenuation
(E)None of the above

9. The biosynthetic origin of building blocks and


coenzymes can be traced back to relatively few
precursors called focal metabolites. Which of the
following are focal metabolites?
(A)α-Ketoglutarate
(B)Oxaloacetate
(C)Phosphoenolpyruvate
(D)Glucose 6-phosphate
(E)All of the above

10. Which of the following is NOT a component of


peptidoglycan?
(A)N-Acetyl muramic acid
(B)N-Acetyl glucosamine
(C)Lipid A
(D)Pentaglycine
(E)Diaminopimelic acid
Chapter VII: Microbial Genetics 7. If the arrangement of bases on the coding DNA strand
is 5′CATTAG3′, then a corresponding strand of mRNA will
Review Questions be which one of the following?
1. Mutations in bacteria can occur by which of the (A)5′GTAATC3′
following mechanisms? (B)5′CUAAUG3′
(A) Base substitutions (C)5′CTAATG3′
(B) Deletions (D)5′GUAAUC3′
(C) Insertions (E)5′CATTAG3′
(D) Rearrangements
(E) All of the above

2. The form of genetic exchange in which donor DNA is


introduced to the recipient by a bacterial virus is
(A)Transformation
(B)Conjugation
(C)Transfection
(D)Transduction
(E)Horizontal transfer

3. The enzyme DNAse degrades naked DNA. If two strains


of bacteria from the same species were mixed in the
presence of DNAse, which method of gene transfer
would be most likely inhibited?
(A)Conjugation
(B)Transduction
(C)Transformation
(D)Transposition
(E)All of the above

4. Replication of which of the following requires physical


integration with a bacterial replicon?
(A)Single-stranded DNA bacteriophage
(B)Double-stranded DNA bacteriophage
(C)Single-stranded RNA bacteriophage
(D)Plasmid
(E)Transposon

5. The formation of a mating pair during the process of


conjugation in Escherichia coli requires
(A)Lysis of the donor
(B)A sex pilus
(C)Transfer of both strands of DNA
(D)A restriction endonuclease
(E)Integration of a transposon

6. Why do bacteria contain restriction enzymes?


(A)To cleave RNA for incorporation into ribosome
(B)To extend the length of bacterial chromosomes
(C)To prevent foreign DNA from incorporating into a
bacterial genome
(D)To process the exons from prokaryotic mRNA
(E)To proteolytically cleave nuclear promoters
Chapter VIII: Immunology 7. Within the innate immune response, this cell acts by
killing virus-infected cells:
Review Questions (A) T cell
1. The immunoglobulin class most frequently responsible (B) NK cell
for inhibition of bacteria on mucosal surfaces is: (C) Macrophage
(A)IgG (D) Neutrophil
(B)IgM (E) B cell
(C)IgA
(D)IgE 8. The interaction of two IgG molecules binding to
(E)IgD antigen followed by the binding of C1 to the Fc portion
of the antibody results in which of the following?
2. As part of the innate immune response, which cells (A)Initiation of antigen presentation
participate in phagocytosis? (B)Initiation of classic complement pathway
(A) Macrophages and mast cells (C)Initiation of alternative complement pathway
(B) Macrophages and plasma cells (D)Initiation of the mannose-binding lectin-binding
(C)NK cells and neutrophils complement pathway
(D)Macrophages and neutrophils
(E)T cells and mast cells 9. What is a characteristic of the adaptive immune
response and not of the innate response?
3. Which of the cytokines attract neutrophils and inhibit (A)Physical barriers
bacteria? (B)Chemical barriers
(A)IFN-γ (C)Clonal expansion of effector cells
(B)IL-8 (D)Inflammatory mediators
(C)IL-2 (E)Phagocytosis
(D)IL-6
(E)TGF-β 10. Which genetic mechanism generates antibodies of
the same specificity but different immunoglobulin
4. MHC class II molecules are critically important in what classes?
immunologic process? (A)V gene segment recombination
(A)Antigen presentation (B)Class switching
(B)Phagocytosis (C)Somatic hypermutation
(C)Immunoglobulin class switching (D) Junctional variability due to imprecise V, D, and J
(D)CD8+ T cell cytotoxicity joining
(E)Opsonization (E) Gene duplication, ie, multiple V, D, and J gene
segments
5. MHC class I molecules are critically important in what
immunologic process? 11. Which major antibody molecule has the ability to
(A)IgE-mediated histamine release cross the placenta?
(B)Phagocytosis (A)IgG
(C)Immunoglobulin class switching (B)IgA
(D)CD8+ T-cell cytotoxicity (C)IgM
(E)Opsonization (D)IgE
(E)IgD
6. The host response to the interaction of a pathogen
with its specific TLR generates which of the following?
(A)IgG production
(B)Cell activation and production of cytokines and
chemokines
(C)Immunoglobulin class switching
(D)Phagocytosis
(E)Presentation of pathogen to helper T cells
12. A man in his twenties presents in the emergency 18. Which two cytokines play an important role in
room with shortness of breath and fatigue. He is also attracting neu-trophils to the site of an infection?
very pale. Two days earlier he was given penicillin for an (A) IFN-α and IFN-γ
infection. He had penicillin previously without problems (B) IL-8 and IL-17
and stated that he had “no allergy” to penicillin. (C) IL-2 and IL-4 (D) IL-6 and IL-12
Laboratory testing shows that antibodies to penicillin are 19. Which one of the following laboratory assays is
present in the patient’s serum and that he is breaking considered to be an in vitro counterpart of the type IV
down his own red blood cells. He is diagnosed with hypersensitivity reactions seen in the TB skin test?
immune hemolytic anemia. The patient has which type (A)Immunoblot for TB antigen
of hypersensitivity reaction? (B)EIA analysis of sera from TB patient
(A)Type I (C)Immunofluorescence assay for TB antibody
(B)Type II (D)IFN-γ production by white blood cell (WBC) treated
(C)Type III with TB antigen
(D)Type IV (DTH)
20. Which of the following laboratory assays can be used
13.Which one of the following cell types expresses to detect the number and types of immune cells in the
receptors for IgE on its cell surface that stimulate the cell peripheral blood?
to mount a response to parasites such as worms? (A)Immunofixation electrophoresis
(A)T cells (B)EIA
(B)B cells (C)Flow cytometry
(C)NK cells (D)Immunoblot
(D)Mast cells
(E)Dendritic cells

14. NK cells express a killer immunoglobulin-like receptor


(KIR), which recognizes:
(A)MHC class I molecules
(B)Cell adhesion molecules
(C)Glycophospholipid molecules
(D)CD40 molecules

15. Prior to class switching, all B cells bound to antigen


have which of the following antibody classes on their
surface?
(A)IgA
(B)IgG
(C)IgM
(D)IgE

16. IgE-mediated histamine release is classified as what


type of hypersensitivity reaction?
(A)Type I
(B)Type II
(C)Type III
(D)Type IV

17. IFN-α and IFN-β are produced by the virus-infected


cell due to the interaction of virus nucleic acid with which
of the following?
(A)C3 (third component of complement)
(B)Defensins
(C)TLR pathway
(D)IL-12
Chapter IX: Pathogenesis of Bacterial Infection (C)The S epidermidis in the biofilm are likely to be more
susceptible to antimicrobial therapy because the
Review Questions bacteria have decreased rates of metabolism.
1. A 22-year-old woman who works in a plant nursery (D)The quorum-sensing ability of S epidermidis results in
presents with a history of fever and cough for 2 months. increased susceptibility to antimicrobial therapy.
Over this period of time she has lost 5 kg. Chest (E)The complex molecular interactions within the biofilm
radiography shows bilateral upper lobe infiltrates with make it difficult to provide effective antimicrobial
cavities. A stain of her sputum shows acid-fast bacilli. The therapy, and it is likely the catheter will have to be
likely means by which the patient acquired her infection removed to cure the infection.
is
(A)Sexual activity 4. The first microorganism to satisfy Koch’s postulates (in
(B)Ingesting the microorganisms in her food the late 19th century) was
(C)Holding onto contaminated hand rails when she takes (A)Treponema pallidum
public transportation (B)Stenotrophomonas maltophilia
(D) Handling potting soil (C)Mycobacterium leprae
(E) Breathing aerosolized droplets containing the (D)Bacillus anthracis
microorganism (E)Neisseria gonorrhoeae

2. During a pandemic of a well-characterized disease, a 5. Which of the following statements about


group of 175 airline passengers flew from Lima, Peru, to lipopolysaccharide is correct?
Los Angeles. Lunch on the plane included crab salad, (A)It interacts with macrophages and monocytes yielding
which was eaten by about two-thirds of the passengers. release of cytokines.
(B)The toxic component is the O side chain.
Aſter landing in Los Angeles, many of the passengers
(C)It forms holes in red blood cell membranes yielding
transferred to other flights with destinations in other hemolysis.
parts of California and other Western states. Two of the (D)It causes hypothermia.
passengers who stayed in Los Angeles developed severe (E)It causes paralysis.
watery diarrhea. The status of the other passengers was
unknown. The likely cause of the diarrhea in the two 6. A 27-year-old man had a rhinoplasty. A nasal tampon
passengers is was placed to control the bleeding. Approximately 8
(A)Escherichia coli O157:H7 (lipopolysaccharide O hours later, he developed headache, muscle aches, and
antigen 157; flagellar antigen 7) abdominal cramps with diarrhea. He then developed an
(B)Vibrio cholerae type O139 (lipopolysaccharide O erythematous rash (resembling sunburn) over much of
antigen 139) his body, including the palms and soles. His blood
(C)Shigella dysenteriae type 1 pressure is 80/50 mm Hg. The nasal tampon remained in
(D)Campylobacter jejuni place. His liver enzyme tests were elevated, and there
(E)Entamoeba histolytica was evidence of moderate renal failure. Tis patient’s
illness was likely to be caused by which of the following?
3. A 65-year-old woman has a long-term central venous (A)Lipopolysaccharide
catheter for intravenous therapy. She develops fever and (B)Peptidoglycan
subsequently has multiple blood cultures positive for (C)A toxin that is a superantigen
Staphylococcus epider-midis. All of the S epidermidis (D)A toxin that has A and B subunits
isolates have the same colony morphology and (E)Lecithinase (alpha toxin)
antimicrobial susceptibility pattern, suggesting that they
are the same strain. An S epidermidis biofilm is thought 7. The organism most likely to be responsible for the
to be present on the catheter. Which one of the following patient’s disease (Question 6) is
statements about such an infection is correct? (A)E coli
(A)The biofilm containing the S epidermidis is likely to (B)Corynebacterium diphtheriae
wash off the catheter. (C)Clostridium perfringens
(B)Production of an extracellular polysaccharide inhibits (D)Neisseria meningitidis
growth of the S epidermidis, limiting the infection. (E)Staphylococcus aureus
8. Which of the following is most likely to be associated 13. A 23-year-old woman has a history of recurrent
with the formation of a bacterial biofilm? urinary tract infections, including at least one episode of
(A)Airway colonization in a cystic fibrosis patient with a pyelonephritis. Blood typing shows the P blood group
mucoid (alginate-producing) strain of P aeruginosa antigen. Which of the following is likely to be the primary
(B)Urinary tract infection with E coli cause of her infections?
(C)Meningitis with N meningitidis (A)E coli that produce heat-stable toxin
(D)Tetanus (B)E coli with K1 (capsular type 1) antigen
(E)Impetigo caused by S aureus (C)E coli O139 (lipopolysaccharide O antigen 139)
(D)E coli with P-pili (fimbriae)
9. Regarding bacterial type III secretions systems, which (E)E coli O157:H7 (lipopolysaccharide O antigen 157;
of the following statements is correct? flagellar antigen 7)
(A)They are commonly found in gram-positive
commensal bacteria. 14. A 55-year-old man presents with gradually increasing
(B)They play an important role in the pathogenesis of weight loss, abdominal pain, diarrhea, and arthropathy.
toxin-induced diseases of Clostridium species, tetanus, During the evaluation process, a small bowel biopsy is
botulism, gas gangrene, and pseudomembranous colitis. done. Aſter processing, examination of the specimen by
(C)They cause release of effectors of pathogenesis into
light microscopy reveals periodic acid-Schiff–positive
the extracellular environment, promoting bacterial
inclusions in the bowel wall. Which of the following tests
colonization and multiplication.
could be done to confirm the diagnosis of Whipple
(D)They directly inject bacterial proteins into host cells
disease caused by Tropheryma whipplei?
across bacterial and host cell membranes, promoting
(A)Culture on agar media
pathogenesis of infections.
(B)Polymerase chain reaction amplification and
(E)Mutations that prevent the bacterial type III secretion
sequencing of an appropriate segment of DNA
from functioning enhance pathogenesis.
(C)Cocultivation with E coli
(D)In situ hybridization
10. Which of the following statements is correct?
(E)Direct fluorescent antibody test
(A)Lipopolysaccharide is part of the cell wall of E coli.
(B)Cholera toxin is attached to the flagella of V cholerae.
15. Which of the following best describes the mechanism
(C)The lecithinase of C perfringens causes diarrhea.
of action of diphtheria toxin?
(D)Toxic shock syndrome toxin-1 is produced by
(A)Forms pores in red blood cells causing hemolysis
hemolytic stains of S epidermidis.
(B)Degrades lecithin in eukaryotic cell membranes
(C)Causes release of tumor necrosis factor
11. A 15-year-old Bangladeshi girl develops severe
(D)Inhibits elongation factor 2
watery diarrhea. The stool looks like “rice water.” It is
(E)Causes increased adenylate cyclase activity
voluminous—more than 1 L in the last 90 minutes. She
has no fever and seems otherwise normal except for the
effects of loss of fluid and electrolytes. The most likely
cause of her illness is
(A)Clostridium difficile enterotoxin
(B)A toxin with A and B subunits
(C)Shigella dysenteriae type 1 that produces Shiga toxin
(D)Enterotoxigenic E coli that produces heat-labile and
heat-stable toxins
(E)Staphylococcal enterotoxin F

12.The most important thing that can be done to treat


the patient (Question 11) is
(A)To give her ciprofloxacin
(B)To give her a toxoid vaccine
(C)To give her the appropriate antitoxin
(D)To treat her with fluid and electrolyte replacement
(E)To culture her stool to make the correct diagnosis and
then treat specifically
Chapter X: Normal Human Microbiota 5. Antimicrobial therapy can decrease the amount of
susceptible bowel flora and allow proliferation of
Review Questions relatively resistant colonic bacteria. Which one of the
1. A 26-year-old woman visits her physician because of following species can proliferate and produce a toxin
an unusual vaginal discharge. On examination, the that causes diarrhea?
physician observes a thin, homogeneous, white-gray (A)Enterococcus species
discharge that adheres to the vaginal wall. The pH of the (B)S epidermidis
discharge is 5.5 (normal) (C)Pseudomonas aeruginosa
(A)Corynebacterium species (D)Clostridium difficile
(B)Staphylococcus epidermidis (E)B fragilis
(C)Prevotella species
(D)Candida albicans 6. Which one of the following microorganisms can be
(E)Lactobacillus species part of the normal vaginal flora and cause meningitis in
newborns?
2. Certain microorganisms are never considered to be (A)C albicans
members of the normal flora. They are always (B)Corynebacterium species
considered to be pathogens. Which one of the following (C)S epidermidis
organisms fits into that category? (D)Ureaplasma urealyticum
(A)Streptococcus pneumoniae (E)Group B streptococci
(B)Escherichia coli
(C)Mycobacterium tuberculosis 7. Dental plaque and periodontal disease can be thought
(D)Staphylococcus aureus of as a continuum of what type of physiological process?
(E)Neisseria meningitidis (A)Biofilm formation
(B)Normal aging
3. A 9-year-old girl develops fever and severe pain on the (C)Abnormal digestion
right side of her throat. On examination, redness and (D)Exaggerated immune response
swelling in the right peritonsillar area are seen. A (E)Chewing gum
peritonsillar abscess is diagnosed. The most likely
organisms to be cultured from this abscess are 8. Which one of the following microorganisms is closely
(A) S aureus associated with dental caries?
(B) S pneumoniae (A)C albicans
(C)Corynebacterium species and Prevotella (B)Streptococcus mutans
melaninogenica (C)P melaninogenica
(D)Normal oral nasal flora (D)Neisseria subflava
(E)Viridans streptococci and C albicans (E)S epidermidis

4. A 70-year-old man with a history of diverticulosis of 9. Anaerobic bacteria such as B fragilis occur in the
the sigmoid colon experiences a sudden onset of severe sigmoid colon in a concentration of about 1011/g of
stool. At what concentration do facultative organisms
leſt lower quadrant abdominal pain. Fever develops. The
such as E coli occur?
severe pain gradually subsides and is replaced by a (A)1011/g
constant aching pain and marked abdominal tenderness. (B)1010/g
A diagnosis of probable ruptured diverticulum is made, (C)109/g
and the patient is taken to the operating room. The (D)108/g
diagnosis of ruptured diverticulum is confirmed, and an (E)107/g
abscess next to the sigmoid colon is found. The most
likely bacteria to be found in the abscess are 10. S pneumoniae can be part of the normal flora of 5–
(A)Mixed normal gastrointestinal flora 40% of people. At what anatomic site can it be found?
(B)Bacteroides fragilis alone (A) Conjunctiva
(C)E coli alone (B) Nasopharynx
(D)Clostridium perfringens alone (C)Colon
(E)Enterococcus species alone (D)Urethra
(E)Vagina
11. Hundreds of phylotypes have been identified in the Chapter XI: Spore-Forming Gram-Positive Bacilli:
human stomach; however, the only microorganism that Bacillus and Clostridium Species
has been shown to persist is
(A)Lactobacillus casei Review Questions
(B)Lactobacillus acidophilus 1. A housewife who lives on a small farm is brought to
(C)E coli the emergency department complaining of double vision
(D)Helicobacter pylori and difficulty talking. Within the past 2 hours, she noted
(E)Bifidobacteria a dry mouth and generalized weakness. Last night she
served home-canned green beans as part of the meal.
12. Resident flora is commonly found in the She tasted the beans before they were boiled. None of
(A) Liver the other family members are ill. On examination, there
(B)Urethra is symmetrical descending paralysis of the cranial nerves,
(C)Kidneys upper extremities, and trunk. The correct diagnosis is
(D)Salivary glands which one of the following?
(E)Gall bladder (A)Tetanus
(B)Strychnine poisoning
13. Resident flora is absent from the (C)Botulism
(A) Pharynx (D)Morphine overdose
(B)Lungs (E)Ricin intoxication
(C)Small intestine
(D)Synovial fluid 2. Which one of the following is an important virulence
(E)Conjunctiva factor of Bacillus anthracis?
(A)Protective antigen
14. A 65-year-old woman was admitted with squamous (B)Lipopolysaccharide
cell carcinoma of the cervix. She underwent extensive (C)Pili
gynecologic surgery and was maintained postoperatively (D)A toxin that inhibits peptide chain elongation factor
on broad-spectrum intravenous antibiotics. The patient EF-2
had a central venous catheter placed on the day of (E)Lecithinase
surgery. Beginning 3 days postoperatively, the patient
became febrile. On day 8, cultures of blood and of the tip 3. A young man sustains major soſt tissue injury and open
of the central line both grew gram-positive organisms
fractures of his right leg aſter a motorcycle accident. One
that were ovoid and reproduced by budding. Which of
the following microorganisms is most likely responsible day later, he has a temperature of 38°C, increased heart
for the patient’s condition? rate, sweating, and restlessness. On examination, the leg
(A)S aureus is swollen and tense, with thin, dark serous fluid draining
(B)S epidermidis from the wounds. The skin of the leg is cool, pale, white,
(C)Enterococcus faecalis and shining. Crepitus can be felt in the leg. His hematocrit
(D)C albicans is 20% (~50% of normal), and his circulating hemoglobin
(E)Saccharomyces cerevisae is normal. His serum shows free hemoglobin. Which of
the following microorganisms is the most likely cause of
15. The most likely portal of entry for the organism in this infection?
Question 14 is (A)Clostridium tetani
(A)During gynecologic surgery (B)Staphylococcus aureus
(B)Aspiration (C)Escherichia coli
(C)During placement of the central venous catheter (D)Bacillus anthracis
(D)During placement of IV line for administration of (E)Clostridium perfringens
antibiotics
(E)Intubation while under anesthesia 4. For the patient described in Question 3, which of the
following is likely to be responsible for the hemolysis?
(A)Elongation factor
(B)Tetanospasmin
(C)Lecithinase
(D)Streptolysin O
(E)Toxin B
5. The reported incubation period for inhalational 10. A 67-year-old man had surgery for a ruptured sigmoid
anthrax can be up to colon diverticulum with an abscess. A repair was done,
(A)2 days and the abscess was drained. He was treated with
(B)10 days intravenous gentamicin and ampicillin. Ten days later
(C)3 weeks and 4 days aſter being discharged from the hospital, the
(D)6 weeks
patient developed malaise, fever, and cramping
(E)6 months
abdominal pain. He had multiple episodes of diarrhea.
His stool was positive for occult blood and the presence
6. A food commonly associated with Bacillus cereus food
of polymorphonuclear cells. On sigmoidoscopy, the
poisoning is
mucosa was erythematous and appeared to be inflamed,
(A)Fried rice
and there were many raised white to yellowish plaques
(B)Baked potato
4–8 mm in diameter. Which of the following is the likely
(C)Hot freshly steamed rice
cause of the patient’s problem?
(D)Green beans
(A)Staphylococcus aureus enterotoxin
(E)Honey
(B)Bacillus cereus toxin
(C)Clostridium difficile toxins
7. Tetanus toxin (tetanospasmin) diffuses to terminals of
(D)Clostridium perfringens toxin
inhibitory cells in the spinal cord and brainstem and
(E)Enterohemorrhagic Escherichia coli
blocks which of the following?
(A)Release of acetylcholine
11. Infant botulinum has been associated with all of the
(B)Cleavage of SNARE proteins
following Clostridium species EXCEPT:
(C)Release of inhibitory glycine and γ-aminobutyric acid
(A)Clostridium baratii
(D)Release of Protective Antigen
(B)Clostridium septicum
(E)Activation of acetylcholine esterase
(C)Clostridium butyricum
(D)Clostridium botulinum
8. A 45-year-old man who immigrated to the United
States 5 years ago sustained a puncture injury to the
12. Which of the following food items is most frequently
lower part of his right leg when his rotary lawn mower
associated with infant botulism?
threw a small stick into his leg. Six days later, he noticed
(A)Corn syrup
spasms in the muscles of his right leg; on day 7, the
(B)Canned infant formula
spasms increased. Today—day 8—he had generalized
(C)Liquid multivitamins
muscle spasms, particularly noticeable in the muscles of
(D)Honey
his jaw. He was unable to open his jaw and came to the
(E)Jarred baby food
emergency department (ED). In the ED, you see a man
who is alert and lying quietly in bed. A door slams down
13. All of the following are properties characteristic of
the hall, and suddenly he has general muscle spasm with
Bacillus anthracis EXCEPT:
arching of his back. The correct diagnosis is which of the
(A)Motility on wet mount examination
following?
(B)Medusa head colonies
(A)Botulism
(C)Poly-d-glutamic acid capsule
(B)Anthrax
(D)In vitro susceptibility to penicillin
(C)Gas gangrene
(E)Absence of hemolysis on 5% sheep blood agar
(D)Tetanus
(E)Toxic shock syndrome
14. Which of the following statements regarding
vaccination for Bacillus anthracis is correct?
9. Which of the following statements about tetanus and
(A)It is routinely available for all citizens of the United
tetanus toxoid is correct?
States.
(A)Tetanus toxin kills neurons.
(B)Recombinant vaccine trials have shown good safety
(B)Tetanus toxoid immunization has a 10% failure rate.
and efficacy.
(C)The mortality rate of generalized tetanus is less than
(C)The current vaccine is well tolerated.
1%.
(D)Double vision is commonly the first sign of tetanus. (D)A single dose is adequate aſter exposure to spores.
(E)Tetanus toxin acts on inhibitor interneuron synapses. (E)Vaccination of animals is not useful.
15. All of the following statements regarding Clostridium Chapter XII: Aerobic Non-Spore-Forming Gram-Positive
perfringens are correct EXCEPT: Bacilli: Corynebacterium, Listeria, Erysipelothrix,
(A)It produces an enterotoxin. Nocardia and Related Pathogens
(B)It produces a double zone of β-hemolysis when grown
on blood agar. Review Questions
(C)Some strains are aerotolerant. 1. Three months ago, a 53-year-old woman had surgery
(D)It is the most common cause of antibiotic-associated and chemotherapy for breast cancer. Four weeks ago,
diarrhea. she developed a cough occasionally productive of
(E)It can cause intravascular hemolysis. purulent sputum. About 2 weeks ago, she noted a slight
but progressive weakness of her leſt arm and leg. On
chest examination, rales were heard over the leſt upper
back when the patient breathed deeply. Neurologic
examination confirmed weakness of the leſt arm and leg.
Chest radiography showed a leſt upper lobe infiltrate.
Contrast-enhanced computed tomography showed two
lesions in the right hemisphere. Gram stain of a purulent
sputum specimen showed branching gram-positive rods
that were partially acid fast. Which of the following
organisms is the cause of this patient’s current illness?
(A)Actinomyces israelii
(B)Corynebacterium pseudodiphtheriticum
(C)Aspergillus fumigatus
(D)Nocardia farcinica
(E)Erysipelothrix rhusiopathiae

2. The drug of choice to treat this patient’s infection


(Question 1) is
(A) Penicillin G
(B)Trimethoprim–sulfamethoxazole
(C)Gentamicin
(D)Amphotericin B
(E)A third-generation cephalosporin

3. It is particularly difficult to differentiate Erysipelothrix


rhusiopathiae from
(A) Corynebacterium diphtheriae
(B)Bacillus cereus
(C)Actinomyces israelii
(D)Nocardia asteroides
(E)Lactobacillus species

4. Movement of Listeria monocytogenes inside of host


cells is caused by
(A)Inducing host cell actin polymerization
(B)The formation of pili (fimbriae) on the listeriae surface
(C)Pseudopod formation
(D)The motion of listeriae flagella
(E)Tumbling motility
5. An 8-year-old boy, who recently arrived in the United 10. A 45-year-old fisherman imbedded a fishhook into his
States, develops a severe sore throat. On examination, a right forefinger. He removed it and did not seek
grayish exudate (pseudomembrane) is seen over the immediate medical therapy. Five days later, he noted
tonsils and pharynx. The differential diagnosis of severe fever, severe pain, and nodular-type swelling of the
pharyngitis such as this includes group A streptococcal finger. He sought medical therapy. The violaceous
infection, Epstein-Barr virus (EBV) infection, Neisseria nodule was aspirated, and aſter 48 hours of incubation,
gonorrhoeae pharyngitis, and diphtheria. The cause of
colonies of a gram-positive bacillus that caused greenish
the boy’s pharyngitis is most likely:
discoloration of the agar and formed long filaments in
(A)A gram-negative bacillus
the broth culture were noted. The most likely cause of
(B)A single-stranded positive-sense RNA virus
this infection is
(C)A catalase-positive, gram-positive coccus that grows
(A)Lactobacillus acidophilus
in clusters
(B)Erysipelothrix rhusiopathiae
(D)A club-shaped gram-positive bacillus
(C)Listeria monocytogenes
(E)A double-stranded RNA virus
(D)Rhodococcus equi
(E)Nocardia brasiliensis
6. The primary mechanism in the pathogenesis of the
boy’s disease (Question 5) is
11. A biochemical reaction that is useful in the
(A)A net increase in intracellular cyclic adenosine
identification of the causative agent of the infection in
monophosphate
Question 10 is
(B)Action of pyrogenic exotoxin (a superantigen)
(A)Catalase positivity
(C)Inactivation of acetylcholine esterase
(B)Acid fastness using modified Kinyoun stain
(D)Action of enterotoxin A
(C)Esculin hydrolysis
(E)Inactivation of elongation factor 2
(D)Tumbling motility
(E)Production of H2S
7. Corynebacterium jeikeium is
(A)Catalase negative
12. Listeria monocytogenes is frequently a foodborne
(B)Gram negative
pathogen because
(C)Oſten multidrug resistant (A)It can survive at 4°C.
(D)Motile (B)It survives under conditions of low pH.
(E)Common but clinically unimportant (C)It survives in the presence of high salt concentrations.
(D)All of the above are correct.
8. Which of the following aerobic gram-positive bacilli is
modified acid-fast positive? 13. Aſter recovery on laboratory media, the aerobic
(A)Nocardia brasiliensis
Actinomycetes are best identified by
(B)Lactobacillus acidophilus
(A)An automated system used in the laboratory
(C)Erysipelothrix rhusiopathiae
(B)Classical biochemicals
(D)Listeria monocytogenes
(C)Antigen detection tests such as an ELISA
(D)Molecular methods such as 16SrRNA gene sequencing
9. Skin diphtheria as occurs in children in tropical areas
typically
14. Which of the following statements regarding
(A) Does not occur in children who have been immunized
Rhodococcus equi is correct?
with diphtheria toxoid
(A)It is transmitted person to person.
(B)Is clinically distinct from skin infections (pyoderma,
(B)It causes tuberculosis in cattle.
impetigo) caused by Streptococcus pyogenes and
(C)It is a rare cause of pulmonary infection in humans.
Staphylococcus aureus
(D)It produces a black pigment on sheep blood agar.
(C)Is also common in northern latitudes
(D)Results in protective antitoxin levels in most children
by the time they are 6–8 years old
(E)Yields toxin-mediated cardiomyopathy
15.A hospitalized patient who had an indwelling urinary Chapter XIII: The Staphylococci
catheter develops fever, chills, suprapubic pain, and
difficulty voiding 48 hours aſter the catheter is removed. Review Questions
His bladder appears obstructed, and he has white blood 1. A 54-year-old woman develops a right shoulder
cells and bacteria on a urinalysis. Cystoscopy reveals a abscess with a strain of Staphylococcus aureus that is
large bladder stone, and the urine culture grows greater resistant to nafcillin. She was treated with a 2-week
than 10,000 CFU/mL of a short, irregular gram-positive course of intravenous vancomycin and improved. Tree
rod. The most likely organism causing this infection is weeks later (week 5), the infection recurred, and she was
(A)Corynebacterium urealyticum given 2 more weeks of intravenous vancomycin and
(B)Nocardia brasiliensis again improved. Four weeks later (week 11), the
(C)Actinomadura infection recurred and the patient was again started on
(D)Erysipelothrix rhusiopathie intravenous vancomycin. The MICs for vancomycin for
(E)Lactobacillus acidophilus the S aureus isolates were as follows: initial isolate (day
1), 1 μg/mL; week 5, 2 μg/mL; and week 11, 8 μg/mL. The
patient failed to improve with the third course of
vancomycin, and alternative therapy was used. The
mechanism that best explains the relative resistance of
the patient’s strain of S aureus to vancomycin is
(A)Acquisition of the vanA gene from another
microorganism
(B)Active transport of vancomycin out of the
Staphylococcus aureus cell
(C)Action of β-lactamase
(D)Increased cell wall synthesis and alterations in the cell
wall structure
(E)Phosphorylation and resultant inactivation of the
vancomycin

2. An 11-year-old boy develops a mild fever and pain in


his upper arm. A radiograph of his arm shows a lytic
lesion (dissolution) in the upper part of the humerus with
periosteal elevation over the lesion. The patient is taken
to surgery, where the lesion is debrided (dead bone and
pus removed). Culture from the lesion yields gram-
positive cocci. A test shows that the organism is a
Staphylococcus and not a Streptococcus. Based on this
information, you know the organism is
(A)Susceptible to nafcillin
(B)β-Lactamase positive
(C)A producer of protein A
(D)Encapsulated
(E)Catalase positive

3. A 36-year-old male patient has an abscess with a strain


of Staphylococcus aureus that is β-lactamase positive. Tis
indicates that the organism is resistant to which of the
following antibiotics?
(A)Penicillin G, ampicillin, and piperacillin
(B)Trimethoprim–sulfamethoxazole
(C)Erythromycin, clarithromycin, and azithromycin
(D)Vancomycin
(E)Cefazolin and ceſtriaxone
4. Seven days ago, a 27-year-old medical student 7. A 16-year-old bone marrow transplant patient has a
returned from Central America, where she had spent the central venous line that has been in place for 2 weeks. He
summer working in a clinic for indigenous people. Four also has a urinary tract catheter, which has been in place
days ago, she developed an erythematous sunburn-like for 2 weeks as well. He develops fever while his white
rash. She also has had headache, muscle aches, and blood cell count is very low and before the transplant has
abdominal cramps with diarrhea. Her blood pressure is engraſted. Tree blood cultures are done, and all grow
70/40 mm Hg. Pelvic examination shows she is having
Staphylococcus epidermidis. Which one of the following
her menstrual period with a tampon in place; otherwise,
statements is correct?
the pelvic examination is normal. Her kidney function
(A)The Staphylococcus epidermidis organisms are likely
test (serum urea nitrogen and creatinine) results are
to be susceptible to penicillin G.
abnormal, indicating mild renal failure. A blood smear for
(B)The Staphylococcus epidermidis organisms are likely
malaria is negative. Her illness is likely to be caused by
to be from the surface of the urinary tract catheter.
which of the following?
(C)The Staphylococcus epidermidis organisms are likely
(A)A toxin that results in greatly increased levels of
to be resistant to vancomycin.
intracellular cyclic adenosine monophosphate (cAMP)
(D)The Staphylococcus epidermidis organisms are likely
(B)A toxin that degrades sphingomyelin
to be from a skin source.
(C)A toxin that binds to the class II major
(E)The Staphylococcus epidermidis organisms are likely
histocompatibility complex (MHC) of an antigen-
to be in a biofilm on the central venous catheter surface.
presenting cell and the Vβ region of a T cell
(D)A two-component toxin that forms pores in white
8. A 65-year-old man develops an abscess on the back of
blood cells and increases cation permeability
his neck. Culture yields Staphylococcus aureus. The
(E)A toxin that blocks elongation factor 2 (EF2)
isolate is tested and found to be positive for the mecA
gene, which means that
5. Over a period of 3 weeks, a total of five newborns in
(A)The isolate is susceptible to vancomycin.
the hospital nursery developed Staphylococcus aureus
(B)The isolate is resistant to vancomycin.
infections with S aureus bacteremia. Te isolates all had
(C)The isolate is susceptible to nafcillin.
the same colony morphology and hemolytic properties
(D)The isolate is resistant to nafcillin
and identical antimicrobial susceptibility patterns,
(E)The isolate is susceptible to clindamycin.
suggesting that they were the same. (Later molecular
(F)The isolate is resistant to clindamycin.
methods showed the isolates were identical.) Which of
the following should be done now?
9. Antimicrobial resistance has become a significant
(A)Prophylactic treatment of all newborns with
problem. Which one of the following is of major concern
intravenous vancomycin
worldwide?
(B)Protective isolation of all newborns
(A)Nafcillin resistance in Staphylococcus aureus
(C)Closing the nursery and referring pregnant women to
(B)Penicillin resistance in Streptococcus pneumoniae
another hospital
(C)Penicillin resistance in Neisseria gonorrhoeae
(D)Hiring all new staff for the hospital nursery
(D)Vancomycin resistance in Staphylococcus aureus
(E)Culture using mannitol salt agar of the anterior nares
(E)Tobramycin resistance in Escherichia coli
of the physicians, nurses, and others who cared for the
infected babies
10. A group of six children younger than 8 years of age
live in a semitropical country. Each of the children has
6. The exfoliative toxins, TSST-1, and the enterotoxins are
several crusted weeping skin lesions of impetigo
all superantigens. Te genes for these toxins are
(pyoderma). The lesions are predominantly on the arms
(A) Present in all strains of Staphylococcus aureus
and faces. Which of the following microorganisms is a
(B) Widely distributed on the staphylococcal
likely cause of the lesions?
chromosome
(A)Escherichia coli
(C) On both the staphylococcal chromosome (TSST-1 and
(B)Chlamydia trachomatis
exfoliative toxins) and on plasmids (enterotoxins)
(C)Staphylococcus aureus
(D) On the staphylococcal chromosome in a
(D)Streptococcus pneumoniae
pathogenicity island
(E)Bacillus anthracis
(E) On plasmids
11. Which of the following statements regarding the role Chapter XIV: The Streptococci, Enterococci and Related
of protein A in the pathogenesis of infections caused by Genera
Staphylococcus aureus is correct?
(A)It is responsible for the rash in toxic shock syndrome. Review Questions
(B)It converts hydrogen peroxide into water and oxygen. 1. A 48-year-old alcoholic man is admitted to a hospital
(C)It is a potent enterotoxin. because of stupor. He is unkempt and homeless and lives
(D)It is directly responsible for lysis of neutrophils. in an encampment with other homeless people, who
(E)It is a bacterial surface protein that binds to the Fc called the authorities when he could not be easily
portion of IgG1. aroused. His temperature is 38.5°C, and his blood
pressure 125/80 mm Hg. He moans when attempts are
12. Which of the following staphylococcal organisms made to arouse him. He has positive Kernig and
produces coagulase and has been implicated in Brudzinski signs, suggesting meningeal irritation.
infections following a dog bite? Physical examination and chest radiography show
(A)Staphylococcus intermedius
evidence of leſt lower lobe lung consolidation. An
(B)Staphylococcus epidermidis
(C)Staphylococcus saprophyticus endotracheal aspirate yields rust-colored sputum.
(D)Staphylococcus hominis Examination of a Gram-stained sputum smear shows
(E)Staphylococcus hemolyticus numerous polymorphonuclear cells and numerous gram-
positive lancet-shaped diplococci. On lumbar puncture,
13. All of the following statements regarding Panton– the cerebrospinal fluid is cloudy and has a white blood
Valentine leukocidin are correct except cell count of 570/μL with 95% polymorphonuclear cells;
(A)It is a two-component toxin. Gram stain shows numerous gram-positive diplococci.
(B)It is commonly produced by community-associated Based on this information, the likely diagnosis is
MRSA strains. (A) Pneumonia and meningitis caused by Staphylococcus
(C)It is an important virulence factor. aureus
(D)It is identical to one of the staphylococcal (B) Pneumonia and meningitis caused by Streptococcus
enterotoxins. pyogenes
(E)It forms pores in the membranes of white blood cells. (C) Pneumonia and meningitis caused by Streptococcus
pneumoniae
14. Which of the following statements best describes the (D) Pneumonia and meningitis caused by Enterococcus
function of the accessory gene regulator in faecalis
Staphylococcus aureus? (E) Pneumonia and meningitis caused by Neisseria
(A)It regulates production of β-hemolysins. meningitidis
(B)It is influenced by environmental oxygen.
(C)It controls the preferential expression of surface 2. The patient in question 1 is started on antibiotic
adhesins. therapy to cover many possible microorganisms.
(D)It is important in the control of autolysis. Subsequently, culture of sputum and cerebrospinal fluid
yields gram-positive diplococci with a minimum
15. All of the following are important infection control inhibitory concentration to penicillin G of greater than 2
strategies in containing spread of MRSA in hospitals μg/mL. The drug of choice for this patient until further
except susceptibility testing can be done is
(A)Aggressive hand hygiene (A)Penicillin G
(B)Routine surveillance for nasal colonization among (B)Nafcillin
high-risk individuals (C)Trimethoprim–sulfamethoxazole
(C)Contact isolation for patients who are colonized or (D)Gentamicin
infected with MRSA (E)Vancomycin
(D)Routine antimicrobial prophylaxis for all patients
hospitalized for more than 48 hours 3. Tis infection (question 1) might have been prevented
(E)Aseptic management of skin lesions by
(A) Prophylactic intramuscular benzathine penicillin
every 3 weeks
(B) A 23-valent capsular polysaccharide vaccine
(C) A vaccine against serogroups A, C, Y, and W135 8. A 40-year-old woman develops severe headache and
capsular polysaccharide fever. Her neurologic examination findings are normal. A
(D) A vaccine of polyribosylribitol capsular brain scan shows a ring-enhancing lesion of the leſt
polysaccharide covalently linked to a protein
hemisphere. During surgery, a brain abscess is found.
(E) Oral penicillin twice daily
Culture of the abscess fluid grows an anaerobic gram-
negative bacillus (Fusobacterium nucleatum) and a
4. The pathogenesis of the organism causing the
catalase-negative gram-positive coccus that on Gram
infection (question 1) includes which of the following?
stain is in pairs and chains. The organism is β-hemolytic
(A)Invasion of cells lining the alveoli and entry into the
and forms very small colonies
pulmonary venule circulation
(A)Streptococcus pyogenes (group A)
(B)Resistance to phagocytosis mediated by M proteins
(B)Enterococcus faecalis (group D)
(C)Migration to mediastinal lymph nodes where
(C)Streptococcus agalactiae (group B)
hemorrhage occurs
(D)Streptococcus anginosus group
(D)Lysis of the phagocytic vacuole and release into the
(E)Staphylococcus aureus
circulation
(E)Inhibition of phagocytosis by a polysaccharide capsule
9. Important methods for classifying and speciating
streptococci are
5. A 13-valent capsular polysaccharide protein conjugate
(A)Agglutination using antisera against the cell wall
vaccine for the pathogen in question 1 is recommended
group-specific substance
(A)For children up to age 18 years and for selected adults
(B)Biochemical testing
(B)Only on exposure to a patient with disease caused by
(C)Hemolytic properties (α-, β-, nonhemolytic)
the organism
(D)Capsular swelling (quellung) reaction
(C)For all children ages 2–23 months plus selected older
(E)All of the above
children and adults with immunocompromising
conditions
10. An 8-year-old girl develops Sydenham’s chorea (“St.
(D)For children ages 24–72 months
Vitus dance”) with rapid uncoordinated facial tics and
(E)For all age groups older than age 2 months
involuntary purposeless movements of her extremities,
strongly suggestive of acute rheumatic fever. She has no
6. An 8-year-old boy develops a severe sore throat. On
other major manifestations of rheumatic fever (carditis,
examination, a grayish-white exudate is seen on the
arthritis, subcutaneous nodules, skin rash). The patient’s
tonsils and pharynx. The differential diagnosis includes
throat culture is negative for Streptococcus pyogenes
group A streptococcal infection, Epstein-Barr virus
(group A streptococci). However, she, her brother, and
infection, severe adenovirus infection, and diphtheria.
her mother all had sore throats 2 months ago. A test that
(Neisseria gonorrhoeae pharyngitis would also be
if positive would indicate recent S pyogenes infections is
included, but the patient has not been sexually abused.)
(A)Antistreptolysin S antibody titer
The cause of the boy’s pharyngitis is most likely
(B)Polymerase chain reaction for antibodies against M
(A)A catalase-negative gram-positive coccus that grows
protein
in chains
(C)ASO antibody titer
(B)A single-stranded positive-sense RNA virus
(D)Esculin hydrolysis
(C)A catalase-positive gram-positive coccus that grows in
(E)Antihyaluronic acid antibody titer
clusters
(D)A catalase-negative gram-positive bacillus
11. All of the following statements regarding the
(E)A double-stranded RNA virus
hyaluronic acid capsule of S pyogenes are correct except
(A)It is responsible for the mucoid appearance of the
7. A primary mechanism responsible for the
colonies in vitro.
pathogenesis of the boy’s disease (question 6) is
(B)It is antiphagocytic.
(A)A net increase in intracellular cyclic adenosine
(C)It binds to CD44 on human epithelial cells.
monophosphate
(D)It is an important virulence factor.
(B)Action of M protein
(E)A vaccine against the capsule is currently available.
(C)Action of IgA1 protease
(D)Action of enterotoxin A
(E)Inactivation of elongation factor 2
12. Enterococci can be distinguished from Chapter XV: Enteric Gram-Negative Rods
nonenterococcal group D streptococci on the basis of (Enterobacteriaceae)
which of the following characteristics?
(A)γ-Hemolysis Review Questions
(B)Esculin hydrolysis 1. A 20-year-old college student goes to the student
(C)Growth in 6.5% NaCl health center because of dysuria, frequency, and urgency
(D)Growth in the presence of bile on urination for 24 hours. She has recently become
(E)Gram stain morphology sexually active. On urinalysis, many polymorphonuclear
cells are seen. The most likely organism responsible for
13. Which of the following statements regarding the these symptoms and signs is
Streptococcus bovis group is correct? (A)Staphylococcus aureus
(A)They possess Lancefield group D antigen. (B)Streptococcus agalactiae
(B)Some strains are vancomycin resistant. (C)Gardnerella vaginalis
(C)Infections caused by these organisms are benign. (D)Lactobacillus species
(D)All subspecies are PYR positive. (E)Escherichia coli
(E)All subspecies are β-hemolytic.
2. A 27-year-old woman is admitted to the hospital
14. Which of the following genera requires pyridoxal for because of fever, with increasing anorexia, headache,
growth? weakness, and altered mental status of 2 days’ duration.
(A)Aerococcus She works for an airline as a cabin attendant, flying
(B)Granulicatella between the Indian subcontinent and other places in
(C)Enterococcus Southeast Asia and the West Coast of the United States.
(D)Leuconostoc Ten days before admission, she had a diarrheal illness
(E)Pediococcus that lasted for about 36 hours. She has been constipated
for the past 3 days. Her temperature is 39°C, heart rate
15. Which of the following genera is typically resistant to is 68 beats/ min, blood pressure is 120/80 mm Hg, and
vancomycin? respirations are 18 breaths/min. She knows who she is
(A)Aerococcus and where she is but does not know the date. She is
(B)Gemella picking at the bedclothes. Rose spots are seen on her
(C)Pediococcus trunk. The remainder of the physical examination is
(D)Streptococcus normal. Blood cultures are done, and an intravenous line
(E)Abiotrophia is placed. The most likely cause of her illness is
(A)Enterotoxigenic Escherichia coli (ETEC)
(B)Shigella sonnei
(C)Salmonella enterica subspecies enterica serotype
Typhimurium (Salmonella Typhimurium)
(D)Salmonella enterica subspecies enterica serotype
Typhi (Salmonella Typhi)
(E)Enteroinvasive Escherichia coli (EIEC)

3. Blood cultures from the patient in question 2 grow a


non– lactose-fermenting gram-negative bacillus. Which
of the following is likely to be a constituent of this
organism?
(A)O antigen 157, H antigen 7 (O157:H7)
(B)Vi antigen (capsule; virulence antigen)
(C)O antigen 139 (O139)
(D)Urease
(E)K1 (capsular type 1)

4. A 37-year-old woman with a history of urinary tract


infections comes to the emergency department with
burning on urination along with frequency and urgency.
She says her urine smells like ammonia. The cause of her initially watery diarrhea. His mother became concerned
urinary tract infection is likely to be because the stools are now blood tinged 24 hours into
(A)Enterobacter aerogenes the illness, and the child appears quite ill. The mother
(B)Proteus mirabilis reports that two other children who attend the same aſt
(C)Citrobacter freundii
er-school daycare have recently had diarrheal disease,
(D)Escherichia coli
one of whom likewise had bloody stools. Which of the
(E)Serratia marcescens
following is the most likely pathogen causing the illness
in these children?
5. An 18-year-old student has abdominal cramps and
(A)An enterotoxigenic strain of Escherichia coli
diarrhea. A selective agar plate is inoculated and grows
(B)Salmonella enterica subspecies enterica serotype
suspicious gram-negative rods. Triple sugar iron agar is
Typhi (Salmonella Typhi)
used to identify the isolates as salmonellae or shigellae.
(C)Shigella sonnei
A result suggesting one of these two pathogens would be
(D)Edwardsiella tarda
(A)Production of urease
(E)Klebsiella oxytoca
(B)Motility in the medium
(C)Inability to ferment lactose and sucrose
9. A 5-year-old girl attended a birthday party at a local
(D)Fermentation of glucose
fast food restaurant. About 48 hours later, she
(E)Production of gas in the medium
developed cramping abdominal pain and a low-grade
fever and had five episodes of loose, bloody stools. She
6. An uncommon serotype of Salmonella enterica
is taken to a local emergency department the next
subspecies enterica was found by laboratories in the
evening because the diarrhea has continued, and she
health departments of adjacent states. The isolates were
now appears pale and lethargic. On presentation, she has
all from a small geographic area on either side of the
a temperature of 38°C, and she is hypotensive and
border between the states, suggesting a common source
tachycardic. The abdominal examination reveals
for the isolates. (All of the isolates were from otherwise
tenderness in the lower quadrants. Laboratory work is
healthy young adults who smoked marijuana; the same
remarkable for a serum creati-nine of 2.0 mg/dL, a serum
Salmonella was isolated from a specimen of the
hemoglobin of 8.0 mg/dL, thrombo-cytopenia, and
marijuana.) By what method did the public health
evidence of hemolysis. What is the most likely pathogen
laboratories determine that these isolates were the
causing this child’s illness?
same?
(A)Escherichia coli O157:H7
(A)Capsular (K antigen) typing
(B)Salmonella enterica subspecies enterica serotype
(B)O antigen and H antigen typing
Typhimurium
(C)DNA sequencing
(C)Enteropathogenic Escherichia coli
(D)Sugar fermentation pattern determination
(D)Edwardsiella tarda
(E)Decarboxylase reaction pattern determination
(E)Plesiomonas shigelloides
7. A 43-year-old man with diabetes has a 4-cm
10. A 55-year-old homeless man with alcoholism
nonhealing foot ulcer. Culture of the ulcer yields
presents with severe multilobar pneumonia. He requires
Staphylococcus aureus, Bacte-roides fragilis, and a gram-
intubation and mechanical ventilation. A Gram stain of
negative bacillus that swarms across the blood agar plate
his sputum reveals numerous polymorphonuclear
covering the entire surface of the agar aſter 36 hours. The leukocytes and gram-negative rods that appear to have
gram-negative bacillus is a member of the genus a capsule. The organism is a lactose fermenter on
(A)Escherichia MacConkey agar and is very mucoid. It is nonmo-tile and
(B)Enterobacter lysine decarboxylase positive. What is the most likely
(C)Serratia organism causing this man’s illness?
(D)Salmonella (A)Serratia marcescens
(E)Proteus (B)Enterobacter aerogenes
(C)Proteus mirabilis
8. A 4-year-old boy from Kansas City who recently started (D)Klebsiella pneumoniae
attending preschool and aſter-school daycare is brought (E)Morganella morganii
to his pediatrician for a diarrheal illness characterized by
fever to 38.2°C, severe lower abdominal pain, and
11. Which of the following statements regarding O Chapter XVI: Pseudomonads and Acinetobacter
antigens is correct?
(A)All Enterobacteriaceae possess identical O antigens. Review Questions
(B)They are found in the polysaccharide capsules of 1. A sputum culture of a patient with cystic fibrosis grows
enteric bacteria. Pseudo-monas aeruginosa that forms very mucoid
(C)They are covalently linked to a polysaccharide core. colonies. The implication of this observation is which one
(D)They do not stimulate an immune response in the of the following?
host. (A)The Pseudomonas aeruginosa is highly susceptible to
(E)They are not important in the pathogenesis of the aminoglycoside antimicrobial tobramycin.
infection caused by enteric bacteria. (B)The Pseudomonas aeruginosa is infected with a
pyocin (a bacteriocin).
12. Which of the following test methods is the least (C)The colonies are mucoid because they have
sensitive procedure for diagnosis of colitis caused by polysaccharide capsule of hyaluronic acid.
Shiga toxin–producing Escherichia coli? 250 SECTION III Bacteriology
(A)Culture on sorbitol MacConkey agar (D)The exotoxin A gene has been disabled and the
(B)Toxin testing using an enzyme immunoassay Pseudomonas aeruginosa is no longer able to block host
(C)Cell culture cytotoxin assay using Vero cells cell protein synthesis.
(D)Polymerase chain reaction for detection of the genes (E)The Pseudomonas aeruginosa has formed a biofilm in
that encode Shiga toxin the patient’s airway.

13. An HIV-positive man recently traveled to the 2. An environmental Gram-negative bacillus that is
Caribbean for a 2-week vacation. He developed acute resistant to cephalosporins, aminoglycosides, and
watery diarrhea and abdominal pain without fever quinolones has become a very important nosocomial
during the second week of his vacation. Tree weeks later, pathogen largely because it is selected by use of those
he is seen in clinic for persistent symptoms, and he is antibiotics. Tis Gram-negative bacillus can take 2–3 days
concerned because he is beginning to lose weight. Given to grow and must be differentiated from Burkholderia
this history, you suspect: cepacia. It is
(A)Enteroinvasive Escherichia coli (A)Pseudomonas aeruginosa
(B)Salmonella typhi (B)Acinetobacter baumannii
(C)Enteropathogenic Escherichia coli (C)Alcaligenes xylosoxidans
(D)Shigella flexneri (D)Klebsiella pneumoniae
(E)Enteroaggregative Escherichia coli (E)Stenotrophomonas maltophilia

14. Heat-labile toxin of ETEC acts by which of the 3. A 17-year-old girl with cystic fibrosis has a slight
following mechanisms? increase in her frequent cough and production of mucoid
(A)Attachment and effacement sputum. A sputum specimen is obtained and plated on
(B)Activation of adenylyl cyclase routine culture media. The predominant growths are
(C)Aggregative adherence
Gram-negative bacilli that form very mucoid colonies aſt
(D)Ribosomal dysfunction
(E)None of the above er 48 hours of incubation. These bacilli are oxidase
positive, grow at 42°C, and have a grapelike odor. Tese
15. A young woman presents with recurrent urinary tract Gram-negative bacilli are which of the following?
infections caused by the same Proteus mirabilis strain. (A) Klebsiella pneumoniae
What is the major concern? (B) Pseudomonas aeruginosa
(A)She does not take her medication. (C) Staphylococcus aureus
(B)She is pregnant because pregnant patients are more (D) Streptococcus pneumoniae
susceptible to UTIs. (E) Burkholderia cepacia
(C)She has a bladder or kidney stone.
(D)Her partner is infected.
(E)She has occult diabetes and should have a glucose
tolerance test.
4. The sputum from a 26-year-old patient with cystic 8. Patients deficient in these cells are at high risk for
fibrosis is plated on a colistin-containing agar. Aſter 72 developing serious systemic infections with
Pseudomonas aeruginosa:
hours of incubation, the colistin-containing agar grows
(A)Eosinophils
Gram-negative bacilli that are oxidase positive but are
(B)Neutrophils
otherwise difficult to identify. Tis microorganism is of
(C)Macrophages
major concern. It is sent to a reference laboratory so that
(D)Natural killer cells
molecular methods can be used to identify or rule out
(E)CD4+ T cells
which of the following?
(A)Pseudomonas aeruginosa
9. A marine wounded in Afghanistan returns to her home
(B)Burkholderia cepacia
a paraplegic. Her past medical history included surgery
(C)Haemophilus influenzae
to amputate both her legs below the knee and the
(D)Pseudomonas putida
placement of a suprapubic tube to repair damage to her
(E)Burkholderia pseudomallei
bladder. She is now at the VA outpatient clinic with a
recurrent urinary tract infection that has not responded
5. Acinetobacter species:
to conventional antibiotic regimens for community-
(A) Are only found in a hospital environment.
acquired cystitis. Her urine is positive for small, plump
(B)May appear as Gram-positive rods.
Gram-negative coccobacilli. When cultured, this
(C)Can mimic the morphology of Hemophilus species in
organism does not ferment carbohydrates, does not
Gram stains of endocervical secretions.
hydrolyze urea, does not reduce nitrates, and does not
(D)Can be a significant cause of ventilator-associated
make hydrogen sulfide. The organism most likely causing
pneumonia in intensive care unit patients.
this marine’s infection is:
(E)Are susceptible to most antibiotics.
(A)Klebsiella oxytoca
(B)Escherichia coli
6. A 37-year-old firefighter sustains smoke inhalation and
(C)Staphylococcus saprophyticus
is hospitalized for ventilatory support. He has a severe
(D)Proteus mirabilis
cough and begins to expectorate purulent sputum. Gram
(E)Acinetobacter baumanii
stain of his sputum specimen shows numerous
polymorphonuclear cells and numerous Gram-negative
10. A 70-year-old neutropenic patient was diagnosed
rods. Sputum culture grows numerous Gram-negative
rods that are oxidase positive. They grow well at 42°C. On with ecthyma gangrenosum 3 days aſter he developed a
clear agar medium, they produce a green color in the fever of 39°C. Blood cultures drawn the day his fever
agar. Te agar where the green color is located fluoresces started grew out overnight a strictly aerobic, Gram-
when exposed to ultraviolet light. The organism causing negative rod that was lactose negative and oxidase
the patient’s infection is positive. Which of the following antibiotic regimens
(A)Pseudomonas aeruginosa would be most appropriate for treating this patient?
(B)Klebsiella pneumoniae (A)Tobramycin + piperacillin/tazobactam
(C)Escherichia coli (B)Vancomycin + metronidazole
(D)Burkholderia cepacia (C)Cefazolin
(E)Burkholderia pseudomallei (D)Tigecycline
(E)Oxacillin
7. The mechanism of action of exotoxin A of
Pseudomonas aeruginosa is
(A)To activate acetylcholine esterase
(B)To block elongation factor 2
(C)To form pores in white blood cells and increase cation
permeability
(D)To increase intracellular cyclic adenosine
monophosphate
(E)To split lecithin into phosphorylcholine and
diacylglycerol
Chapter XVII: Vibrio, Campylobacter and Helicobacter 4. Bacteremia associated with a gastrointestinal infection
is most likely to occur with which of the following?
Review Questions (A)Salmonella typhi
1. Long-term carriage and shedding is most likely to occur (B)Vibrio cholerae
(C)Shigella boydii
aſter gastrointestinal infection with which of the
(D)Vibrio parahaemolyticus
following species? (E)Campylobacter jejuni
(A)Escherichia coli O157:H7
(B)Shigella dysenteriae 5. During the El Niño years in the mid- to late 1990s, the
(C)Vibrio cholerae waters of Puget Sound between Washington State and
(D)Campylobacter jejuni British Columbia warmed considerably. During this time,
(E)Salmonella typhi many people who ate clams and oysters from these
waters became ill with a disease characterized by
2. A 63-year-old man visited his favorite oyster explosive diarrhea and moderately severe abdominal
restaurant in a small town on the eastern shore of the cramps. The diarrhea was usually watery, but in some
Gulf Coast of Texas. He ate two dozen oysters. Two days patients, it was bloody. The diarrhea usually had an onset
later, he was admitted to the hospital because of an
abrupt onset of chills, fever, and light-headedness when within 24 hours aſter eating the shellfish. Stool cultures
he stood up. (In the emergency department [ED], his typically yielded a pathogenic gram-negative bacillus.
blood pressure was 60/40 mm Hg.) While in the ED, he The microorganism of concern in this setting is
developed erythematous skin lesions. These rapidly (A)Enterotoxigenic Escherichia coli
evolved into hemorrhagic bullae, which then formed (B)Vibrio cholerae
ulcers. The man drank a six-pack of beer and one half- (C)Enterohemorrhagic Escherichia coli
bottle of whisky each day. A microorganism of major (D)Vibrio parahaemolyticus
concern for this patient is (E)Shigella dysenteriae
(A)Vibrio vulnificus
(B)Escherichia coli 6. A patient presents to the emergency department with
(C)Salmonella typhi nonbloody diarrhea for 12 hours. The patient lives in
(D)Clostridium perfringens Washington, DC, and has not recently traveled out of the
(E)Streptococcus pyogenes (group A streptococci) area. Which one of the following is unlikely to be the
cause of your patient’s diarrhea?
3. A family of four persons ate a meal that included (A)Salmonella typhimurium
undercooked chicken. Within 3 days, three members (B)Campylobacter jejuni
developed an illness characterized by fever, headache, (C)Shigella sonnei
myalgia, and malaise. Two of the patients had (D)Vibrio cholerae
concomitant diarrhea and abdominal pain. The third (E)Escherichia coli
person developed diarrhea aſter the systemic symptoms
7. An 18-year-old woman in rural Bangladesh develops
had cleared. Stool cultures grew Campylobacter jejuni. profuse (8 L/d) diarrhea. She has no symptoms other
Which of the following culture conditions was most likely than the diarrhea and the manifestations of the fluid and
used to isolate C jejuni? electrolyte loss caused by the diarrhea. The most likely
(A)Tiosulfate-citrate-bile-sucrose medium incubated at cause of her diarrhea is
37°C in 5% oxygen and 10% CO2 (A)Campylobacter jejuni
(B)Salmonella-Shigella selective medium incubated at (B)Enterotoxigenic Escherichia coli
37°C in ambient air (C)Salmonella typhimurium
(C)MacConkey agar and Hektoen enteric agar incubated (D)Vibrio cholerae
at 42°C in 5% oxygen and 10% CO2 (E)Shigella dysenteriae
(D)5% sheep blood agar incubated at 37°C in ambient air
(E)A medium containing vancomycin, polymyxin B, and
trimethoprim incubated at 42°C in 5% oxygen and 10%
CO2
8. Age and geography are major factors in the prevalence (D)Pasteurizing milk
of colonization by Helicobacter pylori. In developing (E)Promoted washing vegetables that were consumed
countries, the prevalence of colonization may be greater raw
than 80% in adults. In the United States, the prevalence
of colonization with this microorganism in adults older 12. A 45-year-old man develops a gastric ulcer that can
than age 60 years is be visualized on a contrast medium–enhanced
(A)1–2% radiograph of his stomach. A biopsy specimen is taken
(B)5–10% from the gastric mucosa at the site of the ulcer. A
(C)15–20% presumptive diagnosis can be reached most rapidly by
(D)40–60% inoculating part of the specimen on which of the
(E)80–95% following?
(A)A medium used to detect urease incubated at 37°C
9. A 59-year-old man comes to the emergency (B)A medium containing vancomycin, polymyxin B, and
department in the aſternoon because of acute swelling trimethoprim incubated at 42°C
(C)MacConkey agar medium incubated at 37°C
and pain in his right leg. Earlier that morning, he had
(D)Tiosulfate-citrate-bile-sucrose medium incubated at
been working on a small sport fishing boat in an estuary
42°C
on the Gulf Coast of Texas. While walking around the
(E)Blood agar medium incubated at 37°C
boat in shallow water, he scratched his leg, breaking the
skin at the site of the current pain and swelling. He was
not wearing boots. About 1 hour aſter the injury, the
scratch became red and painful. Swelling developed.
Within 3 hours, the leg below the knee had become
markedly swollen. The skin was red and tender. There
was serous drainage from the wound, which had
ulcerated and was now much enlarged. Near the wound,
bullae were forming—the largest approximately 2.5 cm
in diameter. The most likely cause of this medical
emergency is
(A)Staphylococcus aureus
(B)Streptococcus pyogenes
(C)Clostridium perfringens
(D)Escherichia coli
(E)Vibrio vulnificus

10. The Vibrio cholerae factor responsible for diarrhea is


a toxin that
(A)Blocks EF-2
(B)Yields increased intracellular levels of cAMP
(C)Cleaves SNARE
(D)Blocks EF-1-dependent binding of amino-acyl-tRNA to
ribosomes
(E)Cleaves VAMP

11. In September 1854, a severe epidemic of cholera


occurred in the Soho/Golden Square area of London. Dr.
John Snow, a father of epidemiology, studied the
epidemic and helped stop it by which of the following
actions?
(A)Banning the sale of apples at the local markets
(B)Removing the handle of the Broad Street water pump
(C)Stopping the sale of shellfish imported from
Normandy
Chapter XVIII: Haemophilus, Bordetella, Brucella and (C)Salmonella Typhi
Francisella (D)Haemophilus ducreyi
(E)Francisella tularensis
Review Questions
1. A 68-year-old woman was seen in the clinic because 4. An 18-month-old boy has been playing with a child
she had felt feverish and had been experiencing who develops Haemophilus influenzae meningitis. The
boy’s parents consult his pediatrician, who says she is
increasing pain and swelling in her leſt knee during the comfortable that the child will be fine because he has
past 3 weeks. Four years earlier, a prosthetic joint had been fully immunized with the polyribitol ribose
been placed in her leſt knee. On examination, the knee phosphate (PRP)–protein conjugate vaccine. For what
was swollen, and fluid could be detected. An aspirate of reason is it necessary to immunize infants of 2 months to
the fluid was obtained. There were 15,000 2 years of age with polysaccharide–protein conjugate
polymorphonuclear cells/mL in the fluid. No organisms vaccines?
were seen on Gram stain. A routine culture was done. On (A)The conjugate protein is diphtheria toxoid, and the
the fourth day of incubation, colorless colonies smaller goal is for the infant to develop simultaneous immunity
than 1 mm in diameter were seen on the blood and to diphtheria.
chocolate agar plates. The organism was a tiny gram- (B)Infants 2 months to 2 years of age do not
negative coccobacillus that was catalase positive and immunologically respond to polysaccharide vaccines that
oxidase positive. A urea slant was inoculated and was are not conjugated to a protein.
(C)The conjugate vaccine is designed for older children
positive for urease activity aſter overnight incubation.
and adults as well as infants.
The patient was probably infected with which of the (D)Maternal (transplacental) antibodies against
following microorganisms? Haemophilus influenzae are gone from the infant’s
(A)Haemophilus influenzae circulation by 2 months of age.
(B)Haemophilus ducreyi (E)None of the above.
(C)Francisella tularensis
(D)Brucella species 5. An 11-year-old boy from Peru was referred to the
(E)Staphylococcus aureus Brain Tumor Institute. Three months earlier he had
developed headaches and then slowly progressive right-
2. Aſter the culture (Question 1) turned positive, sided weakness. A CT scan showed a mass lesion in the le
additional history was obtained. Approximately 4 weeks ſt hemisphere. He was thought to have a brain tumor. A
before the onset of her knee pain, the patient had visited lumbar puncture was not done because of concern about
relatives in Israel and traveled to other countries in the increased intracranial pressure and brain herniation
Mediterranean area. She had a particular fondness for through the tentorium cerebelli. During surgery, a mass
one food product that was the probable vehicle for her
infection. The product most likely was lesion in the leſt hemisphere was found. Frozen sections
(A)Bananas of the tissue were done while the patient was in the
(B)Unpasteurized goat’s cheese operating room. Microscopy of the sections showed a
(C)Rare hamburger granulomatous inflammatory reaction. No tumor was
(D)Fresh orange juice seen. Tissue was submitted for culture for
(E)Green tea Mycobacterium tuberculosis. Middlebrook 7H9 broth
medium was used. Six days aſter the culture was set up,
3. A 55-year-old game warden in Vermont found a dead the automated machine detected that the culture result
muskrat on the bank of a stream. He picked up the was positive. Results of an acid-fast stain and a Gram
animal, thinking it might have been illegally trapped or stain were both negative. Subcultures were done. Two
shot; it was not, and the game warden buried it. Four days later, very small colonies were seen on the sheep
days later, he developed a 1.5-cm painful ulcer on the blood agar plate. The organism was a tiny gram-negative
index finger of his right hand, a 1-cm ulcer on his right coccobacillus that was catalase positive and oxidase
forehead, and pain in his right axilla. Physical
positive. It showed urease activity aſter 2 hours of
examination also revealed right axillary
lymphadenopathy. Tis patient is most likely infected with incubation on urea-containing medium. Tis child had
(A)Brucella species infection with
(B)Rickettsia rickettsii (A)Brucella species
(B)Mycobacterium tuberculosis 9. Which of the following subspecies of Francisella
(C)Francisella tularensis tularensis is the most virulent for humans?
(D)Haemophilus influenzae (A)tularensis
(E)Moraxella catarrhalis (B)holarctica
(C)mediasiatica
6. A 3-year-old child develops Haemophilus influenzae (D)novicida
meningitis. Terapy is begun with cefotaxime. Why is this
third-generation cephalosporin used rather than 10. All of the following statements regarding the
ampicillin? etiologic agent of chancroid are correct except
(A)About 80% of Haemophilus influenzae organisms (A)The organism is a small gram-negative rod.
have modified penicillin-binding proteins that confer (B)The organism requires X factor but not V factor.
resistance to ampicillin. (C)The organism grows well on standard chocolate agar.
(B)The drug of choice, trimethoprim–sulfamethoxazole, (D)On Gram stain of lesions, the organism occurs in
cannot be used because the child is allergic to strands.
sulfonamides. (E)The organism is susceptible to erythromycin.
(C)It is easier to administer intravenous cefotaxime than
intravenous ampicillin. 11. A 3-month-old infant is brought to the pediatric
(D)There is concern that the child will rapidly develop a emergency department in severe respiratory distress.
penicillin (ampicillin) allergy. The child appears dehydrated, and there is a prominent
(E)About 20% of Haemophilus influenzae organisms have peripheral lymphocytosis. The chest radiograph reveals
a plasmid that encodes for β-lactamase. perihilar infiltrates. The child’s grandmother, who
watches the infant now that the mother has returned to
7. A 55-year-old man with severe dental caries presented work, has had a dry hacking cough for about 2 weeks. The
with 1 month of fever, malaise, and back pain and now most likely causative agent is
presents with moderately severe shortness of breath. (A)Haemophilus influenzae type b
The examination reveals a febrile man who appears pale (B)Bordetella pertussis
and dyspneic. Other physical findings include (C)Streptococcus agalactiae
conjunctival petechiae, a grade III/VI systolic murmur, (D)Chlamydia pneumoniae
and an enlarged spleen. Blood cultures grow a (E)Bordetella bronchiseptica
pleomorphic gram-negative rod that is not hemolytic and
that when tested is X and V factor negative. The most 12. In Question 11, the factor responsible for the
likely causative pathogen is profound lymphocytosis is
(A)Haemophilus influenzae (A)A hemagglutinin
(B)Haemophilus ducreyi (B)A polysaccharide capsule
(C)Aggregatibacter aphrophilus (C)An A/B structured toxin
(D)Actinobacillus hominis (D)A heat-labile toxin
(E)Haemophilus parainfluenzae (E)A neuraminidase

8. All of the following statements regarding acellular 13. All of the following cause zoonotic infections except
pertussis vaccines are correct except (A)Francisella tularensis
(A)All formulations of the vaccine contain at least two (B)Brucella melitensis
antigens. (C)Bordetella pertussis
(B)The acellular vaccine has replaced the whole cell (D)Bacillus anthracis
vaccine in the childhood vaccine series. (E)Leptospira interrogans
(C)All children should receive five doses of the vaccine
before school entry. 14. Which of the following is not a recognized virulence
(D)The vaccine is approved only for young children and factor of Bordetella pertussis?
adolescents. (A)Heat-labile toxin
(E)The vaccine is safer than and as immunogenic as (B)Filamentous hemagglutinin
whole-cell vaccines. (C)Tracheal cytotoxin
(D)Pertussis toxin
(E)Dermonecrotic toxin
15. Which of the following pathogens discussed in this Chapter XIX: Yersinia and Pasteurella
chapter is on the select agent list? Review Questions
(A)Haemophilus influenzae 1. An 18-year-old male resident of Arizona came to the
(B)Aggregatibacter aphrophilus emergency department (ED) complaining of fever, pain
(C)Bordetella pertussis
in his leſt groin, and diarrhea for the past 2 days. On
(D)Francisella tularensis
(E)All of the above examination, he was afebrile, had a pulse rate of 126
beats/min, a respiratory rate of 20 breaths/min, and a
blood pressure of 130/80 mm Hg. Leſt groin swelling and
tenderness were noted. A groin muscle strain was
diagnosed, attributed to a fall 2 days earlier. He was
treated with nonsteroidal anti-inflammatory drugs and
released. The next day, the patient reported feeling
weak, had difficulty breathing, and collapsed while taking
a shower. He was transported to a hospital ED and
pronounced dead shortly aſter arrival. Cultures of blood
samples obtained in the ED were positive for Yersinia
pestis. An epidemiologic investigation indicated that the
patient most likely became infected as a result of bites
by Y pestis–infected fleas while walking through a prairie
dog colony (see Chapter 48). Which of the following
statements about the pathogenesis of plague is correct?
(A)Yersinia pestis produces a coagulase when incubated
at 28°C.
(B)There is no risk for pneumonia caused by person-to-
person transmission of Yersinia pestis.
(C)Yersinia pestis organisms multiply in
polymorphonuclear cells.
(D)Aſter the bite of an infected flea, Yersinia pestis
infection seldom, if ever, disseminates beyond the site of
the flea bite and the regional lymph nodes.
(E)Yersinia pestis is transmitted to animals (and humans)
in flea feces excreted when the flea is feeding.

2. The drug of choice to treat the patient in Question 1


would have been
(A)Ampicillin
(B)Cefotaxime
(C)Levofloxacin
(D)Erythromycin
(E)Streptomycin

3. Yersinia pestis entered North America through San


Francisco in the 1890s, carried by rats on ships that had
sailed from Hong Kong, where a plague epidemic
occurred. The current reservoir for Y pestis in the United
States is
(A)Urban feral cats
(B)Urban rats
(C)Domestic cows
(D)Coyotes
(E)Rural wild rodents
4. Which of the following is generally not considered a (B)Humans acquire the infection from ingestion of food
potential agent of bioterrorism and biologic warfare? or drinks contaminated by animals or animal products.
(A)Yersinia pestis (C)Person-to-person spread is quite common.
(B)Botulinum toxin (D)A large inoculum is required to cause infection.
(C)Streptococcus pyogenes (E)Infection is more prevalent in persons with
(D)Brucella species histocompat-ibility antigen HLA-B27.
(E)Bacillus anthracis
10. Optimum recovery of Yersinia enterocolitica from the
5. An 8-year-old boy was bitten by a stray cat. Two days stools of patients with gastroenteritis requires which of
later, the wound was red and swollen and drained the following specialized media?
purulent fluid. Pasteurella multocida was cultured from (A)Cefsulodin-irgasan-novobiocin agar
the wound. The drug of choice to treat this infection is (B)Xylose-lysine decarboxylase agar
(A)Amikacin (C)Hektoen-enteric agar
(B)Erythromycin (D)Regan-Lowe medium
(C)Gentamicin (E)MacConkey agar
(D)Penicillin G
(E)Clindamycin 11. An organism suspected of being Yersinia pestis is
recovered from a patient with sepsis. The isolate has
6. Intimate contacts of patients with suspected plague bipolar staining and is catalase positive but is oxidase and
pneumonia should receive which of the following agents urease negative and is nonmotile. At this point, what
as chemoprophylaxis? should be done?
(A) Gentamicin (A)Nothing; the laboratory has confirmed the diagnosis.
(B) Cefazolin (B)Inoculate the isolate to an identification kit or
(C) Rifampin automated system for confirmation.
(D) Penicillin (C)Call the police because there is a possible bioterrorism
(E) Doxycycline event.
(D)Send the isolate to the nearest public health
7. In a patient who has the bubonic form of plague, all of laboratory for confirmation.
the following specimens are acceptable for diagnosis (E)Send the isolate to the hospital across town for
except sequencing.
(A)Stool culture on hektoen enteric agar
(B)Blood culture using routine laboratory media
(C)Culture of a lymph node aspirate on blood and
MacConkey agars
(D)Acute and convalescent serology
(E)Immunohistochemical staining of lymph node tissue

8. All of the following statements regarding the


pFra/pMT plasmid of Yersinia pestis are true except
(A)It encodes the capsular protein (fraction FI) that
confers antiphagocytic properties.
(B)It contains genes that yield plasminogen-activating
protease that has temperature-dependent coagulase
activity.
(C)It contains genes that encode phospholipase D, which
is required for organism survival in the flea midgut.
(D)It is unique to Yersinia pestis.
(E)It encodes factors that are important for survival in
both the flea and the human.
9. All of the following statements regarding the
epidemiology of infections caused by Yersinia
enterocolitica are correct except
(A)Most human infections are caused by serotype O:1.
Chapter XX: The Neisseriae association with her menstrual period. On pelvic
examination in the emergency department, she has
Review Questions bilateral tenderness when the uterus is palpated. A mass
1. The inhabitants of a group of small villages in rural sub- 2–3 cm in diameter is felt on the leſt, suggestive of tubo-
Saharan Africa experienced an epidemic of meningitis. ovarian abscess. Subsequently, Neisseria gon-orrhoeae is
Ten percent of the people died, most of them younger cultured from her endocervix. The diagnosis is
than the age of 15 years. The microorganism that most gonococcal pelvic inflammatory disease. A common
likely caused this epidemic was sequela of this infection is:
(A)Streptococcus agalactiae (group B) (A)Cancer of the cervix
(B)Escherichia coli K1 (capsular type 1) (B)Urethral stricture
(C)Haemophilus influenzae serotype b (C)Uterine fibroid tumors
(D)Neisseria meningitidis serogroup A (D)Infertility
(E)West Nile virus (E)Vaginal-rectal fistula

2. A 9-year-old boy presented to the clinic with a urethral 5. A 38-year-old vice squad police officer comes to the
discharge for the past 24 hours. Neisseria gonorrhoeae emergency department with a chief complaint expressed
was cultured from the specimen and found to be β- as follows: “I have disseminated gonococcal infection
lactamase positive and resistant to high levels (≥32 again.” He is correct. Cultures of his urethra and knee
μg/mL) of tetracycline. Which of the following fluid yield Neisseria gonorrhoeae. He has previously had
statements about these antimicrobial resistance factors five episodes of disseminated gonococcal infection. The
is correct? patient should be evaluated for
(A)β-lactamase production and high-level resistance to (A)Selective IgA deficiency
tetracycline are both mediated by genes on plasmids. (B)A polymorphonuclear cell chemotactic defect
(B)Whereas β-lactamase production is mediated by a (C)Deficiency of a late-acting complement component
gene on the bacterial chromosome, high-level C5, C6, C7, or C8
tetracycline resistance is mediated by a gene on a (D)Absent lymphocyte adenosine deaminase activity
plasmid. (E)Myeloperoxidase deficiency
(C)Whereas β-lactamase production is mediated by a
gene on a plasmid, high-level tetracycline resistance is 6. Which of the following individuals should routinely
mediated by a gene on the bacterial chromosome. receive vaccination with the conjugate meningococcal
(D)β-lactamase production and high-level resistance to vaccine?
tetracycline are both mediated by genes on the bacterial (A)A healthy young adolescent entering high school
chromosome. (B)A healthy child entering kindergarten
(C)A 60-year-old man with insulin-dependent diabetes
3. A 6-year-old boy develops a fever and headache. He is (D)A healthy 40-year-old technician who works in a
taken to the emergency department, where he is noted cancer research laboratory
to have a stiff neck, suggesting meningeal irritation. A (E)A 65-year-old woman with coronary artery disease
lumbar puncture is done, and culture of the
cerebrospinal fluid grows Neisseria meningitidis 7. A 25-year-old sexually active woman presents with
serogroup B. Which of the following should be
purulent vaginal discharge and dysuria 7 days aſter
considered for his family (household) members?
(A)No prophylaxis or other steps are necessary. having unprotected sexual intercourse with a new
(B)They should be given Neisseria meningitidis pilin partner. Of the choices below, what is the most sensitive
vaccine. diagnostic method for determining the likely etiologic
(C)They should be given Neisseria meningitidis serogroup agent?
B polysaccharide capsule vaccine. (A)Gram stain
(D)They should be given rifampin prophylaxis. (B)An enzyme immunoassay
(E)They should be given sulfonamide prophylaxis. (C)Bacterial culture on selective media
(D)A nucleic acid amplification test
4. An 18-year-old woman who reports unprotected sex (E)Serology
with a new partner 2 weeks previously develops fever
and leſt lower quadrant abdominal pain with onset in
8. What is the currently recommended treatment for (D)Neisseria gonorrhoeae
gonococcal urethritis in men who have sex with men in (E)None of the above
the United States?
(A)Single dose of an oral fluoroquinolone 13. All of the following are virulence factors associated
(B)Seven days of oral doxycycline with N gonorrhoeae except
(C)Ceſtriaxone given intramuscularly as a single dose (A)Pili
(B)Por
(D)Spectinomycin given intramuscularly as a single dose
(C)Lipooligosaccharide
(E)Seven days of oral amoxicillin
(D)Opa proteins
(E)A thick polysaccharide capsule
9. Which of the following cell components produced by
Neisseria gonorrhoeae is responsible for attachment to
14. The prevalence of gonococcal infections increased
host cells?
between 2009 and 2012.
(A)Lipooligosaccharide
(A) True
(B)Pili (fimbriae)
(B) False
(C)IgA1 protease
(D)Outer membrane porin protein
15. A useful test to differentiate Moraxella catarrhalis
(E)Iron-binding protein
from saprophytic neisseriae in respiratory samples is:
(A)Butyrate esterase
10. A 60-year-old man with severe chronic lung disease
(B)Gram stain
presents with fever, cough productive of purulent
(C)Growth on 5% sheep blood agar
sputum, and worsening hypoxemia. A sputum sample is
(D)PYR
collected, and the specimen is sent promptly to the
(E)Oxidase
laboratory. Microscopic examination of a Gram stain
reveals numerous polymorphonuclear leukocytes and
predominately gram-negative diplococci that are both
intracellular and extracellular. Te organism grows well on
5% SBA and chocolate agar and is positive for butyrate
esterase. What is the most likely organism causing this
man’s illness?
(A)Neisseria gonorrhoeae
(B)Neisseria lactamica
(C)Moraxella catarrhalis
(D)Haemophilus influenzae
(E)Neisseria meningitidis

11. One major advantage of the conjugate


meningococcal vaccines compared with the
polysaccharide vaccine is
(A) Stimulation of mucosal secretory IgA
(B) Fewer side effects
(C) A T cell–dependent response to vaccine is induced
(D) Inclusion of serogroup B

12. A 25-year-old woman presents with septic arthritis of


the knee. The fluid aspirate grows a gram-negative
diplococcus on chocolate agar aſter 48 hours of
incubation. Te isolate is oxidase positive and oxidizes
glucose but not maltose, lactose, or sucrose. You suspect
infection with:
(A)Neisseria meningitidis
(B)Neisseria lactamica
(C)Moraxella catarrhalis
Chapter XXI: Infections Caused by Anaerobic Bacteria (C)Prevotella melaninogenica
(D)Clostridium tetani
Review Questions (E)Actinomyces israelii
1. A 55-year-old man visits his physician complaining of a
severe cough and production of purulent sputum. His 5. The drug of choice for treatment of infections caused
breath has a very unpleasant fetid odor. Chest by Actinomyces species is
(A)Tigecycline
radiography shows a large amount of fluid in the leſt
(B)Cefoxitin
pleural space and a 5-cm lung cavity with an air-fluid (C)Metronidazole
level. A needle is inserted through the chest wall, and (D)Imipenem
some of the fluid in the pleural space is removed; it is (E)Penicillin
thick, yellow-gray in color, and malodorous. Which of the
following organisms or sets of organisms are most likely 6. All of the following statements regarding anaerobes are
to be cultured from the pleural fluid? true except
(A)Bacteroides fragilis, Escherichia coli, and enterococci (A)They possess the enzyme cytochrome oxidase
(B)Prevotella bivia, peptostreptococci, and (B)Many species are part of the normal human microbiota
Staphylococcus epidermidis (C)They are oſten found along with aerobes in complicated
(C)Prevotella melaninogenica, Fusobacterium species,
infections
and viri-dans streptococci
(D)Special techniques are required to ensure their recovery
(D)Propionibacterium species, peptostreptococci, and from clinical specimens
Staphylococcus aureus (E)Some species are more tolerant of exposure to oxygen
(E)Streptococcus pneumoniae than others

2. An 18-year-old man develops fever with pain in the 7. Lemierre’s disease is a serious infection of the head and
right lower quadrant of his abdomen. Aſter initial neck associated with which of the following anaerobes?
evaluation, he is taken to the operating room. During (A)Prevotella melaninogenica
surgery, a ruptured appendix with an abscess is found. (B)Bacteroides thetaiotamicron
(C)Porphyromonas gingivalis
Bacteroides fragilis is cultured from the abscess fluid.
(D)Peptococcus niger
Which of the following factors promote abscess
(E)Fusobacterium necrophorum
formation by B fragilis?
(A)Lipopolysaccharide
8. Definitive identification of an anaerobe is likely best
(B)Capsule accomplished by
(C)Superoxide dismutase (A)Colony morphology on anaerobic media
(D)Pili (B)The presence of pigment
(E)Leukocidin toxin (C)Susceptibility to a variety of antimicrobial disks
(D)Cell wall fatty acid analysis using gas liquid
3. Infections caused by Bacteroides species can be chromatography
treated with all of the following antibiotics except (E)Gram stain morphology
(A)Ampicillin–sulbactam
(B)Clindamycin 9. A patient who has not maintained good dentition
(C)Metronidazole presents with induration and swelling of the mandibular
(D)Penicillin area. On examination, you note purulent material draining
(E)Cefoxitin from a small opening. The material appears yellowish, and
there are some visible granules. You perform a Gram stain,
4. A 17-year-old high school senior develops infectious and pleomorphic gram-positive rods with short branches
mononucleosis. About 2 weeks later, he develops a are noted along with cells suggestive of acute and chronic
significantly higher fever, a worsening sore throat, an inflammation. You suspect which of the following
inability to swallow, and severe neck and chest pain. organisms?
Upon admission, he has signs of sepsis and respiratory (A)Bacteroides fragilis
(B)Lactobacillus acidophilus
distress. What is the most likely organism causing this
(C)Clostridium perfringens
complication?
(D)Actinomyces israelii
(A)Fusobacterium necrophorum
(E)Staphylococcus aureus
(B)Bacteroides ovatus
Chapter XXII: Legionella, Bartonella and Unusual over the right lower lung field. Chest radiography shows
Bacterial Pathogens a patchy right lower lobe infiltrate. The differential
diagnosis of this patient’s disease is
Review Questions (A)Streptococcus pneumoniae pneumonia
1. Humans become infected with Legionella (B)Legionella pneumophila pneumonia
pneumophila by (C)Haemophilus influenzae pneumonia
(A)Kissing a person who is a legionella carrier (D)Mycoplasma pneumoniae pneumonia
(B)Breathing aerosols from environmental water sources (E)All of the above
(C)Receiving a mosquito bite
(D)Consuming undercooked pork 5. Routine sputum cultures for the patient in Question 4
grow normal microbiota. Treatment with ampicillin for 2
2. An 11-year-old girl developed an acute onset of fever, days yields no improvement. A diagnosis of Legionnaires’
chills, headache, vomiting, and severe migratory disease is considered, and bronchoscopy is done to
arthralgias (joint pain) and myalgias (muscle pain). Two obtain bronchial alveolar lavage fluid and deep airway
days later, she developed a maculopapular rash over her specimens. Which of the following would suggest a
diagnosis of disease caused by Legionella pneumophila
palms, soles, and extremities. At the same time, her leſt serotype 1?
knee became extremely painful and swollen. On (A) Legionella urinary antigen assay
examination, fluid was demonstrated in the knee. (B) Direct fluorescent antibody on the bronchial alveolar
Further history disclosed that the patient had a pet rat. lavage fluid
Culture of the fluid from her knee on 5% sheep blood (C) Culture of the bronchial alveolar lavage on charcoal
agar showed 2-mm colonies aſter 3 days of incubation. yeast extract medium with antibiotics
Broth culture showed small puffball-like growth. Gram (D) Antibody assay on paired (acute phase and
staining showed a gram-negative bacillus 0.5 μm wide convalescent phase) sera
and 1–4 μm long. Some extremely long forms (up to 150 (E) All of the above
μm) with beadlike chains, fusiform swellings, and large
round bodies were seen. The microbiologist who 6. Charcoal is present in buffered charcoal yeast extract
observed the Gram-stained smear immediately knew the agar used to isolate Legionella pneumophila to
cause of the girl’s infection to be (A)Provide the growth factors ordinarily provided by
(A)Pasteurella multocida free-living amebas present in environmental water
(B)Streptobacillus moniliformis (B)Serve as a carbon source for the growth of Legionella
(C)Francisella tularensis pneumophila
(D)Bartonella bacilliformis (C)Prevent hemolysis of the red blood cells in the
(E)Yersinia pestis medium
(D)Provide a dark background
3. A 70-year-old man presents with bilateral pneumonia. (E)Act as a detoxifying agent
His Legionella urinary antigen test result is positive.
Which of the following is the likely cause of his 7. A 23-year-old, otherwise healthy woman presents
pneumonia? with a 3-day history of low-grade fever and headache.
(A)Legionella pneumophila serogroup 1 Examination reveals enlarged and slightly tender lymph
(B)Legionella micdadei serogroup 4 nodes near her leſt elbow and in the leſt axilla.
(C)Legionella bozemanii serogroup 2 Approximately 2 weeks earlier, she had visited a friend
(D)Legionella longbeachae serogroup 2
whose cat had scratched her on the leſt arm; the site later
(E)All of the above because the urinary antigen test is
genus specific and not species or serotype specific. developed a reddish papule. Which of the following
statements about this disease is correct?
4. A 65-year-old man comes to the emergency (A)Characteristic histopathology in response to infection
department feeling feverish and “really tired.” He has a is acute, neutrophilic inflammation.
chronic cigarette cough, but this has dramatically (B)The diagnosis is based on a suggestive history and
increased in the past week and he has been producing physical examination.
whitish sputum. The previous day he had a temperature (C)β-Lactam/β-lactamase inhibitor combinations are the
of 38°C and watery diarrhea. Physical examination agents of choice for treatment.
reveals inspiratory and expiratory wheezes and rales
(D)The diagnosis is based on negative routine bacterial 12. Which of the following best represents the role of the
cultures of pus aspirated from involved lymph nodes. Mip protein in Legionella pathogenesis?
(E)The disease rapidly leads to sepsis even in (A)It prevents phagosome–lysosome fusion.
immunocompetent people. (B)It acts as a siderophore to capture iron.
(C)It prevents phagocytosis.
8. Which of the following statements about bacillary (D)It facilitates adherence to the macrophage and
angiomato-sis is most correct? stimulates cellular invasion.
(A)It is caused by Bartonella bacilliformis. (E)None of the above.
(B)It is typically confined to the skin.
(C)The major differential diagnosis is Kaposi sarcoma. 13. Pontiac fever is a severe form of pneumonia caused
(D)The etiologic agent can be grown in 1–2 days in by Legio-nella pneumophila serotypes 1 and 6.
routine culture on sheep blood agar. (A) True
(E)Dogs are the reservoir for the etiologic agent. (B) False

9. An important factor in the pathogenesis of 14. All of the following statements regarding
Legionnaires’ disease is that Streptobacillus monil-iformis are correct except
(A)Legionella pneumophila kills polymorphonuclear (A)It is susceptible to penicillin.
cells. (B)It causes the disease rat-bite fever.
(B)Alveolar macrophages phagocytose Legionella (C)It causes Haverhill fever from ingestion of
pneumophila using coiled pseudopods. contaminated food.
(C)Legionella pneumophila invades pulmonary (D)The morphology of the organism is spiral shaped.
capillaries, leading to dissemination and systemic illness.
(D)Legionella pneumophila induces alveolar macrophage 15. The diagnosis of Whipple’s disease is best made by
phagosomes to fuse with lysosomes. (A)Paired serum obtained 8 weeks apart
(E)Legionella pneumophila outer surface protein A (B)Prolonged culture on mycobacterial media
(OspA) is important for invasion of alveolar (C)Nucleic acid amplification testing performed on tissue
macrophages. (D)Histopathology
(E)None of the above
10. True statements regarding Tropheryma whipplei
include all of the following except
(A)It is easy to cultivate on chocolate agar aſter 3 days of
incubation.
(B)It is a gram-positive actinomycete.
(C)It causes fever, abdominal pain, diarrhea, weight loss,
and migratory polyarthralgia.
(D)It stains with PAS.

11. All of the statements below regarding infections with


Legionella are correct except
(A)Hospitals that care for patients at risk for Legionella
infections should know if their potable water systems
contain Legionella.
(B)Human-to-human transmission is the major
mechanism of transmission of Legionella infection.
(C)Legionella species can be visualized with Gram stain if
carbolfuchsin is used for the counter stain.
(D)The chest radiograph of a patient who has Legionella
pneumonia is indistinguishable from that of patients
with pneumonia caused by other pathogens.
(E)A macrolide or quinolone are the drugs of first choice
for treatment of Legionella infections.
Chapter XXIII: Mycobacteria (B)Mycobacterium fortuitum
(C)Mycobacterium ulcerans
Review Questions (D)Mycobacterium gordonae
1. A 60-year-old man has a 5-month history of (E)Mycobacterium tuberculosis
progressive weakness and a weight loss of 13 kg along
with intermittent fever, chills, and a chronic cough 5. It is very important that the patient in question 4 also
productive of yellow sputum, occasionally streaked with be evaluated for
blood. A sputum specimen is obtained, and numerous (A)HIV/AIDS
acid-fast bacteria are seen on the smear. Culture of the (B)Typhoid fever
sputum is positive for M tuberculosis. Which treatment (C)Liver abscess
regimen is most appropriate for initial therapy? (D)Lymphoma
(A)Isoniazid and rifampin (E)Malaria
(B)Sulfamethoxazole–trimethoprim and streptomycin
(C)Isoniazid, rifampin, pyrazinamide, and ethambutol 6. Of concern regarding the patient in question 4 is that
(D)Isoniazid, cycloserine, and ciprofloxacin she could be infected with a Mycobacterium that is
(E)Rifampin and streptomycin (A) Susceptible only to isoniazid
(B) Resistant to streptomycin
2. If the patient’s M tuberculosis isolate (question 1) (C) Resistant to clarithromycin
proves to be resistant to isoniazid, the likely mechanism (D) Susceptible only to ciprofloxacin
for resistance is (E) Resistant to isoniazid and rifampin
(A)β-Lactamase
(B)Mutations in the catalase-peroxidase gene 7. You observe a 40-year-old man begging on a street in
(C)Alterations in the β subunit of RNA polymerase a town in India. He has clawing of the fourth and fiſth
(D)Mutations in the DNA gyrase gene digits with loss of distal parts of the digits of both hands,
(E)Mutations in the genes encoding the S12 protein and strongly suggesting leprosy. The causative agent of this
16S rRNA disease
(A)Is susceptible to isoniazid and rifampin
3. A 47-year-old woman presents with a 3-month history (B)Grows in parts of the body that are cooler than 37°C
of progressive cough, weight loss, and fever. Chest (C)Can be cultured in the laboratory using Middlebrook
radiography shows bilateral cavitary disease suggestive 7H11 medium
of tuberculosis. Sputum culture grows an acid-fast (D)Is seen in high numbers in biopsies of tuberculoid
bacillus that is a photochromogen (makes an orange leprosy lesions
pigment when exposed to light). The organism most (E)Commonly infects people in Texas because armadillos
likely is are hosts of M leprae
(A)Mycobacterium tuberculosis
(B)Mycobacterium kansasii 8. Which of the following statements about the purified
(C)Mycobacterium gordonae protein derivative (PPD) and the tuberculin skin test is
(D)Mycobacterium avium complex most correct?
(E)Mycobacterium fortuitum (A)It is strongly recommended that medical and other
health science students have PPD skin tests every 5
4. A 31-year-old Asian woman is admitted to the hospital years.
with a 7-week history of increasing malaise, myalgia, (B)Persons immunized with BCG rarely, if ever, convert
nonproductive cough, and shortness of breath. She has to positive PPD skin test results.
daily fevers of 38–39°C and a recent 5-kg weight loss. She
(C)The intradermal skin test is usually read 4 hours aſter
had a negative chest radio-graph when she entered the
United States 7 years ago. The patient’s grandmother being applied.
died of tuberculosis when the patient was an infant. A (D)A positive tuberculin test result indicates that an
current chest radiograph is normal; results of other tests individual has been infected with M tuberculosis in the
show a decreased hematocrit and liver function test past and may continue to carry viable mycobacteria.
abnormalities. Liver and bone marrow biopsies show (E)A positive PPD skin test result implies that a person is
granulomas with giant cells and acid-fast bacilli. She is immune to active tuberculosis.
probably infected with
(A)Mycobacterium leprae
9. A 72-year-old woman has an artificial hip joint placed (D)Hispanic men who work outdoors
because of degenerative joint disease. One week aſter (E)Persons living in the Northwestern United States
the procedure, she has fever and joint pain. The hip is
13. A newly characterized rapidly growing
aspirated, and the fluid is submitted for routine culture
Mycobacterium that has emerged as an important cause
and for culture for acid-fast organisms. Aſter 2 days of of central venous catheter associated infections is
incubation, there is no growth on any of the media. Aſter (A)Mycobacterium phlei
4 days, however, bacilli are seen growing on the sheep (B)Mycobacterium mucogenicum
blood agar plate, and similar-appearing acid-fast bacilli (C)Mycobacterium xenopi
are growing on the culture for acid-fast bacteria. The (D)Mycobacterium smegmatis
patient is most likely infected with (E)Mycobacterium terrae
(A)Mycobacterium tuberculosis
(B)Mycobacterium chelonae 14. The definition of extensively drug-resistant (XDR)
(C)Mycobacterium leprae tuberculosis includes
(D)Mycobacterium kansasii (A)Resistance to isoniazid
(E)Mycobacterium avium complex (B)Resistance to a fluoroquinolone
(C)Resistance to capreomycin, amikacin or kanamycin
10. A 10-year-old child has a primary pulmonary M (D)Resistance to rifampin
tuberculosis infection. Which of the following features of (E)All of the above
tuberculosis is most correct?
(A)In primary tuberculosis, an active exudative lesion 15. All of the following organisms are rapidly growing
develops and rapidly spreads to lymphatics and regional mycobacteria except
lymph nodes. (A)Mycobacterium fortuitum
(B)Mycobacterium abscessus
(B)The exudative lesion of primary tuberculosis oſten
(C)Mycobacterium mucogenicum
heals slowly. (D)Mycobacterium nonchromogenicum
(C)If tuberculosis develops years later, it is a result of (E)Mycobacterium chelonae
another exposure to M tuberculosis.
(D)In primary tuberculosis, all of the infecting M
tuberculosis organisms are killed by the patient’s
immune response.
(E)In primary tuberculosis, the immune system is primed,
but the PPD skin test result remains negative until there
is a second exposure to M tuberculosis.

11. Which of the following statements regarding


interferon-γ release assays (IGRAs) is correct?
(A)They are useful for evaluating immunocompromised
patients for active tuberculosis.
(B)They detect antigens present in all Mycobacterium
species.
(C)They are not available yet for testing in the United
States.
(D)They are performed using molecular probes that
detect organism DNA.
(E)They are used as alternatives to the tuberculin skin
test to evaluate for latent tuberculosis.

12. M abscessus most oſten causes pulmonary disease


among which group of individuals?
(A)Young children exposed to dirt
(B)African American smokers
(C)Elderly, nonsmoking white females
Chapter XXIV: Spirochetes and Other Spiral blood smear stained with Wright stain showed
Microorganisms spirochetes suggestive of Borrelia species. Which of the
following statements about relapsing fever is correct?
Review Questions (A)Each relapse is associated with an antigenically
1. A 28-year-old woman who is 10 weeks pregnant distinct variant.
presents to the obstetrics clinic for prenatal care. She has (B)Blood smears should be made when the patient is
a history of treatment for syphilis 7 years previously. The afebrile.
results of serologic tests for syphilis are as follows: (C)Borreliae do not pass transovarially from one
nontreponemal test, RPR, nonreactive; treponemal test generation to the next in ticks.
(TP-PA), reactive. Which of the following statements is (D)The main reservoir for the Borrelia is deer.
most correct? (E)Borrelia is resistant to penicillin and tetracycline.
(A)The mother’s previous treatment for syphilis was
effective. 5. A 23-year-old man presented with a maculopapular
(B)The baby is at high risk for congenital syphilis. rash over much of his trunk but not in his mouth or on
(C)The mother needs to be treated again for syphilis. his palms. Because secondary syphilis was considered in
(D)The mother needs a lumbar puncture and a VDRL test the differential diagnosis, a RPR test was done, and the
of her CSF for neurosyphilis. result was positive at a 1:2 dilution. However, the TP-PA
test result was negative. Which of the following diseases
2. A 12-year-old Boy Scout went to summer camp for 2 can be ruled out?
weeks in late August at a site located just outside Mystic, (A)Secondary syphilis
Connecticut. When he returned home, his mother (B)Atypical measles
noticed a bull’s-eye-shaped rash on the back of her son’s (C)Coxsackie virus infection
(D)Acute HIV 1 infection
leſt calf. Shortly aſter Labor Day, the boy developed a
(E)Allergic drug reaction
flulike illness that resolved aſter 4 days of bed rest. Tree
weeks later, the boy complained to his mother that his 6. Which of the following animals is the source of
body hurt all over whenever he moved. Tis prompted a Leptospira interrogans?
visit to the pediatrician, who ordered an infectious (A)Alligators
disease workup. What is the most likely source of the (B)Ducks
boy’s infection? (C)Frogs
(A)Respiratory transmission from another sick camper (D)Catfish
(B)Ingestion of urine-contaminated water from a stream (E)Swine
(C)The bite of a mosquito harboring a parasite
(D)Ingestion of fecally contaminated food 7. A 27-year-old medical resident was admitted to the
(E)The bite of a spirochete-infected tick hospital because of sudden onset of fever to 39°C and
headache. Two weeks previously, he had vacationed in
3. Nontreponemal serological tests: rural Oregon, where he had frequently gone swimming
(A)Are useful in definitively identifying a Treponema in an irrigation canal that bordered land where cows
pallidum infection. were pastured. Blood tests done shortly aſter admission
(B)Measure antibodies against Treponema pallidum. indicated renal function abnormality and elevated
(C)Can be used to monitor antibiotic treatment of bilirubin and other liver function test results. Routine
primary or secondary syphilis. blood, urine, and CSF culture results were negative.
(D)Measure antibodies against lipids released from Leptospirosis is suspected. Which of the following would
damaged cells. be most likely to confirm this diagnosis?
(E)Are useful in diagnosing a disseminated gonococcal (A)Testing acute and convalescent phase sera using the
infection. RPR test
(B)Culture of urine on human diploid fibroblast cells
4. A 42-year-old woman went camping in the Sierra (C)Testing serum by dark-field examination for the
Nevada Mountains, where she slept for two nights in an presence of leptospires
abandoned log cabin. Aſter the second night, a tick was (D)Testing acute and convalescent phase sera for
found on her shoulder. Six days later, she developed antileptospiral antibodies
fever to 38°C, which lasted for 4 days. Ten days later, she (E)Culture of CSF on blood and chocolate agar
had another similar episode of fever. Examination of a (F)Gram stain of CSF and blood
8. A 47-year-old man presents with slowly progressive Chapter XXV: Mycoplasmas and Cell Wall-Defective
arthritis in his knees. He enjoys hiking in the coastal areas Bacteria
of Northern California, where the prevalence of Borrelia
burgdorferi in the Ixodes ticks is known to be 1–3% Review Questions
(considered low). The patient is concerned about Lyme 1. Ureaplasma urealyticum is so named because
disease. He never noticed a tick on his body and did not (A) It thrives in the upper urinary tract.
see an expanding red rash. The result of an EIA for Lyme (B)It requires urea as a growth substrate.
borreliosis is positive. What should be done now? (C)It is a frequent cause of symptomatic urinary bladder
(A)A biopsy specimen of the synovium of a knee joint infections in young women.
should be examined for Borrelia burgdorferi. (D)It causes chronic urinary tract infections in premature
(B)The patient should be given an antibiotic to treat Lyme babies born to mothers with ureaplasmas as part of the
disease. genital flora.
(C)PCR on the patient’s plasma should be done to detect
Borrelia burgdorferi.
2. An 18-year-old sexually active woman develops leſt
(D)A serum specimen should be submitted for
immunoblot assay to detect antibodies reactive with lower quadrant pain and fever. On pelvic examination,
Borrelia burgdorferi antigens. she has tenderness in the leſt adnexa, and a mass
(E)Culture of synovial fluid on blood and chocolate agar. suggestive of a uterine tube abscess is palpated. The
patient is diagnosed with pelvic inflammatory disease.
9. Which of the following organisms principally infects Which of the following bacteria is considered to be a
the liver and kidneys? common cause of pelvic inflammatory disease?
(A)Leptospira interrogans (A)Bacillus cereus
(B)Staphylococcus aureus (B)Haemophilus influenzae
(C)Escherichia coli (C)Neisseria subflava
(D)Enterococcus faecalis (D)Mycoplasma pneumoniae
(E)Treponema pallidum (E)Chlamydia trachomatis

10. All of the following statements regarding relapsing 3. Which of the following is important in the
fever are correct except pathogenesis of mycoplasmal infections?
(A)Epidemic disease carries a higher mortality rate than (A)The peptidoglycan in the mycoplasmal cell wall
endemic disease. (B)The presence of lacto-N-neotetraose with a terminal
(B)Endemic disease in North America is caused by galactosamine as the host cell receptor
Borrelia recurrentis. (C)The structures and the interactive proteins that
(C)The recurrent febrile episodes are caused by antigenic mediate adhesion to host cells
variation among the spirochetes. (D)The absence of cilia on the surface of the host cells
(D)Penicillin is the drug of choice. (E)Growth in an anatomic site where anaerobic
(E)Crushing a tick could transmit the spirochetes. organisms thrive

4. A 25-year-old woman is referred to the sexually


transmitted diseases clinic because of contact with a
male partner with gonorrhea. The woman has had 15
male sex partners since becoming sexually active. The
likelihood that she also has genital Mycoplasma hominis
infection is
(A)1%
(B)5%
(C)15%
(D)40%
(E)90%

5. A 25-year-old medical student has contact with a


patient who has pneumonia with fever and cough. Four
days later, the medical student develops fever and
cough, and chest radiographs show consolidation of the 10. Infection with Mycoplasma genitalium:
right lower lobe. Routine bacterial sputum culture results (A) is not restricted to the genitourinary tract.
are negative. Pneumonia caused by Mycoplasma (B)results in inflammation causing urethritis in males and
pneumoniae is considered. All of the following are cervicitis in females.
methods to confirm the clinical suspicion except (C)is best treated with a first-generation cephalosporin.
(A)PCR amplification of Mycoplasma pneumoniae DNA in (D)is associated only with nongonococcal urethritis in
sputum males.
(B)Culture of sputum for Mycoplasma pneumoniae (E)is asymptomatic unless a co-infection with Clamydia
(C)Gram stain of sputum smear trachomatis is present.
(D)Culture of a lung aspirate for Mycoplasma
pneumoniae
(E)Enzyme immunoassay test of acute and convalescent
sera

6. Which type of test is most readily used to obtain


laboratory confirmation of Mycoplasma pneumoniae
infection?
(A)Culture in broth containing serum, glucose, and a
penicillin (to inhibit other flora)
(B)PCR
(C)Electron microscopy
(D)EIA tests on acute and convalescent phase sera

7. A 13-year-old boy develops infection with


Mycoplasma pneumoniae. What is the risk for infection
in other members of his household?
(A)None; it is sexually transmitted
(B)1–3%
(C)10–15%
(D)20–40%
(E)50–90%

8. A 19-year-old man develops cough and fever. A chest


radio-graph shows consolidation of the leſt lower lobe. A
diagnosis of pneumonia is made. Which of the following
bacteria is a frequent cause of community-acquired
pneumonia?
(A)Legionella pneumophila
(B)Chlamydia pneumoniae
(C)Streptococcus pneumoniae
(D)Mycoplasma pneumoniae
(E)Klebsiella pneumoniae

9. Initiation of infection by Mycoplasma pneumoniae


begins with
(A) Elaboration of a polysaccharide capsule that inhibits
phagocytosis
(B)Secretion of a potent exotoxin
(C)Endocytosis by ciliated respiratory epithelial cells
(D)Adherence to respiratory epithelial cells mediated by
P1 adhesin
(E)Phagocytic uptake by alveolar macrophages
Chapter XXVI: Rickettsia and Related Genera (C)Ampicillin is the treatment of choice.
(D)Culture is a good method to confirm the diagnosis.
Review Questions (E)Ehrlichiae are typically found in lymphocytes.
1. Morulae (intracellular inclusions in leukocytes) are
characteristic of which of the following diseases? 6. A group of urban teenagers visited a sheep ranch in a
(A)Malaria caused by Plasmodium falciparum infection large Western state for a 2-week experience. While they
but not Plasmodium malariae infection were there, many of the pregnant ewes delivered lambs
(B)Dengue to the delight of the closely observing teenagers. About
(C)Babesia infection 10 days later, three of the teenagers developed flulike
(D)Ehrlichiae infection illnesses characterized by malaise, cough, and fever. One
(E)Loa loa had an infiltrate on chest radiography, indicating
pneumonia. The three teenagers had different doctors,
2. Which of the following statements about epidemic but the physicians each drew a blood specimen and
typhus (Rickettsia prowazekii disease) is most correct? submitted it to the city health department for serologic
(A)The disease occurs primarily in sub-Saharan Africa. testing. All three specimens were positive for Q fever.
(B)It is transmitted by ticks. Public health investigators determined that all of the
(C)Mice are the reservoir. teenagers had been to the sheep ranch. When the
(D)Historically, the disease occurs in times of prosperity. investigators contacted the ranch, they were told that
there was no Q fever there and that no one who lived at
(E)Recrudescence can occur many years aſter the initial
the ranch had been sick. The most likely explanation for
infection. the teenagers’ illnesses and the lack of illness at the
ranch is
3. The most useful drug to treat ehrlichiosis is (A)There was no Q fever at the ranch, and it was acquired
(A)Doxycycline elsewhere.
(B)Penicillin G (B)The people at the ranch had been previously
(C)Trimethoprim–sulfamethoxazole immunized against Q fever.
(D)Gentamicin (C)The teenagers acquired Q fever at the ranch, and the
(E)Nitrofurantoin people who lived there had all previously had Q fever and
were now immune to it.
4. A disease characterized by malaise, headache, rigors, (D)The teenagers had other illnesses, and the positive Q
and fever developed in members of several families living fever serology result was unrelated.
in an unheated war-damaged house in an Eastern (E)The public health laboratory had errors in the Q fever
European country. Erythematous 2–6 mm macular red serologic tests.
rashes appeared on the peoples’ trunks and later on their
extremities. Some of the people had coughs. One elderly 7. A middle-aged sportsman, a resident of Oklahoma,
person, although sick, was much less sick than other took a hike through a rural wooded and brushy area near
adults. The people huddled together to keep warm; body his home. The next morning, he noticed and removed a
lice were common. Which of the following statements is large (>1 cm) tick from his upper arm. About 1 week later,
most correct? he experienced a gradual onset of fever and malaise. He
(A)The disease that these people had is common in the now seeks medical attention because he is concerned
Rocky Mountain states. about a possible infection transmitted by the tick. Which
(B)The elderly person may have had acute epidemic of the following diseases is most likely to be acquired
typhus many years ago and recrudescent typhus now. from a tick?
(C)Fleas from rodents in the house were spreading (A)Dengue
Rickettsia typhi. (B)Rocky Mountain spotted fever
(D)The primary host of the body louse infecting the (C)Typhus
people is the rat. (D)Yellow fever
(E)Epidemic typhus can be prevented by a vaccine. (E)Malaria

5. Which of the following statements about Ehrlichiae 8. Which of the following drugs should not be used to
and ehrlichiosis is most correct? treat Rocky Mountain spotted fever (Rickettsia rickettsii
(A)Dogs and mice are reservoirs. infection)?
(B)Mosquitoes are the vectors. (A)Trimethoprim–sulfamethoxazole
(B)Chloramphenicol (B)Morulae within monocytes
(C)Doxycycline (C)Granulomatous inflammation
(D)Intracellular vacuoles
9. Which of the following should be used to prevent (E)Perivascular lymphocytes
Rocky Mountain spotted fever (Rickettsia rickettsii
infection)? 14. All of the following statements regarding
(A) Attenuated Rickettsia rickettsii vaccine Rickettsialpox are correct except
(B) Prophylactic doxycycline (A)The cause of the disease is R akari.
(C) Preventing tick bites by wearing protective clothing (B)Ticks of the genus Amblyomma are responsible for
(D) Delousing with insecticide transmission.
(C)The disease is mild.
10. One week aſter deer hunting in a wooded area, a 33- (D)Disease is more common in urban than rural areas.
year-old man developed fever to 39°C with headache
15. Reasons why C burnetii could be a potential agent of
and malaise. Over the subsequent 24 hours, he
bioterrorism include
developed nausea, vomiting, abdominal pain, and
(A)It is acquired by the inhalation.
diarrhea. On day 4, he developed a rash, initially around
(B)It is highly infectious.
the wrists and ankles, which then progressively evolved,
(C)It can be difficult to treat depending on the phase of
involving the arms, trunk, palms, and soles. Initially the
infection.
rash was macular, but it quickly evolved into
(D)Pneumonia may be severe.
maculopapules, some with central petechiae. Rocky
(E)All of the above.
Mountain spotted fever caused by Rickettsia rickettsii
was diagnosed. Which of the following statements about
Rocky Mountain spotted fever is correct?
(A)The vectors of Rickettsia rickettsii are ticks of the
genus Ixodes.
(B)A rash consistently appears by day 4 of illness.
(C)Rickettsia rickettsii forms inclusions in monocytes.
(D)The patient’s antibody response may not occur until a
ſter the second week of illness.
(E)The highest incidence of this disease is in the Rocky
Mountain states.

11. The recommended treatment for Q-fever


endocarditis is
(A) Emergent surgery; antibiotics are not effective
(B)Levofloxacin monotherapy for 6 weeks
(C)18 months of combination therapy with doxycycline
and hydroxychloroquine
(D)Penicillin and gentamicin combination therapy using
IgG titers to determine duration

12. Coxiella burnetii can be transmitted by milk when


animals such as goats and cows are infected. The
presently recommended conditions of “high-
temperature, short-time” pasteurization are adequate to
destroy viable Coxiella organisms.
(A) True
(B) False

13. The histopathological hallmark of infection caused by


Rickettsia rickettsiae is
(A)Morulae within granulocytes
Chapter XXVII: Chlamydia spp. (B)A Gram stain of her endocervical specimen would
show Chlamydia trachomatis inside polymorphonuclear
Review Questions cells.
1. Which of the following statements about chlamydial (C)The differential diagnosis includes infection with
antigens is correct? Neisseria gonorrhoeae, Chlamydia trachomatis, or both.
(A)Chlamydiae have shared group or genus-specific (D)The endocervical specimen should be analyzed for
antigens. herpes simplex.
(B)There is no cross-reaction between Chlamydia (E)Initial treatment is with ampicillin.
trachomatis
and Chlamydia pneumoniae antigens. 5. The following statements about trachoma are correct
(C)All five serovars of Chlamydia pneumoniae cross-react except
with Chlamydia psittaci. (A) It follows chronic or recurrent eye infection with
(D)One serovar of Chlamydia trachomatis causes eye Chlamydia trachomatis.
infections, and the second serovar causes genital (B)Millions of people worldwide have trachoma.
infections. (C)Trachoma is readily prevented by a chlamydial
vaccine.
2. The following are part of the control of Chlamydia (D)Progression of trachoma can be slowed by
psittaci and psittacosis in birds except intermittent treatment with azithromycin.
(A)Quarantine of psittacine birds imported into the (E)Trachoma involves scarring of the conjunctiva, eyelid
United States deformities, and eyelash injury to the cornea.
(B)Only allowing sale of psittacine birds hatched in the
United States 6. Elimination of blinding trachoma involves all of the
(C)Testing of birds for C psittaci infection following except
(D)Controlling the shipment of psittacine birds (A)Periodic administration of azithromycin
(E)Putting tetracycline in the feed of psittacine birds (B)Face washing and hygiene
(C)Periodic culture screening of conjunctiva swab
3. All of the following statements about perinatal specimens for Chlamydia trachomatis
Chlamydia trachomatis infections are correct except (D) Environment improvements to sewage systems to
(A)Between 15% and 40% of infants born to infected decrease the number of flies
women develop inclusion conjunctivitis. (E) Surgery on deformed eyelids
(B)Between 10% and 20% of infants born to infected
women develop infant pneumonia. 7. Which one of the following statements about
(C)The incubation period for Chlamydia trachomatis Chlamydia pneumoniae is most correct?
inclusion conjunctivitis is 1–2 days. (A)Transmission from person to person is by the airborne
(D)The incubation period for infant pneumonia is route.
typically 2–12 weeks. (B)It makes glycogen-rich inclusions that stain with
(E)Ocular prophylaxis with erythromycin or tetracycline iodine.
for neonatal Neisseria gonorrhoeae infection is generally (C)There are multiple serovars, including three that
not effective against neonatal Chlamydia trachomatis cause a systemic illness.
infection. (D)They are resistant to macrolides.
(F)Infant pneumonia caused by Chlamydia trachomatis (E)The reservoir is house cats.
often presents with a staccato cough.
8. The serovars of Chlamydia trachomatis generally can
4. An adolescent girl came to the clinic because of a new be divided into groups representing their clinical
and unusual vaginal discharge. She had recently become infections and anatomic site infected. Which of the
sexually active and had two new partners during the following statements about the C trachomatis serovars is
previous month. On pelvic examination, a purulent most correct?
discharge was seen at the opening of her endocervical (A)There is no immunologic cross-reaction between
canal. Which of the following statements about this Chlamydiatrachomatis serovars A, B, Ba, and D and the
patient is most correct? Chlamydiapneumoniae serovar.
(A)A serologic test for syphilis is not indicated because (B)Serovars L1, L2, and L3 are associated with
her symptoms are not those of syphilis. lymphogranu-loma venereum.
(C)The same Chlamydia trachomatis serovars are (C)Rectal swabs obtained from children 12 years of age
associated with blinding trachoma and sexually or younger
transmitted infections. (D)Urethral swab samples obtained from adult men
(D)The antibody titer rise seen beginning around 6–8 (E)Cervical swab samples obtained from adolescent girls
years follows infections with Chlamydia trachomatis
serovars D–K. 13. Chlamydia pneumoniae pneumonia most resembles
infection caused which of the following organisms?
9. In the United States, it has long been known that the (A)Streptococcus pneumoniae
positive seroprevalence for Chlamydia trachomatis (B)Mycoplasma pneumoniae
infection increases greatly during the primary school (C)Haemophilus influenzae
years (ages 6–10 years). A likely reason for this is (D)Chlamydia trachomatis
(A)Frequent adenovirus infections (E)Rhinovirus
(B)Increased incidence of infections with Chlamydia
trachomatis 14. Inclusion conjunctivitis of the newborn
(C)Cross-reactive antibodies with M protein of group A (A) Is a mucopurulent conjunctivitis that occurs 7–12
streptococci (Streptococcus pyogenes) days aſter delivery
(D)Children oſten have psittacosis (B) Is caused by Chlamydia psittaci
(E)Frequent infections with Chlamydia pneumoniae (C) Is a result of exposure to pet birds in the home
(D) Is treated with systemic penicillin because it may
10. All of the following statements about progress to pneumonia
lymphogranuloma vene-reum (LGV) are correct except (E) None of the above
(A)Chronic LGV proctitis can lead to rectal strictures and
fistula formation. 15. The diagnostic method of choice for Chlamydia
(B)The disease is more common in northern latitudes. trachomatis pneumonia in the newborn is
(C)There may be marked systemic symptoms, including (A)A nucleic acid amplification test that targets the ompA
fever, nausea, vomiting, headache, and meningismus. gene
(D)Chronic inflammation with LGV can lead to lymphatic (B)Culture of respiratory secretions in McCoy cells or
obstruction. other cell lines
(E)Inguinal lymph nodes may become enlarged and (C)Enzyme immunoassay testing of respiratory
matted, draining pus through the skin. secretions
(F)A few days or weeks aſter exposure, the disease (D)IgG antibodies detected by complement fixation
manifests itself as a genital papule or vesicle.

11. Which of the following methods are considered the


diagnostic tests of choice for urogenital infections
caused by Chlamydia trachomatis?
(A)Serology using complement fixation
(B)Cell culture using cycloheximide containing McCoy
cells
(C)Direct fluorescent antibody testing on urethral and
cervical specimens
(D)Nucleic acid amplification methods
(E)Enzyme immunoassays performed on genital tract
specimens

12. Nucleic acid amplification tests that are currently


available in the United States for diagnosing chlamydial
infections are approved for testing all of the following
specimens except
(A)Self-collected vaginal swabs in women
(B)First void urine samples obtained from men
Chapter XXVIII: Antimicrobial Chemotherapy in the past 6 weeks. Her chest radiograph shows bilateral
upper lobe infiltrates with cavities. Given the history and
Review Questions chest radiography findings, which of the following drug
1. The antimicrobial agent whose structure is shown regimens would be the best appropriate initial therapy
below is considered the drug of choice to treat infections while awaiting culture results?
caused by which one of the following microorganisms? (A)Isoniazid, rifampin, pyrazinamide, and ethambutol
(B)Penicillin G and rifampin
(C)Cefotaxime, clindamycin, and trimethoprim–
sulfamethoxazole
(D)Ampicillin–sulbactam (E)Vancomycin, gentamicin,
and clindamycin
(A)Bacteroides fragilis
(B)Pseudomonas aeruginosa 6. Aminoglycoside antibiotics typically cause which of the
(C)Herpes simplex virus following adverse events?
(D)Streptococcus pyogenes (group A streptococci) (A)They cause aplastic anemia.
(E)Mycobacterium tuberculosis (B)They cause nonspecific stimulation of B cells.
(C)They cause ototoxicity and nephrotoxicity.
2. Resistance of Staphylococcus aureus to the drug (D)They cause photosensitivity.
shown in Question 1 is caused by
(A)The action of acetyltransferase 7. Which one of the following groups of antimicrobial
(B)The action of β-lactamase agents acts on microorganisms by inhibiting protein
(C)Substitution of the d-Ala-d-Ala dipeptide with the d- synthesis?
Ala-d-Lac dipeptide in the cell wall peptidoglycan (A)Fluoroquinolones
(D)Decreased permeability of the bacterial cell wall to (B)Aminoglycosides
the drug (C)Penicillins
(E)Staphylococcus aureus being an intracellular (D)Glycopeptides (eg, vancomycin)
pathogen (E)Polymyxins

3. Streptococcus pneumoniae resistance to the drug 8. There are many bacterial–antimicrobial resistance
shown in Question 1 is caused by combinations. Which one of the following is of major
(A)The action of acetyltransferase international concern?
(B)The action of β-lactamase (A) Sulfonamide resistance in Neisseria meningitidis
(C)Substitution of the d-Ala-d-Ala dipeptide with d-Ala-d- (B) Penicillin G resistance in Neisseria gonorrhoeae
Lac dipeptide in the cell wall peptidoglycan (C) Ampicillin resistance in Haemophilus influenzae
(D)Decreased permeability of the bacterial cell wall (D) Erythromycin resistance in Streptococcus pyogenes
(E)Genetically modified binding proteins in the bacterial (group A streptococci)
cell wall (E) Vancomycin resistance in Staphylococcus aureus

4. All of the following statements about antimicrobial 9. Which of the following factors is not generally
resistance of enterococci are correct except considered when selecting initial antimicrobial therapy
(A)Enterococci are resistant to sulfamethoxazole– for an infection?
trimethoprim in vivo. (A)Age of the patient
(B)Cephalosporins are not active against enterococci. (B)Anatomic site of the infection (eg, meningitis or
(C)Resistance to the streptogramins (quinupristin– urinary tract infection)
dalfopristin) has emerged. (C)Whether or not the patient is immunocompromised
(D)Vancomycin-resistant enterococci are rare in Europe (D)Whether or not the patient has implanted devices in
and the United States. place (eg, artificial hip joint, artificial heart valve, urinary
(E)Vancomycin-resistant enterococci once consistently catheter)
clonal are now heterogeneous. (E)Waiting for culture and susceptibility test results

5. A 20-year-old Asian woman, a recent immigrant to the


United States, develops fever and a cough productive of
blood-streaked sputum. She has lost 6 kg of body weight
10. All of the following agents have good activity against Chapter XXIX: General Properties of Viruses
gram-positive organisms except Review Questions
(A)Daptomycin 1. Some viruses are characterized by helical symmetry of
(B)Vancomycin the viral nucleocapsid. Which of the following
(C)Aztreonam statements about viruses with helical symmetry is most
(D)Quinupristin–dalfopristin accurate?
(E)Tigecycline (A)All enveloped viruses with helical symmetry are
classified into the same virus family.
11. Tigecycline, a new glycylcycline antibiotic with good (B)Helical nucleocapsids are found primarily in DNA-
activity against a variety of pathogens, is best used for containing viruses.
treatment of which of the following infections? (C)All human viruses with helical nucleocapsids possess
(A)Meningitis an envelope.
(B)Intra-abdominal infections caused by mixed aerobic (D)Excess empty helical particles containing no nucleic
and anaerobic bacteria acid are commonly produced in infected cells.
(C)Neonatal sepsis
(D)Urethritis caused by Chlamydia trachomatis
2. Virus-infected cells oſten develop morphologic
(E)As monotherapy for bacteremia caused by
Acinetobacter baumannii changes referred to as cytopathic effects. Which of the
following statements about virus-induced cytopathic
12. Which of the following carbapenem antibiotics has changes is most accurate?
no activity against Pseudomonas aeruginosa? (A)They are pathognomonic for an infecting virus.
(B)They are rarely associated with cell death.
(A)Imipenem
(C)They may include giant cell formation.
(B)Meropenem
(D)They can only be seen with an electron microscope.
(C)Doripenem
(D)Ertapenem
3. Viruses usually initiate infection by first interacting
13. Which of the following agents would not be expected with receptors on the surface of cells. Which of the
to demonstrate postantibiotic affect against gram- following statements is most accurate about cellular
receptors for viruses?
negative bacilli?
(A)Imipenem (A)Cellular receptors for viruses have no known cellular
(B)Ciprofloxacin function.
(B)All viruses within a given family use the same cellular
(C)Gentamicin
receptor.
(D)Ampicillin
(C)All cells in a susceptible host express the viral
receptor.
14. All of the following are common mechanisms of
resistance to the penicillins except (D)Successful infection of a cell by a virus may involve
(A)Production of β-lactamases interaction with more than one type of receptor.
(B)Alterations in target receptors (PBPs)
(C)Inability to activate autolytic enzymes 4. Which of the following can be used to quantitate the
(D)Failure to synthesize peptidoglycans titer of infectious viruses?
(E)Methylation of ribosomal RNA (A)Plaque assay
(B)Electron microscopy
(C)Hemagglutination
15. The drug of first choice for the treatment of serious
(D)Polymerase chain reaction
anaerobic infections caused by Bacteroides fragilis is
(E)Enzyme immunoassay
(A)Clindamycin
(B)Ampicillin
(C)Cefoxitin 5. Which one of the following states a principle regarding
(D)Metronidazole viral nucleic acid?
(A) Viruses contain both RNA and DNA.
(E)Amoxicillin–clavulanate
(B) Some viruses contain a segmented genome.
(C) Purified viral nucleic acid from any virus is usually
infectious.
(D) Viral genome sizes are similar among known human
viruses.
6. Two mutants of poliovirus have been isolated, one (E)Helical nucleocapsids are found with single-stranded
(MutX) with a mutation in gene X and the second (MutY) DNA viruses.
with a mutation in gene Y. If cells are infected with each
mutant alone, no virus is produced. If a cell is coinfected 11. Many viruses can be grown in the laboratory. Which
with both MutX and MutY, which one of the following is of the following statements about virus propagation is
most likely to occur? not true?
(A)Reassortment of genome segments may occur and (A)Some viruses can be propagated in cell-free media.
give rise to a viable wild-type virus. (B)Some mammalian viruses can be cultivated in hen’s
(B)The genomes may be reverse transcribed to DNA and eggs.
both MutX and MutY viruses produced. (C)Some viruses with broad host ranges can multiply in
(C)Complementation between the mutant gene products many types of cells.
may occur and both MutX and MutY viruses produced. (D)Some human viruses can be grown in mice.
(D)The cells will transform at high frequency because (E)Most virus preparations have particle-to-infectious
they will not be killed by the poliovirus mutants. unit ratios greater than 1.

7. Which one of the following viruses possesses an RNA 12. Laboratory infections can be acquired when working
genome that is infectious when purified? with viruses unless good laboratory safety practices are
(A)Influenza virus followed. Which of the following is not a good biosafety
(B)Poliovirus practice?
(C)Papillomavirus (A)Use of biosafety hoods
(D)Measles virus (B)Use of laboratory coats and gloves
(E)Rotavirus (C)Avoidance of pipetting by mouth
(D)Flushing experimental waste down laboratory sink
8. Viruses belonging to which of the following groups are (E)Not eating or drinking in the laboratory
likely to establish latent infections?
(A)Poxviruses 13. Small viruses are in the same size range as which of
(B)Filoviruses the following?
(C)Herpesviruses (A)Staphylococcus species
(D)Influenza viruses (B)Serum globulin
(E)Caliciviruses (C)Red blood cells
(D)Eukaryotic ribosomes
9. Some viruses encode for a viral RNA-dependent RNA (E)Mitochondria
polymerase. Which of the following states a principle
about viral RNA polymerases? 14. Which of the following is not an important factor
(A)All RNA viruses carry RNA polymerase molecules contributing to the phenomenon of emerging viral
inside virus particles because they are needed to initiate diseases?
the next infectious cycle. (A)International air travel
(B)Antibodies against the viral RNA polymerase (B)Antibiotic resistance
neutralize virus infectivity. (C)Deforestation
(C)Negative-strand RNA viruses supply their own RNA- (D)War
dependent RNA polymerase because eukaryotic cells (E)Organ and tissue transplantation
lack such enzymes.
(D)The viral RNA polymerase protein also serves as a 15. Arboviruses are classified into several different virus
major core structural protein in the virus particle. families but are grouped together based on which of the
following common characteristics?
10. Which of the following statements regarding virus (A)Replicate only in humans
morphology is true? (B)Contain both RNA and DNA
(A)All RNA viruses are spherical in shape. (C)Are transmitted by vectors
(B)Some viruses contain flagella. (D)Cause hemorrhagic fevers
(C)Some viruses with DNA genomes contain a primitive (E)Cause encephalitis
nucleus.
(D)Viral surface proteins protect the viral genome from
nucleases.
Chapter XXX: Pathogenesis and Control of Viral inhibits viral reverse transcriptase and is active against
Diseases both HIV and hepatitis B virus. That drug is
(A)Acyclovir
Review Questions (B)Amantadine
1. Interferons are an important part of the host defense (C)Ribavirin
against viral infections. What is interferon’s principal (D)Saquinavir
mode of action? (E)Lamivudine
(A)It is present in the serum of healthy individuals, (F)Fuzeon
providing a viral surveillance role.
(B)It coats viral particles and blocks their attachment to 6. Regarding the HIV/AIDS patient in Question 5, a
cells. peptidomimetic agent that blocks virus-mediated
(C)It induces synthesis of one or more cellular proteins cleavage of viral structural protein precursors is chosen
that inhibit translation or transcription. as a second drug. That drug is
(D)It protects the virus-infected cell that produced it (A)Acyclovir
from cell death. (B)Amantadine
(C)Ribavirin
2. A 9-month-old girl is taken to the emergency room (D)Saquinavir
because of fever and persistent cough. Rales are heard in (E)Lamivudine
(F)Fuzeon
her leſt chest on physical examination. An infiltrate in her
leſt lung is seen on the chest radiograph. Pneumonia is 7. A 63-year-old woman is hospitalized for treatment of
diagnosed. Which of the following is the most likely leukemia. One day aſter admission she develops chills,
cause? fever, cough, headache, and myalgia. She states that her
(A)Rotavirus husband had a similar illness a few days earlier. There is
(B)Rhinovirus major concern about a respiratory virus outbreak in the
(C)Adenovirus staff of the chemotherapy ward and in the patients on
(D)Respiratory syncytial virus that ward. A synthetic amine that inhibits influenza A
(E)Coxsackievirus virus by blocking viral uncoating is chosen for
prophylactic treatment of the staff and patients. That
3. Which one of the following is a fundamental principle drug is
of viral disease causation? (A)Acyclovir
(A)One virus type induces a single disease syndrome. (B)Amantadine
(B)Many viral infections are subclinical and do not (C)Ribavirin
produce clinical disease. (D)Saquinavir
(C)The type of disease produced by a virus can be (E)Lamivudine
predicted by the morphology of that virus. (F)Fuzeon
(D)A particular disease syndrome has a single viral cause.
8. Which one of the following statements describes an
4. The skin is a formidable barrier to virus entry, but a advantage of killed-virus vaccines over attenuated live-
few viruses are able to breach this barrier and initiate virus vaccines?
infection of the host. Which of the following is an (A)Killed-virus vaccines induce a broader range of
example of a virus that enters through skin abrasions? immune responses than do attenuated live-virus
(A)Adenovirus vaccines.
(B)Rotavirus (B)Killed-virus vaccines more closely mimic natural
(C)Rhinovirus infections than do attenuated live-virus vaccines.
(D)Papilloma virus (C)Killed-virus vaccines pose no risk that vaccine virus
(E)Influenza virus might be transmitted to susceptible contacts.
(D)Killed-virus vaccines are efficacious against
5. A 40-year-old man has HIV/AIDS characterized by a respiratory virus infections because they induce good
low CD4 count and a high viral load. Highly active mucosal immunity.
antiretroviral therapy (HAART) will be initiated. One of
the drugs under consideration is a nucleoside analog that
9. What type of hepatitis B vaccine is currently in use in (D)Interferons induce the synthesis of a ribonuclease
the United States? that degrades viral mRNA.
(A)Synthetic peptide vaccine
(B)Killed-virus vaccine 15. Each of the following statements concerning viral
(C)Attenuated live-virus vaccine vaccines is correct except
(D)Subunit vaccine produced using recombinant DNA (A)In live attenuated vaccines, the virus has lost its ability
to cause disease but has retained its ability to induce
10. Which one of the following phrases accurately neutralizing antibody.
describes viral neutralizing antibodies? (B)In live attenuated vaccines, the possibility of reversion
(A)Directed against viral protein determinants on the to virulence is of concern.
outside of the virus particle (C)With inactivated vaccines, IgA mucosal immunity is
(B)Appear in the host sooner aſter viral infection than usually induced.
(D)With inactivated vaccines, protective immunity is
interferon
mainly caused by the production of IgG.
(C)Directed against viral nucleic acid sequences
(D)Induced only by disease-causing viruses
(E)Of little importance to immunity to viral infection

11. Many viruses use the respiratory tract as the route of


entry to initiate infections. Which of the following virus
groups does not?
(A)Adenovirus
(B)Coronavirus
(C)Hepadnavirus
(D)Paramyxovirus
(E)Poxvirus

12. Which of the following licensed virus vaccines is a


subunit vaccine prepared using recombinant DNA
technology?
(A)Measles–mumps–rubella
(B)Varicella
(C)Hepatitis A
(D)Papilloma
(E)Rotavirus
(F)Rabies

13. Which of the following viruses is the most common


cause of neonatal infections in the United States?
(A)Rubella
(B)Parvovirus B19
(C)Hepatitis B
(D)Cytomegalovirus
(E)Varicella (F)Human immunodeficiency virus

14. Which one of the following statements concerning


interferons is least accurate?
(A)Interferons are proteins that influence host defenses
in many ways, one of which is the induction of an
antiviral state.
(B)Interferons are synthesized only by virus-infected
cells.
(C)Interferons inhibit a broad range of viruses, not just
the virus that induced the interferon.
Chapter XXXI: Parvoviruses 6. Which one of the following best describes the
replication of human parvovirus B19?
Review Questions (A)Stimulates resting cells to proliferate
1. Which one of the following best describes a (B)Uses blood group antigen P as cellular receptor
physicochemical property of parvoviruses? (C)Readily establishes persistent infections
(A)Enveloped virus particle. (D)Entire replication cycle occurs in cytoplasm
(B)Single-stranded DNA genome. (E)Production of infectious progeny requires the
(C)Infectivity is inactivated by ether treatment. presence of a helper virus
(D)Virion exhibits helical symmetry.
(E)Virion is about the same size as herpesviruses. 7. Which one of the following statements is most
accurate concerning human infections by parvovirus
2. An 8-year-old child recently had erythema B19?
infectiosum. Her 33-year-old mother subsequently (A)Parvovirus B19 is transmitted readily by sexual
developed arthralgia followed by painful arthritis with intercourse.
swelling in the small joints of both hands. In addition to (B)Patients with disseminated disease caused by
the apparent tropism for joints, human parvovirus B19 is parvovirus B19 should be treated with acyclovir.
highly tropic for which cell type? (C)Parvovirus B19 does not cause any human disease.
(A) CD4 T lymphocytes (D)There is no vaccine for human parvovirus.
(B) Renal tubule cells
(C) Erythroid cells 8. Human bocavirus is a newly discovered parvovirus. It
(D) Glial cells has been detected most frequently in which type of
(E) Peyer patches sample?
(A)Urine
3. The 8-year-old child in Question 2 had an illness with (B)Cord blood
more than one phase. Which symptoms coincide with (C)Respiratory secretions
the second phase of the illness? (D)Fetal liver
(A)Sore throat (E)Bone marrow
(B)Skin rash
(C)Headache 9. Which of the following is available as a treatment or
(D)Diarrhea preventive for parvovirus B19 infections?
(E)Cough (A)Commercial immunoglobulin
(B)Vaccine containing recombinant VP2 viral antigen
4. A 42-year-old man with HIV/AIDS presented with (C)Bone marrow transplantation
aplastic anemia. Using the PCR, parvovirus B19 was (D)Antiviral drug that blocks virus–receptor interaction
detected in his serum. The patient presumably acquired
his parvovirus B19 infection from another person. The 10. Human erythroviruses and bocaviruses share the
most likely route of transmission is following properties except for which one?
(A)By contact with respiratory secretions or droplets (A)Small, nonenveloped virus particles.
(B)By contact with a skin rash (B)Difficult to culture.
(C)Trough sexual activity (C)Cause anemia.
(D)Trough a recent blood transfusion (D)Global distribution.
(E)No vaccine exists.
5. Which one of the following is a disease in which the
role of par-vovirus B19 has not been established?
(A)Erythema infectiosum (fiſth disease)
(B)Transient aplastic crisis
(C)Hydrops fetalis
(D)Fulminant hepatitis
Chapter XXXII: Adenoviruses (D)Keratoconjunctivitis
(E)Gastroenteritis
Review Questions (F)Hemorrhagic cystitis
In what follows, singular may be construed as plural (or
vice versa) as the sense dictates. 6. A 2½-year-old child attending nursery school acquires
1. What adenovirus protein or proteins regulate early a mild respiratory infection. Other children in the nursery
transcription of the viral genes and modulate the cell school have similar illnesses. Which adenovirus types are
cycle? the most likely causes of the illnesses?
(A)Fiber (A)Types 40 and 41
(B)Hexon (B)Types 8, 19, and 37
(C)Penton (C)Types 1, 2, 5, and 6
(D)Terminal protein (D)Types 3, 4, and 7
(E)E1 region protein (E)Types 21, 22, 34, and 35
(F)Cysteine proteinase
(G)E3 region protein 7. Which adenovirus types are frequent causes of acute
respiratory disease among military recruits?
2. What adenovirus protein serves as primer for initiation (A)Types 40 and 41
of viral DNA synthesis? (B)Types 8, 19, and 37
(A)Fiber (C)Types 1, 2, 5, and 6
(B)Hexon (D)Types 3, 4, and 7
(C)Penton (E)Types 21, 22, 34, and 35
(D)Terminal protein
(E)E1 region protein 8. Which of the following events led to reappearance of
(F)Cysteine proteinase acute respiratory disease outbreaks among U.S. military
(G)E3 region protein recruits in the late 1990s?
(A)Emergence of a new virulent strain of adenovirus
3. What adenovirus protein comprises the majority of (B)Cessation of adenovirus vaccination program for
capsomeres making up the virus capsid? recruits
(A)Fiber (C)Change in military housing and training conditions for
(B)Hexon recruits
(C)Penton (D)Cessation of adenovirus antiviral drug therapy
(D)Terminal protein program for recruits
(E)E1 region protein
(F)Cysteine proteinase 9. Your summer research project is to study the viruses
(G)E3 region protein that cause gastroenteritis. You recover a virus from a
stool sample and notice that the growth medium on the
4. A 3-month-old infant had watery diarrhea and fever infected cultures is highly acidic. You find that the viral
for 10 days. Rotavirus or adenovirus types 40 and 41 are genome is double-stranded DNA. Of the following, which
the suspected agents. What type of specimen would be one is the most appropriate conclusion you could draw?
most appropriate for detection of adenovirus types 40 (A)There is a high likelihood that the agent is a rotavirus.
and 41 infection in this patient? (B)You need to determine the viral serotype to establish
(A)Blood whether the virus was important in causing the disease.
(B)Urine (C)The patient should have been treated with the
(C)Conjunctival swab antiviral drug amantadine to shorten the duration of
(D)Stool symptoms.
(E)Troat swab (D)The virus particle would contain a reverse
(F)Cerebrospinal fluid transcriptase enzyme.

5. Which of the following human diseases has not been


associated with adenoviruses?
(A)Cancer
(B)Common colds
(C)Acute respiratory diseases
10. Which of the following groups of individuals is at the Chapter XXXIII: Herpesviruses
lowest risk of adenovirus disease?
(A)Healthy adults Review Questions
(B)Young children 1. A previously healthy 3-year-old boy develops a classic
(C)Bone marrow transplant recipients viral childhood illness. Which of the following primary
(D)Military recruits viral infections of childhood is usually symptomatic?
(E)AIDS patients (A)Cytomegalovirus
(B)Epstein-Barr virus
11. Adenoviruses can cause eye infections that are highly (C)Hepatitis B virus
contagious. Which of the following is least likely to be a (D)Varicella-zoster virus
means of transmission during an outbreak of epidemic (E)Parvovirus B19
keratoconjunctivitis?
(A)Swimming pools 2. Which one of the following is a recommended therapy
(B)Hand towels for herpes simplex virus genital infection?
(C)Mosquito bites (A)Acyclovir
(D)Hand-to-eye (B)Attenuated live virus vaccine
(E)Contaminated ophthalmic equipment (C)Herpes immune globulin
(D)Interferon-α
12. There are 57 known sero types of human (E)Ribavirin
adenoviruses. Which of the following statements is most
accurate? 3. Most herpesvirus infections are endemic worldwide.
(A)Types cannot be distinguished serologically. Which one of the following viruses shows marked
(B)All cause respiratory infections in children. geographic differences in seroprevalence?
(C)Most types replicate well in T lymphocytes. (A)Cytomegalovirus
(D)Two types can cause gastroenteritis. (B)Epstein-Barr virus
(C)Herpes simplex virus type 2
13. Each of the following statements concerning (D)Kaposi sarcoma herpesvirus
adenoviruses is correct except (E)Varicella-zoster virus
(A)Adenoviruses are composed of a double-stranded
DNA genome and a capsid without an envelope. 4. A 19-year-old female college student has a fever, sore
(B)Adenoviruses cause both sore throat and pneumonia. throat, and lymphadenopathy accompanied by
(C)Adenoviruses have only one serologic type. lymphocytosis with atypical cells and an increase in
(D)Adenoviruses are implicated as a cause of tumors in sheep cell agglutinins. Te diagnosis is most likely
animals but not humans. (A)Infectious hepatitis
(B)Infectious mononucleosis
14. Which of the following conditions is least likely to be (C)Chickenpox
caused by adenoviruses? (D)Herpes simplex infection
(A)Conjunctivitis (E)Viral meningitis
(B)Pneumonia
(C)Pharyngitis 5. A Tzanck smear of a scraping obtained from a vesicle
(D)Glomerulonephritis on the skin demonstrates multinucleated giant cells.
Multinucleated giant cells are associated with which of
the following viruses?
(A)Varicella-zoster
(B)Variola major
(C)Coxsackievirus
(D)Molluscum contagiosum

6. Which of the following statements about


betaherpesviruses is not true?
(A)They establish latent infections and persist
indefinitely in infected hosts.
(B)They are reactivated in immunocompromised 11. An outbreak of a rash called “mat herpes” occurred
patients. among high school students who had competed in a
(C)Most infections are subclinical. wrestling tournament. Which of the following
(D)They can infect lymphoid cells. statements is most accurate?
(E)They have short, cytolytic growth cycles in cultured (A)The rash is not contagious among wrestlers.
cells. (B)The causative agent is herpes simplex virus type 1.
(C)The causative agent is varicella-zoster virus.
7. A 28-year-old woman has recurrent genital herpes. (D)Lesions typically last 1 month or longer.
Which of the following statements about genital herpes (E)Students should be vaccinated before participating in
infections is true? wrestling tournaments.
(A) Reactivation of latent virus during pregnancy poses
no threat to the newborn. 12. The shingles vaccine is recommended for which of
(B) Virus cannot be transmitted in the absence of the following groups?
apparent lesions. (A)Healthy adolescents
(C) Recurrent episodes caused by reactivation of latent (B)Individuals older than age 60 years
virus tend to be more severe than the primary infection. (C)Pregnant women
(D) They can be caused by either herpes simplex virus (D)Those who never had chickenpox
type 1 or type 2.
(E) Latent herpes simplex virus can be found in dendritic 13. The most common congenital infection is caused by
cells. (A)Varicella-zoster virus
(B)Herpes simplex virus type 2
8. Which of the following viruses causes a (C)Human herpesvirus 8 (Kaposi sarcoma herpesvirus)
mononucleosis-like syndrome and is excreted in the (D)Cytomegalovirus
urine? (E)Parvovirus
(A)Cytomegalovirus
(B)Epstein-Barr virus 14. Which of the following groups are at increased risk
(C)Human herpesvirus 6 for herpes zoster?
(D)Varicella-zoster virus (A)Persons at advanced age
(E)Herpes simplex virus type 2 (B)Patients with atopic dermatitis
(C)Pregnant women
9. A 53-year-old woman develops fever and focal (D)Persons who have been vaccinated with varicella
neurologic signs. Magnetic resonance imaging shows a le vaccine
ſt temporal lobe lesion. Which of the following tests (E)Infants with congenital infections
would be most appropriate to confirm a diagnosis of
15. Which one of the following is the best explanation for
herpes simplex encephalitis in this patient?
the selective action of acyclovir (acycloguanosine) in
(A)Brain biopsy
herpes simplex virus (HSV)-infected cells?
(B)Tzanck smear
(A)Acyclovir binds specifically to viral receptors only on
(C)Polymerase chain reaction assay for viral DNA in
the surface of the HSV-infected cell.
cerebrospinal fluid
(B)Acyclovir is phosphorylated by a virus-encoded
(D)Serologic test for viral IgM antibody
phospho-kinase only within HSV-infected cells.
(C)Acyclovir selectively inhibits the RNA polymerase in
10. Which of the following tumors is caused by a virus
the HSV virion.
other than Epstein-Barr virus?
(D)Acyclovir specifically blocks the matrix protein of HSV,
(A)Posttransplant lymphomas
thereby preventing release of progeny HSV.
(B)Hodgkin disease
(C)Kaposi sarcoma
16. Each of the following statements concerning
(D)AIDS-related central nervous system non-Hodgkin
herpesvirus latency is correct except
lymphomas
(A)Exogenous stimuli can cause reactivation of latent
(E)Burkitt lymphoma
infection, with induction of symptomatic disease.
(B)During latency, antiviral antibody is not demonstrable
in the sera of infected individuals.
(C)Reactivation of latent herpesviruses is more common Chapter XXXIV: Poxviruses
in patients with impaired cell-mediated immunity than in
immunocompetent patients. Review Questions
(D)Virus can be recovered from latently infected cells by 1. A patient presents to the emergency room with
cocultivation with susceptible cells. vesicular lesions on both hands potentially resembling
smallpox. A public health investigation is begun to rule
17.Vaccines have been demonstrated to be efficacious in out smallpox. The patient is an immigrant working as a
preventing herpesvirus disease in which one of the shepherd in several states. What is the most likely cause
following situations? of his skin lesions?
(A)Herpes simplex virus type 1 primary infection (A)Vaccinia virus
(B)Herpes simplex virus type 2 reactivation (B)Variola virus
(C)Varicella-zoster reactivation (C)Monkeypox virus
(D)Cytomegalovirus primary infection (D)Tanapox virus
(E)Epstein-Barr virus reactivation (E)Orf virus

18. Herpes simplex virus and cytomegalovirus share 2. An emergency services worker is considering smallpox
many features. Which one of the following features is vaccination because of the potential for bioterrorism.
least likely to be shared? Which one of the following conditions is not a
(A) Important cause of morbidity and mortality in the contraindication for the use of vaccinia (smallpox)
newborn vaccine under routine nonemergency conditions?
(B) Congenital abnormalities caused by transplacental (A)Immunosuppression
passage (B)Severe allergy to a component of the vaccine
(C) Important cause of serious disease in (C)Household contact with a person with eczema
immunosuppressed individuals (D)Pregnancy
(D) Mild or inapparent infection (E)Previous smallpox vaccination

19. Herpes simplex virus type 1 (HSV-1) is distinct from 3. Which of the following poxviruses infects only
herpes simplex virus type 2 (HSV-2) in several different humans?
ways. Which one of the following is the least accurate (A)Monkeypox
statement? (B)Molluscum contagiosum
(A)HSV-1 causes lesions above the umbilicus more (C)Tanapox
frequently than HSV-2 does. (D)Cowpox
(B)Infection by HSV-1 is not associated with any tumors (E)Yaba tumor virus
in humans.
(C)Antiserum to HSV-1 neutralizes HSV-1 much more
4. A 7-year-old boy has pox-like lesions on his leſt hand
effectively than HSV-2.
(D)Whereas HSV-1 causes frequent recurrences, HSV-2 and arm. He has a pet rodent imported from West Africa.
infection rarely recurs. Monkeypox is diagnosed in the boy and the rodent.
Which of the following statements about monkeypox
20. Each of the following statements concerning Epstein- virus is most correct?
Barr virus is correct except (A)Clinical disease resembles smallpox.
(A)Many infections are mild or inapparent. (B)Human infections are never fatal.
(B)The earlier in life primary infection is acquired, the (C)Smallpox vaccination is not protective.
more likely the typical picture of infectious (D)Infections are readily transmitted among family
mononucleosis will be manifest. members.
(E)Virus particles can be distinguished from smallpox
(C)Latently infected lymphocytes regularly persist aſter virus by electron microscopy.
an acute episode of infection.
(D)Infection confers immunity against second episodes 5. Which of the following best describes the currently
of infectious mononucleosis. licensed smallpox vaccines?
(A)Live attenuated smallpox virus
(B)Inactivated smallpox virus
(C)Live vaccinia virus
(D)Inactivated vaccinia virus 11. Laboratory scientists who work with vaccinia virus–
(E)Reassortant vaccine containing both vaccinia and infected cultures or animals are at risk of accidental
smallpox viruses exposure to the virus. Which of the following procedures
by the laboratory worker is of least benefit in protecting
6. Which of the following does not apply to vaccinia virus against inadvertent infection with vaccinia virus?
replication in cultured cells? (A)Proper use of personal protective equipment such as
(A)Viral replication cycle takes place in the cytoplasm of gloves and goggles
infected cells. (B)Cleaning of laboratory work space before
(B)Te uncoating step leading to release of the viral experimentation
genome requires a newly synthesized viral protein. (C)Smallpox vaccination
(C)Early transcription of more than 50 viral genes occurs (D)Safe needle-handling practices
within viral cores and precedes viral DNA replication. (E)Use of biosafety hoods
(D)Newly formed virus particles mature by budding
through the nuclear membrane. 12. Vaccinia virus has all of the following attributes
except
7. Which feature of the variola virus makes it an extreme (A) Can cause severe localized or disseminated disease.
bioter-rorism threat? (B)Is a live, attenuated smallpox virus.
(A)Wide availability of the virus (C)Can induce immunity that lasts only a few years.
(B)Weaponized strains present in several laboratories (D)Has been in use for more than 200 years.
(C)Limited immunity in present population (E)Gene sequences coding for other viral proteins can be
(D)Low stockpiles of effective drugs for treatment inserted into its genome.
(E)Potential emergence from animal reservoir
13. The eradication of smallpox was facilitated by several
8. A patient presents with skin lesions similar in features of the virus. Which one of the following
appearance to molluscum contagiosum. How is diagnosis contributed least to eradication?
of this condition typically made? (A)It has one antigenic type.
(A)Viral culture (B)Inapparent infection is rare.
(B)Rapid antigen test (C)Administration of live vaccine reliably induces
(C)PCR for viral DNA immunity.
(D)Clinical appearance (D)It multiplies in the cytoplasm of infected cells.
(E)Inoculation of chorioallantoic membrane of chick
embryos 14. Vaccination with the vaccinia (smallpox) vaccine
protects against infections by the following poxviruses
9. Which of the following does not fulfill the criteria for except
exposure to vaccinia? (A)Molluscum contagiosum
(A)Smallpox vaccination (B)Variola
(B)Close contact with a recent smallpox vaccine (C)Cowpox
(C)Intrauterine exposure (D)Monkeypox
(D)Injection of vaccinia immune globulin

10. A researcher wishes to obtain a full-length genome of


variola virus for vaccine studies. Which of the following
is the appropriate source of the viral DNA?
(A)The Centers for Disease Control and Prevention.
(B)A World Health Organization collaborating center.
(C)The American Type Culture Collection.
(D)A colleague with a variola virus clone.
(E)Distribution of a full-length viral genome is prohibited.
Chapter XXXV: Hepatitis Viruses (B) A recombinant DNA vaccine
(C)A formalin-inactivated virus vaccine
Review Questions (D)An envelope glycoprotein subunit vaccine
1. A 24-year-old woman in New York City is admitted to (E)A chimeric poliovirus that expresses HAV neutralizing
the hospital because of jaundice. On workup, she is epitopes
found to have HCV infection. The major risk factor for
HCV infection in the United States is 6. The following statements about HCV infection and
(A)Tattoos associated chronic liver disease in the United States are
(B)Injecting drug use correct except
(C)Blood transfusion (A)HCV is responsible for 40% of chronic liver disease.
(D)Sexual activity (B)Chronic infection develops in most (70–90%) HCV-
(E)Working in health care occupations infected persons.
(C)HCV-associated liver disease is the major cause for
2. Which of the following exposures poses a risk for liver transplantation.
hepatitis infection? (D)HCV viremia occurs transiently during early stages of
(A)A nurse sustains a needlestick while drawing up infection.
insulin to administer to an HBV-infected patient with (E)HCV-infected patients are at high risk (5–20%) for liver
diabetes. cancer.
(B)While cleaning the bathroom, a housekeeper’s intact
skin has contact with feces. 7. A middle-aged man complained of acute onset of
(C)An operating room technician with chapped and fever, nausea, and pain in the right upper abdominal
quadrant. There was jaundice, and dark urine had been
abraded hands notices blood under his gloves aſter
observed several days earlier. A laboratory test was
assisting in an operation on a patient with HCV infection. positive for HAV IgM antibody. The physician can tell the
(D)A child drinks out of the same cup as her mother, who patient that
has an HAV infection. (A)He probably acquired the infection from a recent
(E)A shopper eats a sandwich prepared by a worker with blood transfusion.
an asymptomatic HBV infection. (B)He will probably develop chronic hepatitis.
(C)He will be at high risk of developing hepatocellular
3. An epidemic of jaundice caused by HEV occurred in carcinoma.
New Delhi. HEV is (D)He will be resistant to infection with hepatitis E.
(A)Found in rodents and pigs. (E)He may transmit the infection to family members by
(B)A major cause of bloodborne hepatitis. person-to-person spread for up to 2 weeks.
(C)The cause of a disease that resembles hepatitis C.
(D)Capable of establishing chronic infections. 8. Several different viruses can cause hepatitis. One of
(E)Associated with an increased risk of liver cancer. the following statements applies to all four viruses: HAV,
HCV, HDV, and HEV.
4. HDV (delta agent) is found only in patients who have (A)It contains a single-stranded RNA genome.
either acute or chronic infection with HBV. Which of the (B)It is transmitted primarily by the parenteral route.
following is most correct? (C)It is transmitted primarily by the fecal–oral route.
(A)HDV is a defective mutant of HBV. (D)It is associated with fulminant hepatitis.
(B)HDV depends on HBV surface antigen for virion (E)It undergoes sequence variation during chronic
formation. infection.
(C)HDV induces an immune response indistinguishable
from that induced by HBV. 9. A 30-year-old student goes to the emergency room
(D)HDV is related to HCV. because of fever and anorexia for the past 3 days. She
(E)HDV contains a circular DNA genome. appears jaundiced. Her liver is enlarged and tender. A
laboratory test shows elevated aminotransferases. She
5. A 23-year-old woman is planning a 1-year trip through reports a history of having received hepatitis B vaccine 2
Europe, Egypt, and the Indian subcontinent and receives years ago but has not had hepatitis A vaccine. The results
a vaccine for hepatitis A. The current hepatitis A vaccine of her hepatitis serologic tests are as follows: HAV IgM-
is negative, HAV IgG-positive, HBsAg-negative, HBsAb-
(A) A live attenuated virus vaccine
positive, HBcAb-negative, HCV Ab-positive. The most (E)Persons seeking treatment for a sexually transmitted
accurate conclusion is that she probably disease
(A)Has hepatitis A now, has not been infected with HBV,
and had hepatitis C in the past. 14. Which of the following statements regarding HBIG is
(B)Has hepatitis A now and has been infected with both not true?
HBV and HCV in the past. (A) HBIG provides temporary protection when
(C)Has been infected with HAV and HCV in the past and administered in standard doses.
has hepatitis B now. (B) HBIG typically is used instead of hepatitis B vaccine
(D)Has been infected with HAV in the past, has not been for postexposure immunoprophylaxis to prevent HBV
infected with HBV, and has hepatitis C now. infection.
(E)Has been infected with HAV and HCV in the past, has (C) No evidence exists that HBV, HCV, or HIV have ever
not been infected with HBV, and has hepatitis E now. been transmitted by HBIG in the United States.
(D) MHBIG is not used as protection against HCV
10. A 36-year-old nurse is found to be both HBsAg infection.
positive and HBeAg positive. The nurse most likely
(A) Has acute hepatitis and is infectious. 15. Each of the following statements concerning HAV is
(B) Has both HBV and HEV infections. correct except
(C)Has a chronic HBV infection. (A)The hepatitis A vaccine contains inactivated HAV as
(D)Has cleared a past HBV infection. the immunogen.
(E)Was previously immunized with HBV vaccine prepared (B)HAV commonly causes asymptomatic infection in
from healthy HBsAg-positive carriers. children.
(C)The diagnosis of hepatitis A is usually made by
11. The following persons are at increased risk for HAV isolating HAV in cell culture.
infection and should be routinely vaccinated except for (D)γ-Globulin is used to prevent hepatitis A in exposed
which group? persons.
(A)Persons traveling to or working in countries that have
high levels of HAV infection 16. Which of the following serologic patterns is
(B)Men who have sex with men suggestive of a patient with chronic hepatitis B with a
(C)Users of illegal drugs (both injecting and noninjecting) pre-core mutation?
(D)Persons who have an occupational risk for infection (A)HBsAg positive, HBsAbnegative, anti-HBc positive,
(E)Persons who have a clotting factor disorder HBeAg positive, HBV DNApositive
(F)Susceptible persons who have chronic liver disease (B)HBsAg positive, HBsAbnegative, anti-HBc positive,
(G)Teachers in elementary schools HBeAg positive, HBV DNApositive
(C)HBsAg positive, HBsAb positive, anti-HBc positive,
12. There is global variation in the prevalence of HBV HBeAg negative, HBV DNA positive
infection. Which of the following geographic areas has (D)HBsAg negative, HBsAb positive, anti-HBc positive,
low endemicity (HBsAg prevalence of < 2%)? HBeAg negative, HBV DNA negative
(A)Southeast Asia
(B)The Pacific Islands 17. A 35-year-old man addicted to intravenous drugs has
(C)Eastern Europe been a carrier of HBsAg for 10 years. He suddenly
(D)Australia develops acute fulminant hepatitis and dies within 10
(E)Sub-Saharan Africa days. Which of the following laboratory tests would
contribute most to diagnosis?
13. Which of the following persons are not (A)Anti-HBs antibody
recommended to receive hepatitis B vaccine because (B)HBeAg
they have a risk factor for HBV infection? (C)Anti-HBc antibody
(A)Sexually active persons who are not in long-term, (D)Anti-delta virus antibody
mutually monogamous relationships
(B)Injection drug users
(C)Pregnant women
(D)Persons who live in a household with a person who is
HBsAg positive
18. Each of the following statements concerning HCV and Chapter XXXVI: Picornaviruses (Enterovirus and
HDV is correct except Rhinovirus Groups)
(A)HCV is an RNA virus.
(B)HDV is transmitted primarily by the fecal–oral route. Review Questions
(C)HDV is a defective virus that can replicate only in a cell 1. Which of the following statements about rhinoviruses
that is also infected with HBV. is correct?
(D)People infected with HCV commonly become chronic (A) There are three antigenic types.
carriers of HCV and are predisposed to hepatocellular (B) Amantadine protects against infection.
carcinoma. (C) They do not survive on environmental surfaces.
(D) They are the most frequent causative agent of the
19. Which of the following statements about HBV is common cold.
false? (E) They share physicochemical similarities with
(A)Replication involves reverse transcriptase. coronaviruses.
(B)Infected persons may have large numbers of
noninfectious viral particles circulating in their 2. A 26-year-old man develops myopericarditis with mild
bloodstream. congestive heart failure that increases over several
(C)Infection can result in cirrhosis. weeks. Coxsackie-virus B5 infection is diagnosed. Which
(D)Asymptomatic infections can last for years. of the following clinical syndromes is not associated with
(E)In the United States, the incidence of infection has coxsackievirus infections?
been steadily increasing over the past few years. (A)Herpangina
(B)Myocarditis or pericarditis
20. Treatment of hepatitis C can involve drugs of which (C)Aseptic meningitis
of the following classes? (D)Acute hemorrhagic conjunctivitis
(A)Protease inhibitors, polymerase inhibitors, and (E)Progressive postpolio muscle atrophy
interleukins
(B)Non-nucleoside polymerase inhibitors, protease 3. A 3-month-old child develops fever, restlessness, and
inhibitors, and interferons unusual crying. These are followed by apparent lethargy.
(C)Transcription inhibitors and interferons Physical examination shows a normal-appearing infant
(D)Protease inhibitors, polymerase inhibitors, and who is minimally responsive to stimuli. A lumbar
interferons puncture yields cerebrospinal fluid with 200 white blood
(E)Reverse transcriptase inhibitors and interferons cells per microliter, predominantly lymphocytes. Acute
aseptic meningitis is diagnosed, probably caused by an
enterovirus. Enteroviruses are characterized by
(A)Latency in sensory ganglia and reactivation primarily
in immunocompromised patients
(B)Transmission primarily by the fecal–oral route
(C)The presence of a DNA polymerase enzyme
(D)The entry of cells following binding to the intercellular
adhesion molecule-1 (ICAM-1) receptor
(E)Undergoing antigenic shiſt and driſt

4. Picornavirus vaccines have been used for several


decades in the prevention of human disease. Which of
the following statements is correct?
(A)The live, attenuated poliovirus vaccine produces
gastrointestinal tract resistance.
(B)There is an effective killed vaccine against the three
major types of rhinoviruses.
(C)The live, attenuated poliovirus vaccine induces
protective immunity against the closely related coxsackie
B viruses.
(D)None of the available echovirus vaccines should be 9. Outbreaks of hand-foot-and-mouth disease,
given to immunocompromised patients. characterized by oral ulcerations and vesicular rashes,
(E)Only the live attenuated poliovirus vaccine is currently occur and may result in infant deaths. The disease is
recommended for use in the United States. caused by
(A)Foot-and-mouth disease virus
5. One month aſter school has been let out for the (B)Chickenpox virus
(C)Nonpolio enteroviruses
summer, a 16-year-old girl develops fever, myalgia, and
(D)Rhinoviruses
headache. An outbreak of an illness with similar
(E)Rubella virus
symptoms caused by an echo-virus is known to be
occurring in the community. The primary anatomic site
10. Epidemiologic studies indicate that a core group of
of echovirus multiplication in the human host is
enteroviruses is consistently circulating in the United
(A)The muscular system
States. Which of the following statements is most
(B)The central nervous system
accurate?
(C)The alimentary tract
(A)Members of the core group all display an epidemic
(D)The blood and lymph system
pattern of outbreaks of disease.
(E)The respiratory system
(B)The group includes about half of the known
enteroviruses.
6. Which of the following properties of enteroviruses is
(C)Disease occurs predominantly in adolescents and
not shared by rhinoviruses?
adults.
(A) Single-stranded RNA genome
(D)Members of the group are all classified as coxsackie A
(B) Production by cleavage of viral proteins from a
and B viruses.
polyprotein precursor
(E)Tis core group determines the majority of enterovirus
(C) Resistance to lipid solvents
disease.
(D) Stability at acid pH (pH 3.0)
(E) Icosahedral symmetry
11. Each of the following statements concerning
rhinoviruses is correct except
7. A person with asthma has an acute exacerbation with
(A)Rhinoviruses are one of the most frequent causes of
increased lower respiratory illness. A virus is recovered.
the common cold.
The isolate is most likely to be which of the following
(B)Rhinoviruses grow better at 33°C than at 37°C; thus,
virus types?
they tend to cause disease in the upper respiratory tract
(A)Parainfluenza virus
rather than the lower respiratory tract.
(B)Parechovirus
(C)Rhinoviruses are members of the picornavirus family
(C)Rhinovirus
and resemble poliovirus in their structure and
(D)Respiratory syncytial virus
replication.
(E)Echovirus
(D)The immunity provided by the rhinovirus vaccine is
excellent because there is only one serotype.
8. The use of live oral polio vaccine has been replaced by
inactivated polio vaccine in many countries. Which of the
12. Live, attenuated oral polio vaccine (OPV) and
following is the primary reason?
inactivated polio vaccine (IPV) are both available. In
(A)It is more cost effective to use the inactivated vaccine.
which one of the following situations is the use of OPV
(B)There is a greater risk of vaccine-induced disease than
preferred?
wild virus–induced disease in areas where poliovirus has
(A)Routine infant vaccination
been eradicated.
(B)Mass immunization programs in areas of high
(C)Only a single dose of inactivated vaccine is necessary
poliomyelitis endemicity
compared with multiple doses of the oral vaccine.
(C)Adult immunization
(D)Circulating poliovirus strains have changed and the
(D)Patients who are receiving immunosuppressive
live vaccine is no longer effective in many countries.
therapy
(E)Family contacts of immunocompromised patients
13. Which of the following statements about enteroviral Chapter XXXVII: Reoviruses, Rotaviruses and
meningitis is true? Caliciviruses
(A)Vaccines are generally available to protect against the
disease. Review Questions
(B)The main symptom is muscle paralysis. 1. A 36-year-old man enjoyed a meal of raw oysters.
(C)Transmission is usually by the fecal–oral route. Twenty-four hours later, he became ill, with a sudden
(D)The causative agents do not survive well in the onset of vomiting, diarrhea, and headache. The most
environment. likely cause of his gastroenteritis is
(E)Recovery is rarely complete. (A)Astrovirus
(B)Hepatitis A virus
14. The major barrier to the control of rhinovirus upper (C)Norwalk virus
respiratory infections by immunization is (D)Rotavirus, group A
(A)The poor local and systemic immune response to (E)Echovirus
these viruses
(B)The large number of rhinovirus serotypes 2. This virus is the most important cause of
(C)The side effects of the vaccine gastroenteritis in infants and young children. It causes
(D)The inability to grow the viruses in cell culture
infections that are oſten severe and may be life
15. Each of the following clinical syndromes is associated threatening, especially in infants.
with infection by picornaviruses except (A)Echovirus
(A)Myocarditis or pericarditis (B)Norwalk virus
(B)Hepatitis (C)Rotavirus, group A
(C)Mononucleosis (D)Orbivirus
(D)Meningitis (E)Parvovirus

3. An outbreak of epidemic gastroenteritis occurred at a


wooded summer camp 24 hours aſter a party for visiting
families. Some of the visiting parents also became ill.
Samples taken 2 weeks later from the well that was the
source of drinking water at the camp were negative for
fecal coliforms. The most likely source of the outbreak
was
(A)Mosquitoes or ticks, present in high numbers in the
area
(B)Contaminated food served at the party
(C)A nearby stream used for fishing
(D)A visiting parent who was developing pneumonia
(E)The swimming pool

4. This viral gastroenteritis agent has a segmented,


double-stranded RNA genome and a double-shelled
capsid. It is a member of which virus family?
(A)Adenoviridae
(B)Astroviridae
(C)Caliciviridae
(D)Reoviridae
(E)Coronaviridae

5. Rotavirus and Norwalk virus are distinctly different


viruses. However, they share which one of the following
characteristics?
(A)Fecal–oral mode of transmission.
(B)They mainly cause disease in infants and young 10. Norwalk virus illness might be prevented by any of
children. the following except
(C)They induce generally mild disease in young children. (A)Avoidance of raw fruits
(D)Infection patterns show no seasonal variation. (B)Live, reassortant vaccine
(E)A double-stranded RNA genome (C)Careful handwashing
(D)Avoidance of local drinking water
6. Because rotavirus infections can be serious, a vaccine (E)Avoidance of raw oysters
would be beneficial. Which of the following is most
correct regarding a rotavirus vaccine? 11. Which of the following statements about noroviruses
(A)A killed human rotavirus group A vaccine is licensed is false?
for use in the United States. (A) They cause almost half the cases of viral
(B)Live attenuated vaccines are licensed for use in the gastroenteritis in the United States.
United States. (B)They can be responsible for epidemics of
(C)Vaccine development is complicated by rapid gastroenteritis.
antigenic variation by the virus. (C)They generally produce an illness lasting 1–2 weeks.
(D)Available antiviral drugs make a vaccine unnecessary. (D)Similar viruses are widespread among marine
(E)Vaccine development is complicated because the animals.
virus cannot be grown in cell culture. (E)They typically cause disease in children and adults
rather than infants.
7. Rotaviruses and astroviruses share a number of
characteristics. Which of the following is not shared? 12. Each of the following statements regarding
(A)Multiple serotypes exist rotaviruses is correct except
(B)Can cause gastroenteritis in infants and children (A)The rotavirus vaccine contains recombinant RNA
(C)Can cause gastroenteritis in elderly institutionalized polymerase as the immunogen.
patients (B)Rotaviruses are a leading cause of diarrhea in young
(D)Live vaccine available children.
(E)Fecal–oral route of transmission (C)Rotaviruses are transmitted primarily by the fecal–
oral route.
8. A 20-year-old man was on a 3-week tour of Italy with (D)Rotaviruses belong to the reovirus family, which has a
other college students. One day he abruptly became ill double-stranded segmented RNA genome.
with nausea and vomiting followed 5 hours later by
abdominal cramps and watery diarrhea. No fever was
noted. Which of these viruses is the most likely cause of
the man’s illness?
(A) Calicivirus
(B) Rotavirus
(C)Reovirus
(D)Adenovirus
(E)Astrovirus

9. Which statement about rotaviral gastroenteritis is


false?
(A) The name of the causative agent was suggested by its
appearance.
(B)Most of the estimated 600,000 deaths occurring
worldwide from this disease are from dehydration.
(C)Most cases of the disease occur in infants and
children.
(D)The causative agent infects mainly the stomach.
(E)The disease is transmitted by the fecal–oral route.
Chapter XXXVIII: Arthropod-Borne and Rodent-Borne 5. A 27-year-old man develops fever, chills, headache,
Viral Diseases and backache. Four days later he develops a high fever
and jaundice. Yellow fever is diagnosed. Which of the
Review Questions following statements concerning yellow fever is correct?
1. A 74-year-old man develops fever, malaise, and a sore (A)The virus is transmitted by culicine mosquitoes in the
urban form of disease.
throat followed shortly thereaſter by nausea, vomiting, (B)Monkeys in the jungle are a major reservoir of yellow
and then stupor. Eastern equine encephalitis is fever virus.
diagnosed. Control of this disease in humans could be
(C)Yellow fever oſten has long-term complications.
accomplished by eradication of which of the following?
(A)Horses (D)All infections lead to apparent disease.
(B)Birds (E)Ribavirin is specific therapy.
(C)Sandflies
(D)Mosquitoes 6. Regarding the patient in Question 5, yellow fever
(E)Ticks occurs in which region or regions of the world?
(A) Asia
2. An arbovirus common in the Middle East, Africa, and (B) Africa and South America
Southwest Asia first appeared in New York in 1999. By (C)North America
2002 the virus had spread throughout the continental (D)Africa and Middle East
United States. This arbovirus, a member of the Japanese (E)Throughout the world
B encephalitis antigenic complex, is which of the
following? 7. African hemorrhagic fevers, Marburg and Ebola, are
(A)Japanese B encephalitis virus severe diseases oſten ending in death. Which of the
(B)Tick-borne encephalitis virus following is most accurate about Ebola virus?
(C)West Nile virus (A)It is spread by contact with blood or other body fluids.
(D)Dengue virus (B)It is transmitted by mosquitoes.
(E)Riſt Valley fever virus (C)It is a flavivirus.
(D)It causes infections but no disease in nonhuman
3. Which of the following descriptions of or statements primates.
about Lassa fever is correct? (E)It is antigenically related to Lassa fever virus.
(A)It is found in eastern Africa.
(B)Human-to-human transmission does not occur. 8. Which of the following groups can be vaccinated
(C)It seldom causes death or complications. routinely with yellow fever vaccine without special safety
(D)It occurs from contact with the house rat Mastomys considerations?
natalensis. (A)Children younger than 9 months old
(E)There is no drug that is effective in treating Lassa (B)Pregnant women
fever. (C)Persons with compromised immune systems
(D)All of the above
4. Arboviruses are transmitted by bloodsucking (E)None of the above
arthropods from one vertebrate host to another.
Arboviruses are found in the following virus families 9. Hantaviruses, which are emerging pathogens in the
except which of the following? United States, can be described by which of the
(A)Togaviridae following?
(B)Flaviviridae (A)They are arenaviruses.
(C)Bunyaviridae (B)They are readily transmitted human to human.
(D)Reoviridae (C)They cause influenza-like symptoms followed rapidly
(E)Arenaviridae by acute respiratory failure.
(D)They are acquired by inhalation of aerosols of deer
urine.
(E)They show a high frequency of antigenic variation.
10. A microbiologist was performing a necropsy in a 13. Each of the following statements concerning
laminar flow biosafety cabinet on a blue-jay submitted as arboviruses is correct except
part of a state’s arbovirus surveillance program. He (A)The pathogenesis of dengue hemorrhagic shock
lacerated his thumb while using a scalpel to remove the syndrome is associated with the heterotypic anamnestic
bird’s brain. Four days later, he developed a headache, response.
myalgia, and malaise followed by chills, sweats, and (B)Wild birds are the reservoir for encephalitis viruses
lymph node swelling. Two days later, a rash began on his but not for yellow fever virus.
face and spread to the trunk, arms, and legs, persisting (C)Ticks are the main mode of transmission for both
for about 3 days. He sought medical care and reported a encephalitis viruses and yellow fever virus.
history of dengue fever and vaccinations with yellow (D)There is a live, attenuated vaccine that effectively
fever and Japanese B encephalitis vaccines. A serum prevents yellow fever.
sample taken the day of the injury contained anti-
flavivirus immunoglobulin G (IgG) antibody by enzyme- 14. Which of the following statements about yellow fever
linked immunosorbent assay. A serum sample collected is false?
13 days aſter the onset of illness showed an increased (A) There is no animal reservoir.
(B) The name “yellow” comes from the fact that many
titer of antiflavivirus IgG antibody and the presence of
victims have jaundice.
West Nile virus IgM antibody. The physician could
(C) Certain mosquitoes are biological hosts for the
conclude that the most likely cause of the
causative agent.
microbiologist’s illness was which virus?
(D) Outbreaks of the disease could occur in the United
(A)Dengue virus
States because a suitable vector is present.
(B)Yellow fever virus
(E) An attenuated vaccine is widely used to prevent the
(C)West Nile virus
disease.
(D)St. Louis encephalitis
(E)Not identifiable until neutralizing antibody titers from
15. Which of the following statements about
paired sera could be assessed against a panel of
hantaviruses in the United States is correct?
arboviruses
(A)They are limited in range to southwestern states.
(B)They are carried only by deer mice.
11. Which of the following statements about dengue
(C)They infect human beings with a fatality rate that can
virus is not true?
be above 30%.
(A)It is the most important mosquito-borne viral disease
(D)They were first identified in the early 1970s.
affecting humans.
(E)They are contracted mainly in bat caves.
(B)It is distributed worldwide in tropical regions.
(C)It can cause a severe hemorrhagic fever.
(D)There is a single antigenic type.
(E)One form of disease is characterized by increased
vascular permeability.

12. Which of the following diseases occurring in the


United States lacks a known insect vector?
(A)Hantavirus pulmonary syndrome
(B)West Nile fever
(C)La Crosse encephalitis
(D)Colorado tick fever
(E)St. Louis encephalitis
Chapter XXXIX: Orthomyxoviruses (Influenza Viruses) (F)Lipid in the viral envelope

Review Questions 6. A 70-year-old nursing home patient refused the


1. Which of the following statements regarding the influenza vaccine and subsequently developed influenza.
prevention and treatment of influenza is correct? She died of acute pneumonia 1 week aſter contracting
(A)Booster doses of vaccine are not recommended. the flu. The most common cause of acute postinfluenza
(B)Drugs that inhibit neuraminidase are active only pneumonia is which of the following?
against influenza A. (A)Legionella
(C)As with some other live vaccines, the attenuated (B)Staphylococcus aureus
influenza vaccine should not be given to pregnant (C)Measles
women. (D)Cytomegalovirus
(D)The influenza vaccine contains several serotypes of (E)Listeria
virus.
(E)The virus strains in the influenza vaccine do not vary 7. Which of the following statements concerning
from year to year.
antigenic driſt in influenza viruses is correct?
2. Which of the following statements about the (A) It results in major antigenic changes.
neuraminidase of influenza virus is not correct? (B) It is exhibited only by influenza A viruses.
(A)Is embedded in the outer surface of the viral envelope (C)It is caused by frameshiſt mutations in viral genes.
(B)Forms a spike structure composed of four identical (D)It results in new subtypes over time.
monomers, each with enzyme activity (E)It affects predominantly the matrix protein.
(C)Facilitates release of virus particles from infected cells
(D)Lowers the viscosity of the mucous film in the 8. A 32-year-old male physician developed a “flulike”
respiratory tract syndrome with fever, sore throat, headache, and
(E)Is antigenically similar among all mammalian influenza myalgia. To provide laboratory confirmation of influenza,
viruses a culture for the virus was ordered. Which of the
following would be the best specimen for isolating the
3. Which of the following statements reflects the virus responsible for this infection?
pathogenesis of influenza? (A)Stool
(A)The virus enters the host in airborne droplets. (B)Nasopharyngeal swab
(B)Viremia is common. (C)Vesicle fluid
(C)The virus frequently establishes persistent infections (D)Blood
in the lung. (E)Saliva
(D)Pneumonia is not associated with secondary bacterial
infections. 9. Which of the following statements about isolation of
(E)Viral infection does not kill cells in the respiratory influenza viruses is correct?
tract. (A)Diagnosis of an influenza virus infection can only be
made by isolating the virus.
4. Which of the following symptoms is not typical of (B)Isolation of influenza virus is done using newborn
influenza? mice.
(A) Fever (C)Isolation of virus can help determine the
(B) Muscular aches epidemiology of the disease.
(C) Malaise (D)Primary influenza virus isolates grow readily in cell
(D) Dry cough culture.
(E) Rash
10. The principal reservoir for the antigenic shiſt variants
5. The type-specific antigen (A, B, or C) of influenza
of influenza virus appears to be which of the following?
viruses is found on which viral constituent?
(A)Chronic human carriers of the virus
(A)Hemagglutinin
(B)Sewage
(B)Neuraminidase
(C)Pigs, horses, and fowl
(C)Nucleocapsid
(D)Mosquitoes
(D)Polymerase complex
(E)Rodents
(E)Major nonstructural protein
11. Highly pathogenic H5N1 avian influenza (HPAI) can (C)The worldwide epidemics caused by influenza A virus
infect humans with a high mortality rate, but it has not are caused by antigenic shiſts.
yet resulted in a pandemic. The following are
characteristics of HPAI, except for one. Which one is not? (D)The protein involved in antigenic driſt is primarily the
(A)Efficient human-to-human transmission internal ribonucleoprotein.
(B)Presence of avian influenza genes
(C)Efficient infection of domestic poultry 16. Which of the following infectious agents is most likely
(D)Contains segmented RNA genome to cause a pandemic?
(A)Influenza A virus
12. Which of the following statements about diagnostic (B)Streptococcus pyogenes
testing for influenza is true? (C)Influenza B virus
(A)Clinical symptoms reliably distinguish influenza from (D)Respiratory syncytial virus
other respiratory illnesses. (E)Influenza C virus
(B)Viral culture is the “gold standard” diagnostic test
because it is the most rapid assay.
(C)Patient antibody responses are highly specific for the
strain of infecting influenza virus.
(D)Reverse transcription polymerase chain reaction is
preferred for its speed, sensitivity, and specificity.

13. The mechanism of “antigenic driſt” in influenza


viruses includes all but one of the following
(A)Can involve either hemagglutinin or neuraminidase
antigens
(B)Mutations caused by viral RNA polymerase
(C)Can predominate under selective host population
immune pressures
(D)Reassortment between human and animal or avian
reservoirs
(E)Can involve genes encoding structural or
nonstructural proteins

14. Each of the following statements concerning the


prevention and treatment of influenza is correct except
(A)The inactivated influenza vaccine contains H1N1 virus
but the live, attenuated influenza vaccine contains H3N2
virus.
(B)The vaccine is recommended to be given each year
because the antigenicity of the virus driſts.
(C)Oseltamivir is effective against both influenza A and
influenza B viruses.
(D)The main antigen in the vaccine that induces
protective antibody is the hemagglutinin.

15. Each of the following statements concerning the


antigenicity of influenza A virus is correct except
(A)Antigenic shiſts, which represent major changes in
antigenicity, occur infrequently and are caused by the
reassortment of segments of the viral genome.
(B)Antigenic shiſts affect both the hemagglutinin and the
neuraminidase.
Chapter XL: Paramyxoviruses and Rubella Virus (B) Congenital abnormalities occur when a nonimmune
pregnant woman is infected at any time during
Review Questions pregnancy.
1. A 4-year-old boy develops an acute febrile illness. His (C) Deafness is a common defect associated with
pediatrician diagnoses mumps. The organ most congenital rubella syndrome.
commonly exhibiting signs of mumps is the (D) Only rare strains of rubella virus are teratogenic.
(A)Lungs (E) None of the above
(B)Ovary
(C)Parotid glands 6. A 5-year-old child develops a low-grade fever, coryza,
(D)Skin conjunctivitis, and Koplik spots. The physician can
(E)Testes conclude that
(A)The child has probably not been successfully
2. The paramyxoviruses include the most important vaccinated with the MMR vaccine.
causes of respiratory infections in infants and young (B)The child’s pregnant mother is at risk of becoming
children. Which of the following is not characteristic of infected and her unborn child developing congenital
paramyxoviruses? abnormalities, including mental retardation.
(A)Genome is negative-sense RNA. (C)A rash will soon develop on the child’s face and will
(B)Envelope contains a glycoprotein with fusion activity. last only 2–3 days.
(C)Paramyxoviruses do not undergo genetic (D)Treatment of the child with the antiviral drug ribavirin
reassortment. should be initiated immediately to minimize the chance
(D)Replication cycle occurs in cytoplasm of susceptible of development of acute encephalitis.
cells.
(E)Genome is segmented. 7. Parainfluenza viruses are ubiquitous and cause
respiratory illnesses in people of all ages. However,
3. A 2-month-old infant developed a respiratory illness reinfections with parain-fluenza viruses are common
that the pediatrician diagnosed as bronchiolitis. The because
most likely cause of the disease is (A)Many antigenic types of parainfluenza viruses exist,
(A)Parainfluenza virus type 4 and exposure to new strains results in new infections.
(B)Respiratory syncytial virus (B)Infections in the respiratory tract do not elicit a
(C)Influenza virus systemic immune response.
(D)Metapneumovirus (C)Limited virus replication occurs, which fails to
(E)Measles virus stimulate antibody production.
(D)Secretory immunoglobulin A antibody in the nose is
4. Several paramyxoviruses can cause pneumonia in short lived, disappearing a few months aſter infection.
infants or children. For which of the following
paramyxoviruses is there an effective vaccine available 8. A 20-month-old boy had an illness characterized by
that would prevent pneumonia? fever, irritability, conjunctivitis, and a brick-red rash
(A)Parainfluenza virus type 1 initially on the face but spreading downward and
(B)Measles virus outward. At age 9 years, the boy had a gradual onset of
(C)Respiratory syncytial virus severe, generalized neurologic deterioration. Subacute
(D)Mumps virus sclerosing panencephalitis (SSPE) was diagnosed. Which
(E)Metapneumovirus of the following statements about SSPE is correct?
(A)Defective varicella-zoster virus is present in brain
5. A 27-year-old woman who is 2 months’ pregnant cells.
develops fever, malaise, and arthralgia. A fine (B)High titers of measles antibody are found in
maculopapular rash appears on her face, trunk, and cerebrospinal fluid.
extremities. Rubella is diagnosed, and there is concern (C)The incidence of the disease is rising since the
that the fetus will be infected, resulting in the congenital introduction of MMR vaccine.
rubella syndrome. Which of the following statements (D)Rapidly progressive deterioration of brain function
about this syndrome is correct? occurs.
(A) The disease can be prevented by vaccination of (E)The disease is a rare, late complication of rubella
school-age children with measles vaccine. infection.
9. Which of the following paramyxoviruses has an HN 14. Each of the following statements concerning rubella
surface glycoprotein lacking hemagglutinin activity? vaccine is correct except
(A)Measles virus (A)The vaccine prevents reinfection, thereby limiting the
(B)Mumps virus spread of virulent virus.
(C)Parainfluenza virus type 1 (B)The immunogen in the vaccine is killed rubella virus.
(D)Respiratory syncytial virus (C)The vaccine induces antibodies that prevent
(E)Rubella virus dissemination of the virus by neutralizing it during the
viremic stage.
10. A 3-year-old girl develops an acute respiratory virus (D)The incidence of both childhood rubella and
infection that requires hospitalization. Ribavirin therapy congenital rubella syndrome has decreased significantly
is considered. Ribavirin is approved for treatment of since the advent of the vaccine.
which of the following situations?
(A)Lower respiratory tract disease caused by respiratory 15. Each of the following statements concerning mumps
syncytial virus in infants is correct except
(B)Congenital rubella syndrome (A)Mumps virus is a paramyxovirus and hence has a
(C)Aseptic meningitis caused by mumps infection single-stranded RNA genome.
(D)Pneumonia caused by measles virus in adults (B)Meningitis is a recognized complication of mumps.
(E)Encephalitis related to Nipah virus (C)Mumps orchitis in children before puberty oſten
(F)All of the above
causes sterility.
(D)During mumps, the virus spreads through the
11. Reverse transcription polymerase chain reaction (RT-
bloodstream (viremia) to various internal organs.
PCR) assays are useful in diagnosis of paramyxovirus
infections. Which of the following statements about RT-
16. Each of the following statements concerning
PCR is not accurate?
subacute sclerosing panencephalitis is correct except
(A)More sensitive assay than virus isolation
(A) Immunosuppression is a frequent predisposing
(B)Can identify virus strains
factor.
(C)More rapid assay than antigen detection
(B) Aggregates of helical nucleocapsids are found in
(D) Can provide data about genetic variation for
infected cells.
molecular epidemiology studies
(C) High titers of measles antibody are found in
(E) More specific assay for parainfluenza viruses than
cerebrospinal fluid.
serology
(D) Slowly progressive deterioration of brain function
occurs.
12. Each of the following statements concerning measles
vaccine is correct except
17. Which one of the following is the best evidence on
(A) The vaccine contains live, attenuated virus.
which to base a decisive diagnosis of acute mumps
(B)The vaccine should not be given at the same time as
disease?
the mumps vaccine because the immune system cannot
(A)A positive skin test result
respond to two viral antigens given simultaneously.
(B)A fourfold rise in antibody titer to mumps antigen
(C)Virus in the vaccine contains only one serotype.
(C)A history of exposure to a child with mumps
(D)The vaccine should not be given before 15 months of
(D)Orchitis in young adult man
age because maternal antibodies can prevent an
immune response.

13. Each of the following statements concerning rubella


is correct except
(A)Congenital abnormalities occur primarily when a
pregnant woman is infected during the first trimester.
(B)Women who say that they have never had rubella can,
nevertheless, have neutralizing antibody in their serum.
(C)In a 6-year-old child, rubella is a mild, self-limited
disease with few complications.
(D)Acyclovir is effective in the treatment of congenital
rubella syndrome.
18. Which one of the following statements concerning Chapter XLI: Coronaviruses
mumps is correct?
(A)Although the salivary glands are the most obvious Review Questions
sites of infection, the testes, ovaries, and pancreas can 1. A 63-year-old woman develops fever, headache,
be involved as well. malaise, myalgia, and cough. It is early in the winter
(B)Because there is no vaccine against mumps, passive respiratory virus season, and the patient’s physician does
immunization is the only means of preventing the not know what viruses are present in the community.
disease. Which of the following viruses is not a cause of acute
(C)The diagnosis of mumps is made on clinical grounds respiratory disease?
because the virus cannot be grown in cell culture and (A)Influenza virus
serologic tests are inaccurate. (B)Adenovirus
(D)Second episodes of mumps can occur because there (C)Respiratory syncytial virus
are two serotypes of the virus, and protection is type (D)Coronavirus
specific. (E)Rotavirus

19. Which of the following statements is more likely to 2. Based on sequence analysis and serologic assays, the
be true of measles (rubeola) than German measles most likely origin of the SARS coronavirus is which of the
(rubella)? following?
(A)Koplik spots are present. (A)Recombination between a human and an animal
(B)It causes birth defects. coronavirus that created a new virus
(C)It causes only a mild illness. (B)Jump of an animal coronavirus into humans
(D)Human beings are the only natural host. (C)Mutation of a human coronavirus that resulted in
(E)Attenuated virus vaccine is available for prevention. increased virulence
(D)Acquisition of human cellular genes by a human
coronavirus via recombination that allowed viral evasion
of the host immune response

3. The coronavirus SARS epidemic of 2002–2003 resulted


in many cases and deaths. What is the primary route of
transmission of human coronaviruses?
(A)Fecal–oral
(B)Respiratory
(C)Blood
(D)Perinatal mother-to-infant
(E)Sexual activity

4. Coronavirus infections in humans usually cause a


common cold syndrome. However, a recent outbreak of
SARS was characterized by pneumonia and progressive
respiratory failure. The prevention or treatment of these
diseases can be accomplished by
(A)A subunit vaccine
(B)A cold-adapted live-attenuated vaccine
(C)The antiviral drug amantadine
(D)Infection control measures, including isolation and
wearing of protective gear
(E)The antiviral drug acyclovir
5. An epidemic of acute respiratory virus infections Chapter XLII: Rabies, Slow Virus Infections and Prion
occurred among the elderly residents of a nursing home. Diseases
Influenza viruses and coronaviruses, which can cause
serious respiratory disease in the elderly, are suspected. Review Questions
Which of the following characteristics is shared by these 1. Rabies virus is rapidly destroyed by
viruses? (A)Ultraviolet radiation
(A)Segmented genome (B)Heating at 56°C for 1 hour
(B)Infectious RNA genome (C)Ether treatment
(C)High frequency of recombination during replication (D)Trypsin treatment
(D)Single serotype infects humans (E)All of the above
(E)Negative-sense genome
2. Prions are readily destroyed by
6. The following are common characteristics of (A)Ionizing radiation
coronaviruses, except for one. Which is not correct? (B)Formaldehyde
(A)Possess cross-reactive antigens with influenza viruses (C)Boiling
(B)Contain the largest genomes among RNA viruses (D)Proteases
(C)Can cause gastroenteritis (E)None of the above
(D)Are distributed worldwide
3. The presence in neurons of eosinophilic cytoplasmic
7. SARS coronavirus shares some characteristics, but not inclusion bodies, called Negri bodies, is characteristic of
all, with human coronavirus HCoV-OC43. Which of the which of the following central nervous system
following statements is true for SARS coronavirus? infections?
(A)Causes annual outbreaks during the winter (A)Borna disease
(B)Is distributed worldwide (B)Rabies
(C)Populations at high risk of disease included health (C)Subacute sclerosing panencephalitis
care workers (D)New variant Creutzfeldt-Jakob disease
(D)Natural hosts are palm civets (E)Postvaccinal encephalitis

8. A traveler returning from Mecca presents with 4. Which of the following statements about rabies
pneumonia, fever, and cough. What is the best test to vaccines for human use is true?
diagnose MERS coronavirus? (A)Contain live, attenuated rabies virus
(A)Coronavirus antigen assay (B)Contain multiple antigenic types of rabies virus
(B)Human coronavirus PCR (C)Can treat clinical cases of rabies
(C)MERS-CoV PCR (D)Can be used for postexposure prophylaxis
(D)Respiratory viral culture (E)They are associated with Guillain-Barre syndrome

9. Risk factors for severe MERS coronavirus infection


include which of the following?
(A)Recent camel exposure
(B)Prior coronavirus infection
(C)Seasonal allergies
(D)Chronic obstructive pulmonary disease
5. A 22-year-old man is a resident of a small town near aſter developing dysphagia, hypersalivation, agitation,
London. He likes to eat beefsteak. He develops a severe
and generalized muscle twitching. Vital signs and blood
progressive neurologic disease characterized by
tests were normal, but within hours the patient became
psychiatric symptoms, cerebellar signs, and dementia.
confused. The consulting neurologist suspected rabies.
Probable bovine spongiform encephalopathy (BSE) is
Rabies immune globulin, vaccine, and acyclovir were
diagnosed. New variant Creutzfeldt-Jakob disease in
administered. The patient was placed on mechanical
humans and BSE appear to be caused by the same agent.
ventilation the following day. Renal failure developed,
Which of the following statements is true of both
and the patient died 3 days later. Rabies test results were
diseases?
positive. The patient’s wife reported the patient had
(A)Immunosuppression of the host is a predisposing
suffered no bites by dogs or wild animals. The most likely
factor.
explanation for treatment failure is
(B)It is an immune-mediated degenerative neurologic
(A)The rabies test results were falsely positive and the
disorder.
patient did not have rabies.
(C)There is a long incubation period (months to years)
from time of exposure to appearance of symptoms. (B)Treatment was initiated aſter the onset of clinical
(D)The agent is recoverable only from the central symptoms of rabies.
nervous system of an infected host. (C)The vaccine was directed against dog rabies and the
(E)The interferon response persists throughout the patient was infected with bat rabies.
incubation period. (D)The rabies immune globulin should not have been
(F)There is a high-titer antibody response toward PrPSc administered as it interfered with the vaccine.
protein of the agent. (E)Interferons—and not the treatment regimen
administered—are the treatment of choice once rabies
6. Rabies virus has a wide host range and the ability to symptoms develop.
infect all warm-blooded animals, including humans.
Which statement about the epidemiology of human 9. Which of the following animals is most commonly
rabies is true? reported rabid in the United States?
(A)Africa accounts for the majority of rabies fatalities. (A)Squirrels
(B)Dog bites cause most cases of human rabies in (B)Raccoons
England. (C)Rabbits
(C)Domestic animals are the source of most human (D)Swine
rabies in the United States. (E)Rats
(D)Human-to-human rabies transmission places medical
personnel at serious risk. 10. A runner reports an “unprovoked bite” from a
(E)Bat rabies has caused most human rabies cases in the neighborhood dog. The dog was captured by local animal
United States since the 1990s. control authorities, and it appears healthy. What is the
appropriate action?
7. Infectious scrapie agent can be detected in amyloid (A)Confine and observe the dog for 10 days for signs
plaques in infected brains of sheep and hamsters. Te suggestive of rabies.
genome of the infectious agent is characterized by which (B)Begin postexposure prophylaxis of the bitten person.
of the following nucleic acid types? (C)Immediately euthanize the dog.
(A)Negative-sense, single-stranded RNA (D)Because canine rabies has been eliminated in the
(B)Small interfering RNA, smallest known infectious RNA United States, dog bites are no longer an indication for
(C)DNA copy of RNA genome, integrated in postexpo-sure prophylaxis, and no further action is
mitochondrial DNA needed.
(D)Single-stranded, circular DNA (E)Test the dog for rabies antibody.
(E)No detectable nucleic acid
11. The slow virus disease that most clearly has
8. A 49-year-old man visited a neurologist aſter 2 days of immunosuppres-sion as an important factor in its
pathogenesis is
increasing right arm pain and paresthesias. The
(A)Progressive multifocal leukoencephalopathy
neurologist diagnosed an atypical neuropathy. The
(B)Subacute sclerosing panencephalitis
symptoms increased and were accompanied by hand
(C)Creutzfeldt-Jakob disease
spasms and sweating on the right side of the face and
(D)Scrapie
trunk. The patient was admitted to the hospital the day
12. Scrapie and kuru possess all of the following Chapter XLIII: Human Cancer Viruses
characteristics except Review Questions
(A)A histologic picture of spongiform encephalopathy 1. Viruses can cause cancer in animals and humans. A
(B)Transmissibility to animals associated with a long principle of viral carcinogenesis is that
incubation period (A)Retroviruses cause most types of human cancer.
(C)Slowly progressive deterioration of brain function (B)Not all infections with a human cancer virus lead to
(D)Prominent intranuclear inclusions in tumor formation.
oligodendrocytes (C)Short latent periods elapse between time of virus
infection and tumor appearance.
13. A 5-year-old boy in San Francisco reaches into a car (D)Animal models seldom predict cellular mechanisms in
to pet another family’s dog and is bitten on the finger. Six human cancer.
weeks aſter the bite, the child develops fever, headache, (E)Host factors are insignificant in influencing the
and a seizure. He becomes combative and hallucinates. development of virus-induced human cancer.
What is the best diagnostic test to perform on the patient
to rule in rabies as a cause of his illness? 2. Cellular oncogenes represent activated genes involved
(A)Detection of serum antirabies antibody in cancer. A second class of cancer genes is involved in
(B)Culture of cerebrospinal fluid for virus cancer development only when both alleles of such a
(C)Direct fluorescent antibody stain of a biopsy from the gene are inactivated. The second class of genes is called
nape of the neck (A)Proto-oncogenes
(D)Brain biopsy (B)T antigen genes
(E)Cerebrospinal fluid antirabies antibody (C)Tumor suppressor genes
(D)Transduced genes
14. Each of the following statements concerning rabies (E)Silent genes
and rabies virus is correct except
(A)The virus has a lipoprotein envelope and single- 3. A 38-year-old woman is diagnosed with cervical
stranded RNA as its genome. cancer. Tis cancer is common worldwide and has a
(B)The virus has a single antigenic type (serotype). sexually transmitted viral etiology. The causative agent
(C)In the United States, dogs are the most common of human cervical cancer is
reservoir. (A)Hepatitis C virus
(D)The incubation period is usually long (several weeks) (B)Hepatitis B virus
rather than short (several days). (C)Human papillomaviruses, high-risk types
(D)Polyomaviruses
15. A 20-year-old man, who for many years had received (E)Herpesviruses
daily injections of growth hormone prepared from
human pituitary glands, develops ataxia, slurred speech, 4. Retroviruses encode an enzyme called reverse
and dementia. At autopsy the brain shows widespread transcriptase. The function of the reverse transcriptase
neuronal degeneration, a spongy appearance due to enzyme is
many vacuoles between the cells, no inflammation, and (A)DNase activity
no evidence of virus particles. The most likely diagnosis (B)RNA-dependent DNA polymerase activity
is (C)DNA-dependent RNA polymerase activity
(A)Herpes encephalitis (D)RNA-dependent RNA polymerase activity
(B)Creutzfeldt-Jakob disease (E)Topoisomerase activity
(C)Subacute sclerosing panencephalitis
(D)Progressive multifocal leukoencephalopathy 5. Two months aſter a kidney transplant, a 47-year-old
(E)Rabies
man developed nephropathy. Up to 5% of renal allograſt
recipients develop nephropathy. A viral cause of some of
the nephropathy cases has been identified as
(A)Polyomavirus BK
(B)Human papillomavirus, all types
(C)Human papillomavirus, low-risk types
(D)Hepatitis C virus
(E)Human cytomegalovirus
(D)All adolescents and young adults, both boys and girls
6. Human papillomavirus can cause cancer in humans (E)Adolescent and young adult females
and is most commonly associated with
(A)Rectal polyps 12. Which of the following best describes available HPV
(B)Breast cancer vaccines?
(C)Prostate cancer (A)Live attenuated virus
(D)Anogenital cancers (B)Live recombinant virus
(E)Mesotheliomas (C)Noninfectious subunit
(D)Toxoid
7. A virus that causes human cancer is also associated
with a nervous system disorder called tropical spastic 13. Many of the oncogenic retroviruses carry oncogenes
paraparesis. That virus is closely related to normal cellular genes, called proto-
(A)Polyomavirus JC oncogenes. Which one of the following statements
(B)Polyomavirus SV40 concerning proto-oncogenes is incorrect?
(C)Herpes simplex virus (A)Several proto-oncogenes have been found in mutant
(D)Human T-lymphotropic virus form in human cancers that lack evidence for viral
(E)Human immunodeficiency virus etiology.
(B)Several viral oncogenes and their progenitor proto-
8. The polyomaviruses encode oncoproteins called T oncogenes encode protein kinases specific for tyrosine.
antigens. These viral gene products (C)Some proto-oncogenes encode cellular growth
(A) Are not needed for virus replication factors and receptors for growth factors.
(B) Interact with cellular tumor suppressor proteins (D)Proto-oncogenes are closely related to transposons
(C) Function to integrate the viral provirus into the found in bacteria.
cellular chromosome
(D) Mutate rapidly to allow the virus to escape immune
clearance by the host
(E) Are not able to transform cells in culture

9. Cancer viruses are classified in several virus families.


Which of the following virus families contains a human
cancer virus with an RNA genome?
(A)Adenoviridae
(B)Herpesviridae
(C)Hepadnaviridae
(D)Papillomaviridae
(E)Flaviviridae

10. Laryngeal papillomas in children are generally caused


by the same viruses that cause benign genital
condylomas. These viruses are
(A)Papillomaviruses, types 6 and 11
(B)Polyomavirus JC
(C)Epstein-Barr virus
(D)Molluscum contagiosum virus
(E)Papillomaviruses, types 16 and 18

11. Vaccines against the most common HPV types that


cause genital infections were approved in 2006 and
2007. They are aimed for use in which of the following
population(s)?
(A)All adults, both men and women
(B)All female adults
(C)Women with precancerous cervical lesions
Chapter XLIV: AIDS and Lentiviruses 6. What are the most common symptoms of acute HIV
infection?
Review Questions (A)Rash and sore throat
1. HIV-1 is classified as a member of the Lentivirus genus (B)Fever and malaise
in the Retroviridae family. Lentiviruses (C)Diarrhea
(A)Contain a DNA genome (D)Jaundice and hepatitis
(B)Cause tumors in mice (E)Neuropsychiatric and behavioral changes
(C)Infect cells of the immune system
(D)Have related sequences endogenous in normal cells 7. A 36-year-old nurse suffered a needlestick with blood
(E)Cause rapidly progressive neurologic disease from an HIV-positive patient. Six months later, the
nurse’s serum was positive in an EIA test, gave equivocal
2. HIV-1 encodes an envelope glycoprotein, gp120. This results in a repeat EIA test, and was negative by Western
protein blot. The nurse
(A)Causes membrane fusion (A)Is probably infected with HIV
(B)Binds to the viral coreceptor on the cell surface (B)Is in the window between acute infection with HIV and
(C)Is highly conserved among different isolates seroconversion
(D)Fails to elicit neutralizing antibody (C)Is probably not infected with HIV
(E)Induces chemokine production (D)May be infected with a drug-resistant strain of HIV
(E)May be a long-term nonprogressor
3. HIV/AIDS has become a worldwide epidemic that
continues to expand. The geographic area with the 8. A 41-year-old HIV-infected male who had refused
antiretrovi-ral therapy is diagnosed with Pneumocystis
largest number of HIV-infected people aſter sub-Saharan
jiroveci infection. Tis patient
Africa is (A)Probably has a CD4 T-cell count below 200 cells/μL
(A)Central and South America and the Caribbean (B)Is at elevated risk for lung cancer
(B)China (C)Is not longer a candidate for HAART
(C)North America (D)Probably has declining levels of plasma viremia
(D)South/Southeast Asia (E)Is unlikely to develop dementia at this stage
(E)Eastern Europe and Central Asia
9. A 48-year-old HIV-positive man with a CD4 count of 40
4. The typical course of an untreated HIV infection complains of memory loss to his doctor. Four months
extends over 10 or more years. There is usually a long later, he becomes paralyzed and dies. An autopsy reveals
period (clinical latency) between the time of primary HIV demyelination of many neurons in the brain, and
infection and the development of AIDS. During this electron microscopy shows clusters of nonenveloped
period of clinical latency viral particles in the neurons. The most likely cause of the
(A)HIV is not detectable in the plasma disease is
(B)CD4 cell counts remain unchanged (A)Adenovirus type 12
(C)Virus replicates at a very low rate (B)Coxsackievirus B2
(D)Virus is present in lymphoid organs (C)Parvovirus B19
(E)Neutralizing antibodies are not elicited (D) Epstein-Barr virus
(E) JC virus
5. Viral coinfections occur in HIV-1-infected individuals
and may contribute to morbidity and mortality. The most 10. Highly active antiretroviral combination therapy for
common coinfection in HIV-1-positive persons in the HIV infection usually includes a protease inhibitor such
United States involves as saquinavir. Such a protease inhibitor
(A)Hepatitis C virus (A)Is effective against HIV-1 but not HIV-2
(B)Hepatitis D virus (B)Seldom gives rise to resistant mutants of HIV
(C)HIV type 2 (C)Inhibits a late step in virus replication
(D)Human T-lymphotropic virus (D)Degrades the CD4 receptor on cells
(E)Kaposi sarcoma herpesvirus (E)Interferes with virus interaction with coreceptor
11. In a person with HIV infection, potentially infectious Chapter XLV: Medical Mycology
fluids include all of the following except
(A)Blood Review Questions
(B)Saliva visibly contaminated with blood 1. Which statement regarding fungi is correct?
(C)Urine not visibly contaminated with blood (A)All fungi are able to grow as yeasts and molds.
(D)Genital secretions (B)Although fungi are eukaryotes, they lack
(E)Amniotic fluid mitochondria.
(C)Fungi are photosynthetic.
12. Of the more than 1 million persons estimated to be (D)Fungi have one or more nuclei and chromosomes.
living with HIV in the United States in 2011, how many (E)Few fungi possess cell membranes.
are thought to be unaware of their infection?
(A)About 5% 2. Which statement regarding fungal growth and
(B)About 10% morphology is correct?
(C)About 20% (A)Pseudohyphae are produced by all yeasts.
(D)About 25% (B)Molds produce hyphae that may or may not be
(E)About 30% partitioned with cross-walls or septa.
(F)About 50% (C)Conidia are produced by sexual reproduction.
(D)Most yeasts reproduce by budding and lack cell walls.
13. Each of the following statements concerning HIV is (E)Most pathogenic dimorphic molds produce hyphae in
correct except the host and yeasts at 30°C.
(A) Screening tests for antibodies and nucleic acid are
useful to prevent transmission of HIV through transfused 3. Which statement regarding fungal cell walls is correct?
blood. (A)The major components of fungal cell walls are
(B)The opportunistic infections seen in AIDS are primarily proteins such as chitin, glucans, and mannans.
the result of a loss of cell-mediated immunity. (B)The cell wall is not essential for fungal viability or
(C)Zidovudine (azidothymidine) inhibits the RNA- survival.
dependent DNA polymerase. (C)Ligands associated with the cell walls of certain fungi
(D)The presence of circulating antibodies that neutralize mediate attachment to host cells.
HIV is evidence that an individual is protected against (D)Fungal cell wall components are the targets for the
HIV-induced disease. major classes of antifungal antibiotics, such as the
polyenes and azoles.
14. Highly active antiretroviral therapy (HAART) is less (E)Fungal cell wall components rarely stimulate an
than ideal because immune response.
(A)It does not eliminate latent HIV infection
(B)Its cost is too great for 90% of AIDS sufferers 4. A 54-year-old man developed a slowly worsening
(C)It oſten has severe side effects headache followed by gradual, progressive weakness in
(D)Some HIV strains are resistant to it his right arm. A brain scan revealed a leſt cerebral lesion.
(E)All of the above At surgery an abscess surrounded by granulomatous
material was found. Sections of the tissue and
15. Each of the following statements concerning HIV is subsequent culture showed darkly pigmented septate
correct except hyphae indicating phaeohyphomycosis. Tis infection may
(A) The CD4 protein on the T-cell surface is one of the be caused by species of which genus below?
receptors for the virus. (A)Aspergillus
(B)There is appreciable antigenic diversity in the (B)Cladophialophora
envelope glycoprotein of the virus. (C)Coccidioides
(C)One of the viral genes codes for a protein that (D)Malassezia
augments the activity of the viral transcriptional (E)Sporothrix
promoter.
(D)A major problem with testing for antibody to the virus
is its cross-reactivity with human T-lymphotropic virus
type 1.
5. A 35-year-old man is a farmer in a tropical area of West (E)No diagnostic significance
Africa. He developed a persistent scaly papule on his leg.
Ten months later a new crop of wart-like purplish scaly 9. Which statement regarding the epidemiology of
lesions appeared. These lesions slowly progressed to a candidiasis is correct?
cauliflower-like appearance. Chromoblastomycosis (A)Patients receiving bone marrow transplants are not at
(chromomycosis) was diagnosed. Which statement risk for systemic candidiasis.
regarding this disease is most correct? (B)Patients with impaired or low numbers of neutrophils
(A)In tissue, the organisms convert to spherical cells that and monocytes are not at risk for systemic candidiasis.
reproduce by fission and exhibit transverse septations. (C)Patients with any form of diabetes have enhanced
(B)The etiologic agents are endogenous members of the resistance to candidiasis.
mammalian flora and possess melanized cell walls. (D)Patients with AIDS frequently develop
(C)The disease is caused by a single species. mucocutaneous candidiasis, such as thrush.
(D) Most infections are systemic. (E)Pregnancy lowers the risk of candidal vaginitis.
(E) Most infections are acute and clear spontaneously.
10. Which statement regarding dermatophytosis is
6. A 42-year-old HIV-positive male, originally from correct?
Vietnam but now residing in Tucson, Arizona, presents (A) Chronic infections are associated with zoophilic
with a painful ulcerative lesion on his upper lip (cheilitis). dermatophytes, such as Microsporum canis.
A biopsy was obtained, and the histopathologic slide (B)Acute infections are associated with zoophilic
(hematoxylin and eosin stain) revealed spherical dermato-phytes, such as Microsporum canis.
structures (20–50 μm in diameter) with thick refractory (C)Chronic infections are associated with anthropophilic
cell walls. What is the likely disease consistent with this der-matophytes, such as Microsporum canis.
finding? (D)Acute infections are associated with anthropophilic
(A)Infection with Penicillium marneffei der-matophytes, such as Microsporum canis.
(B)Cryptococcosis
(C)Blastomycosis 11. Which statement regarding the laboratory
(D)Coccidioidomycosis identification of fungi is correct?
(E)No diagnostic significance (A)Histoplasma capsulatum typically requires less than
48 hours of incubation to yield positive cultures from
7. A 47-year-old man with poorly controlled diabetes clinical specimens.
mellitus developed a bloody nasal discharge, facial (B)Since many saprobic (nonpathogenic) molds resemble
edema, and necrosis of his nasal septum. Culture of his dimorphic mycotic agents in culture at 30°C, the
cloudy nasal secretions yielded Rhizopus species. What is identification of putative dimorphic pathogenic fungi
the most important implication of this finding? must be confirmed by conversion to the tissue form in
(A)No diagnostic value because this mold is an airborne vitro or by the detection of species-specific antigens or
contaminant. DNA sequence analysis.
(B)Consider treatment for rhinocerebral mucormycosis (C)Molds are routinely speciated by a battery of
(zygomycosis). physiologic tests, such as the ability to assimilate various
(C)Strongly suggestive of ketoacidosis. sugars.
(D)Strongly suggestive of HIV infection. (D)A positive germ tube test provides a rapid
(E)The patient has been exposed to indoor mold presumptive identification of Candida glabrata.
contamination. (E)Budding yeast cells and abundant pseudohyphae are
typical of Pneumocystis jiroveci.
8. An 8-year-old boy develops a circular dry, scaly, and
pruritic lesion on his leg. What is the diagnostic
significance of observing branching, septate,
nonpigmented hyphae in a potassium
hydroxide/calcofluor white preparation of a scraping
from this skin lesion?
(A)Chromomycosis
(B)Dermatophytosis
(C)Phaeohyphomycosis
(D)Sporotrichosis
12. A 28-year-old female sex worker from southern (A)Aspergillosis
California complained of headaches, dizziness, and (B)Candidiasis
occasional episodes of “spacing out” during the past 2 (C)Hyalohyphomycosis
weeks. A lumbar puncture revealed reduced sugar, (D)Phaeohyphomycosis
elevated protein, and 450 mononuclear leukocytes per (E)No diagnostic significance
milliliter. She was seropositive for HIV. Her history is
compatible with fungal meningitis due to Cryptococcus 16. A middle-aged male resident of southern California
neoformans, Coccidioides posadasii, or a species of received a liver transplant. During the following months,
Candida. Which one of the following tests is he gradually developed fatigue, weight loss, cough, night
confirmatory? sweats, dyspnea, and a nonhealing subcutaneous nodule
(A)Meningitis due to Coccidioides posadasii would be on his nose. The chest radiograph revealed hilar
confirmed by a positive test of the CSF for cryptococcal lymphadenopathy and diffuse infiltrates. Direct
capsular antigen. examination and culture of a respiratory specimen were
(B)Meningitis due to Cryptococcus neoformans would be negative. Skin tests with PPD, blastomycin, coc-cidioidin,
confirmed by a positive test of the CSF for complement and histoplasmin were negative. Serologic test results
fixation antibodies to coccidioidin. were as follows: Negative serum test for cryptococcal
(C)Meningitis due to a species of Candida would be capsular antigen in blood, positive immunodiffusion test
confirmed by the microscopic observation of oval yeast for serum pre-cipitins to fungal antigen F, and negative
cells and pseu-dohyphae in the CSF. immunodiffusion tests for precipitins to antigens h, m,
(D)Meningitis due to Coccidioides posadasii would be and A. Serum tests for fungal complement fixation
confirmed by a positive skin test to coccidioidin. antibodies were negative for Blastomyces dermatitidis,
as well as both the mycelial and yeast antigens of
13. Which statement about phaeohyphomycosis is Histoplasma capsulatum but yielded a titer of 1:32 to
correct? coccidi-oidin. Which interpretation of these data is the
(A)The infection only occurs in immunocompetent most tenable?
patients. (B)Infected tissue reveals branching, septate (A)Clinical and serologic findings are inconclusive.
nonpigmented hyphae. (B)Clinical and serologic findings are most consistent
(C)The causative agents are members of the normal with active disseminated histoplasmosis.
microbial flora and can be isolated readily from the skin (C)Clinical and serologic findings are most consistent
and mucosa of healthy persons. with active disseminated blastomycosis.
(D)Phaeohyphomycosis may exhibit several clinical (D)Clinical and serologic findings are consistent with
manifestations, including subcutaneous or systemic active disseminated coccidioidomycosis.
disease, as well as sinusitis. (E)Clinical and serologic findings exclude a diagnosis of
(E)Cases rarely respond to treatment with itraconazole. blas-tomycosis, histoplasmosis, and coccidioidomycosis.

14. A 37-year-old man with AIDS, currently living in 17. Which statement regarding aspergillosis is correct?
Indianapolis, Indiana, presented with osteomyelitis of (A)Patients with allergic bronchopulmonary aspergillosis
the leſt hip. A needle biopsy of the bone marrow was rarely have eosinophilia.
(B)Patients receiving parenteral corticosteroids are not
obtained, and the calcofluor white smear revealed a
at risk for invasive aspergillosis.
variety of myelogenous cells, monocytes, and
(C)The diagnosis of pulmonary aspergillosis is frequently
macrophages containing numerous intracellular yeast
established by culturing Aspergillus from the sputum and
cells that were elliptical and approximately 2 × 4 μm.
blood.
What is the most likely diagnosis?
(D)The clinical manifestations of aspergillosis include
(A)Blastomycosis
local infections of the ear, cornea, nails, and sinuses.
(B)Candidiasis
(E)Bone marrow transplant recipients are not at risk for
(C)Cryptococcosis
invasive aspergillosis.
(D)Histoplasmosis
(E)No diagnostic significance

15. The potassium hydroxide examination of sputum


from a heart transplant patient with fever and
pulmonary infiltrates contains oval budding yeast cells
and pseudohyphae. What is the diagnostic significance?
18. Which statement regarding sporotrichosis is correct? (D)The vast majority of patients with active disease are
(A)The most common etiologic agent is Pseudallescheria males.
boy-dii (Scedosporium apiospermum). (E)The etiologic agent is inherently resistant to
(B)The etiologic agent is a dimorphic fungus. amphotericin B.
(C)The ecology of the etiologic agent is unknown.
(D)Most cases are subcutaneous and nonlymphangitic. 23. Your kidney transplant patient has developed
(E)Most patients are immunocompromised. nosocomial systemic candidiasis, but the patient’s isolate
of Candida glabrata is resistant to fluconazole. A
19. A 24-year-old, HIV-negative migrant worker from reasonable alternative would be oral administration of:
Colombia presented with a painful ulcerative lesion on (A)Flucytosine
the tongue. The edge of the lesion was gently scraped (B)Posaconazole
and a calcofluor white-potassium hydroxide smear (C)Griseofulvin
revealed tissue cells, debris, and several large, spherical, (D)Amphotericin B
multiply budding yeast cells. Based on this observation, 24. Which one of the following antifungal drugs does not
what is the most likely diagnosis? target the biosynthesis of ergosterol in the fungal
(A)Blastomycosis membrane?
(B)Candidiasis (A)Voriconazole
(C)Coccidioidomycosis (B)Itraconazole
(D)Histoplasmosis (C)Terbinafine
(E)Paracoccidioidomycosis (D)Fluconazole
(E)Micafungin
20. Which statement about blastomycosis is correct?
(A)Similar to other endemic mycoses, this infection 25. Which one of the following pathogenic yeasts is not
occurs equally in men and women. a common member of the normal human flora or
(B)Infection starts in the skin, and the organisms microbiota?
commonly disseminate to the lungs, bone, genitourinary (A)Candida tropicalis
tract, or other sites. (B)Malassezia globosa
(C)The disease is endemic to certain areas of South (C)Cryptococcus neoformans
America. (D)Candida glabrata
(D)In tissue, one finds large, thick-walled, single budding (E)Candida albicans
yeast cells with broad connections between the parent
yeast and bud.
(E)All cases require treatment with amphotericin B.

21. Which statement regarding dermatophytosis is


correct?
(A)Chronic infections are associated with zoophilic
dermatophytes, such as Trichophyton rubrum.
(B)Acute infections are associated with zoophilic
dermatophytes, such as Trichophyton rubrum.
(C)Chronic infections are associated with anthropophilic
dermatophytes, such as Trichophyton rubrum.
(D)Acute infections are associated with anthropophilic
dermatophytes, such as Trichophyton rubrum.

22. Which statement regarding paracoccidiomycosis is


not correct?
(A) The etiologic agent is a dimorphic fungus.
(B)Most patients acquired their infections in South
America.
(C)Although the infection is acquired by inhalation and is
initiated in the lungs, many patients develop cutaneous
and mucocutaneous lesions.
Chapter XLVI: Medical Parasitology (A) Brugia malayi
(B) Mansonella ozzardi
Review Questions (C)Dracunculus medinensis
1. A mother states that she has observed her 4-year-old (D)Wuchereria bancroſti
son scratching his anal area frequently. The most likely (E)Onchocerca volvulus
cause of this condition is
(A)Trichomonas vaginalis 6. An 18-year-old male complains of abdominal pain,
(B)Enterobius vermicularis bloating, frequent loose stools, and loss of energy. He
(C)Ascaris lumbricoides returned a month ago from a 3-week hiking and camping
(D)Necator americanus trek to the Mount Everest Base Camp in Nepal. Te trek
(E)Entamoeba histolytica involved only high-elevation hiking, since he flew in and
out of the 12,000-ſt starting point. Which of the following
2. Chagas disease is especially feared in Latin America
because of the damage that can occur to the heart and is an important consideration for the diagnosis?
parasympathetic nervous system and the lack of an (A)Exposure to high-level UV radiation
effective drug for the symptomatic later stages. Your (B)The source and purification of water
patient is planning to reside in a Venezuelan village for (C)The use of insect repellents while hiking
1–2 years. Which one of the following suggestions would (D)The presence of domestic animals en route
be of special value for avoiding Chagas disease? (E)The degree of contact with villagers en route
(A)Boil or treat all of your drinking water.
(B)Sleep under a bed net. 7. Which one of the following diagnostic tests should be
(C)Do not keep domestic pets in your house. conducted for the patient in Question 6?
(D)Never walk barefoot in the village compound. (A)Blood and urine bacteriologic examination
(E)Do not eat lettuce or other raw vegetables or (B)Series of ova and parasite tests and fecal smears
unpeeled fruit. (C)ELISA or hemagglutination serologic tests for malaria
(D)Skin snip microfilarial test
3. A sexually active 24-year-old woman complains of (E)Endoscopic exam for whipworms
vaginal itching and vaginal discharge. To verify your
tentative diagnosis of trichomoniasis, you should include 8. The parasite most likely to be responsible for the
which of the following in your workup? illness of the patient in Question 6 is
(A) Specific serologic test (A)Leishmania major
(B) Ova and parasite fecal smear (B)Plasmodium vivax
(C) Wet mount of vaginal fluid (C)Trichomonas vaginalis
(D) Enzyme-linked immunoassay (ELISA) test of serum (D)Naegleria gruberi
(E) Stool culture (E)Giardia lamblia

4. You are working in a rural medical clinic in China and a 9. Several Papua New Guinea villagers known to eat pork
3-year-old girl is brought in by her mother. The child during celebrations were reported to be suffering from
appears emaciated and, upon testing, is found to have a an outbreak of epileptiform seizures. One of the first
hemoglobin level of 5 g/dL. Her feet and ankles are things you should investigate is
swollen, and there is an extensive rash on her feet, (A)The prevalence of Ascaris infections in the population
ankles, and knees. Te most likely parasitic infection that (B)The prevalence of schistosomiasis in the population
causes the child’s condition is (C)The presence of Trypanosoma brucei gambiense in
(A)Schistosomiasis the villagers
(B)Cercarial dermatitis (D)The presence of Giardia cysts in the drinking water
(C)Cyclosporiasis (E)The presence of Taenia solium in the pigs
(D)Hookworm infection
(E)Trichuriasis 10. A 32-year-old male tourist traveled to Senegal for 1
(F)Ascariasis month. During the trip, he swam in the Gambia river.
Two months aſter his return, he began complaining of
5. Pathologic effects of filariae in humans are caused by intermittent lower abdominal pain with dysuria.
the adult worms in all but one species. In this case, the Laboratory results of ova and parasites revealed eggs
principal damage is caused by the microfilariae of
with a terminal spine. Which of the following parasites is (F)Echinococcus granulosus
the cause of the patient’s symptoms? (G)Paragonimus westermani
(A)Toxoplasma gondii
(B)Schistosoma mansoni 15. An apparently fatigued but alert 38-year-old woman has
(C)Schistosoma haematobium spent 6 months as a teacher in a rural Thailand village
(D)Ascaris lumbricoides school. Her chief complaints include frequent headaches,
(E)Taenia solium occasional nausea and vomiting, and periodic fever. You
suspect malaria and indeed find parasites in red blood cells
in a thin blood smear. To rule out the dangerous falciparum
11. What type of specimen was collected for laboratory
form of malaria, which one of the following choices is NOT
analysis based on the answer in the previous question?
consistent with a diagnosis of Plasmodium falciparum
(A)Tick blood smear
malaria based on a microscopic examination of the blood
(B)Stool sample
smear?
(C)Urine sample (A)Red blood cells containing trophozoites with Schuffner’s
(D)Blood for serology dots
(E)Sputum sample (B)Red blood cells containing >1 parasite per RBC
(C)Banana-shaped or crescent-shaped gametocytes
12. A previously healthy 23-year-old woman recently (D)Parasites within normal-sized red blood cells
returned from her vacation aſter visiting friends in (E)Parasites with double nuclei
Arizona. She complained of severe headaches, saw
“flashing lights,” and had a purulent nasal discharge. She 16. Given a diagnosis of uncomplicated Plasmodium
falciparum malaria for the patient in Question 15, which
was admitted into the hospital with a diagnosis of
one of the following treatment regimens is appropriate
bacterial meningitis and died 5 days later. Which of the
where chloroquine-resistance is known?
following parasites should have been considered in the
(A)Oral artemisinin-based combination therapy (ACT)
diagnosis? She had no prior history of travel outside of (B)Oral chloroquine
the United States. (C)Intravenous chloroquine
(A)Plasmodium falciparum (D)Oral proguanil
(B)Toxoplasma gondii (E)Intravenous quinidine
(C)Strongyloides stercoralis
(D)Entamoeba histolytica 17. Given a diagnosis of Plasmodium falciparum, you should
(E)Naegleria fowleri tell the patient in Question 15 that (select one)
(A)Relapse occurs with Plasmodium vivax and Plasmodium
13. How could the person have acquired the parasite in ovale, not Plasmodium falciparum and therefore no
Question 12? treatment for hypnozoites is necessary.
(A)Ingesting cysts from fecally contaminated drinking (B)Primaquine is used to prevent relapse of Plasmodium
water falciparum.
(B)Eating improperly cooked fish (C)Returning to the tropics would be dangerous because
(C)Eating improperly cooked beef hypersensitivity to the parasite may have developed.
(D)Walking barefoot in the park (D)The use of insecticide treated bednets in endemic areas
(E)Engaging in unprotected sexual intercourse is not necessary since she already had malaria.
(F)Getting bitten by a sandfly (E)It is not necessary for her to take antimalarials when
(G)Plunging into a natural hot spring traveling in endemic areas.

14. A 37-year-old sheep farmer from Australia presents with 18. A 52-year-old male, returning from a travel tour in India
upper right quadrant pain and appears slightly jaundiced. A and Southeast Asia, was diagnosed with intestinal
stool exam was negative for ova and parasites but a CT scan amebiasis and successfully treated with iodoquinol. A
of the liver reveals a large 14-cm cyst that appears to month later, he returned to the clinic complaining of the
contain fluid. Which of the following parasites should be following conditions. Which of the conditions is most likely
considered? the result of systemic amebiasis (even though the intestinal
(A)Toxoplasma gondii infection appears to be cured)?
(B)Taenia solium (A)High periodic fever
(C)Taenia saginata (B)Bloody urine
(D)Clonorchis sinensis (C)Tender, enlarged liver
(E)Schistosoma mansoni (D)Draining skin lesion
(E)Enlarged painful spleen
Chapter XLVII: Principles of Diagnostic Medical 4. A 42-year-old man is known to have HIV/AIDS. Which
Microbiology of the following is the most appropriate method to
follow the progress of his highly active antiretroviral
Review Questions therapy (HAART)?
1. A 47-year-old woman had a bone marrow transplant (A) Determination of viral load
as part of her treatment for chronic myelogenous (B) Following anti-HIV-1 antibody levels
leukemia. While in the hospital she had a central venous (C)Using Western blot to assess his anti-p24 levels
catheter in place for administration of fluids. In the time (D)Repeated culture of his blood for HIV-1 to determine
when the culture becomes negative
following the transplant, but before it had engraſted, the (E)Genotyping of his HIV-1 isolate to determine its
patient had a very low white blood cell count. She antiretroviral susceptibility
developed a fever, and blood cultures were done. Which
of the following scenarios suggests that the positive 5. A 2-year-old child develops diarrhea. Rotavirus
blood cultures resulted from a contaminant? infection is suspected. Which of the following would be
(A)Two positive peripheral vein blood cultures with most useful in diagnosing a rotavirus infection?
Staphylococcus aureus (A)Fluorescent antibody staining of the stool specimen
(B)Two positive peripheral vein blood cultures with (B)Light microscopy to detect mucosal cells with
Staphylococcus epidermidis along with two positive cytopathic effect
central line blood cultures with Staphylococcus (C)Detection of virus antigen in stool by enzyme-linked
epidermidis immunosorbent assay
(C)One positive peripheral vein blood culture and one (D)Virus culture
positive central line blood culture with Escherichia coli
(D)One positive central venous line blood culture with a 6. Which of the following is appropriate to determine the
Corynebacterium species and two negative peripheral etiologic diagnosis of infection?
vein blood cultures (A)Culture and identification of the agent
(E)Two positive central line blood cultures with Candida (B)DNA–DNA or DNA–RNA hybridization to detect
albicans pathogen-specific genes in patients’ specimens
(C)Demonstration of a rise in antibody titer or cell-
2. Two days ago, a 22-year-old man returned from a 2- mediated immune response to an infectious agent
week trip to Mexico. Within 24 hours, he developed (D)Morphologic identification of the agent in stains of
diarrhea. Which of the following will not establish the specimens or sections of tissues by light or electron
etiology of his diarrhea? microscopy
(A)Stool culture for Salmonella, Shigella, and (E)Detection of antigen from the agent by immunologic
Campylobacter assay
(B)Stool culture for rotavirus and Norwalk-like virus (F)All of the above
(C)Stool enzyme immunoassay for Giardia lamblia
antigen 7. A 45-year-old woman is admitted to the hospital
(D)Stool examination for Entamoeba histolytica because of fever, a 6-kg weight loss, and a new heart
murmur. Probable endocarditis is diagnosed. How many
3. A 37-year-old man traveled to Peru during the time of blood cultures over what period of time should be done
the cholera epidemic. One day aſter returning home, he to provide evidence of specific bacterial infection in
developed severe watery diarrhea. To enhance the endocarditis?
isolation of Vibrio cholerae from his stool, the laboratory (A)One
needs to include: (B)Two over 10 minutes
(A)MacConkey agar (C)Three over 2 hours
(B)Campylobacter blood agar (D)Three over 24 hours
(C)Tiosulfate citrate bile salts sucrose agar (E)Six over 3 days
(D)Bismuth sulfite agar
(E)Hektoen agar
8. A 4-year-old boy develops bloody diarrhea. 12. In August, a 2-year-old boy presents acutely with
Hemorrhagic colitis due to Escherichia coli O157:H7 is fever, signs of headache, decreased mental status, and
suspected. What medium should be inoculated to help neck stiffness. On physical examination the fever is
the laboratory staff make the diagnosis of this infection? confirmed, mild nuchal rigidity is present, and although
(A)Blood agar the child is irritable and mildly somnolent, he is arousable
(B)Sorbitol MacConkey agar and is taking some oral fluids. The cerebrospinal fluid
(C)Hektoen enteric agar parameters reveal a protein of 60 μg/dL, glucose of 40
(D)CIN (cefsulodin, irgasan, novobiocin) agar μg/dL, and a total of 200 WBCs, predominately
(E)Tiosulfate citrate bile salts sucrose agar mononuclear. The most likely cause of this child’s
infection is:
9. A 43-year-old black man frequently drove his long-haul (A)Bacterial
18-wheel truck through the Central Valley of California. (B)Viral
Two months ago there was a major windstorm while he (C)Protozoan
was driving through the Valley. Two weeks aſter that he (D)Fungal
(E)Mycobacterial
developed fever with a cough and pleuritic chest pain. An
infiltrate was seen on chest x-ray. Pneumonia was
13. In the case above, the most useful test for making a
diagnosed, and the patient was given erythromycin. Te
rapid definitive diagnosis of the most likely causative
fever, cough, pleuritic pain, and infiltrate cleared over a
agent is:
3-week period. Two weeks ago, he developed
(A)An antigen test for Streptococcus pneumoniae
progressively severe headaches, and for the past 2 days
(B)A latex agglutination test for cryptococcal antigen
he has had vomiting. His cerebrospinal fluid contains 150
(C)A nucleic acid amplification test for viral RNA
white blood cells per microliter, predominantly
detection
lymphocytes, and the glucose concentration is low.
(D)Culture on selective media combined with a probe
Meningitis due to Coccidioides immitis is suspected.
test for confirmation
Which of the following tests is the most sensitive and
(E)Giemsa-stained smear of cerebrospinal fluid
useful in confirming this diagnosis?
(A)Test for coccidioidal antigens performed on CSF
14. Susceptibility testing using an MIC method, as
(B)Test of cerebrospinal fluid for antibodies against
opposed to disk diffusion, is preferred for all of the
Coccidioides immitis
following types of infections except:
(C)PCR for Coccidioides immitis DNA
(A)Urinary tract infections
(D)Cerebrospinal fluid culture for Coccidioides immitis
(B)Endocarditis
(E)Complement fixation test of serum for antibodies
(C)Osteomyelitis
against Coccidioides immitis
(D)Bacteremia in a neutropenic patient
(E)Bacterial meningitis
10. A 5-year-old kidney transplant patient being treated
with cyclosporine develops a lymphoproliferative
15. Bacterial vaginosis is best diagnosed by all of the
disorder. Which of the following viruses is most likely
following except:
responsible for this disorder?
(A)Measurement of the vaginal pH
(A)Cytomegalovirus
(B)Detection of a fishy odor when the discharge is
(B)Herpes simplex virus
alkalinized with KOH
(C)Coxsackie B virus
(C)Bacterial culture for aerobes and anaerobes
(D)Hepatitis B virus
(D)Examination of a Gram-stained smear for “clue cells”
(E)Epstein-Barr virus
16. A 45-year-old man presents to the emergency room
11. All of the following are appropriate indications for the
with a 3-day history of cough productive of blood-tinged
use of serologic tests for viruses except:
sputum and a fever. The Gram stain of the sputum has
(A)As an indication of one’s susceptibility to a particular
many white blood cells and gram-positive diplococci. The
viral infection
most likely causative organism is
(B)For diagnosis when the virus has a long incubation
(A)Staphylococcus aureus
period
(B)Streptococcus pneumoniae
(C)For screening purposes
(C)Mycoplasma pneumoniae
(D)For confirmation of a viral infection
(D)Klebsiella pneumonia
(E)To monitor the response to treatment
17. How many organisms must be present in a clean
catch midstream urine sample to be considered
indicative of an infection?
(A)> 102 CFU/ml
(B)> 103 CFU/ml
(C)> 104 CFU/ml
(D)> 105 CFU/ml

18. Common contaminants of blood cultures include:


(A)Gram-negative rods
(B)Coagulase-negative staphylococci
(C)Staphylococcus aureus
(D)Anaerobes

19. Which of the following specimens does not usually


contain anaerobes?
(A)Aspiration from an infected maxillary sinus
(B)Throat swab from a patient with a sore throat
(C)Cerebrospinal fluid from a patient with meningitis
(D)Expectorated sputum from a patient with community-
acquired pneumonia

20. The proportion of antibiotic resistant bacteria has


increased along with the widespread use of antibiotics.
Tis is due to the fact that antibiotics:
(A)Are unstable in vivo
(B)Act as agents of selection for resistant organisms
(C)Are mainly bacteriostatic in vivo
(D)Are powerful mutagens

You might also like